Sunteți pe pagina 1din 70

Abc_INSURANCE CASE DIGEST

CONSTRUCTION OF INSURANCE CONTRACT


Calanoc vs. Court of Appeals, 98 Phil. 79 [1955]

Facts:
Facts: Melencio Basilio, a watchman and, secured a life insurance policy from the Philippine American
Insurance Company in the amount of P2,000 which had a supplemental contract covering death by
accident. He later died from a gunshot wound on the occasion of a robbery committed;

his widow was paid P2,000 for the face value of the policy but the company refused to give the other
2000 for the death by accident because the deceased died by murder during the robbery and while
making an arrest as an officer of the law which were expressly excluded in the contract. The Court of
Appeals upheld the Company and said that the circumstances surrounding Basilio’s death was caused
by one of the risks excluded by the supplementary contract which exempts the company from liability.

Issue: Is the Philippine American Life Insurance Co. liable to the petitioner for the amount covered by
the supplemental contract?

Held: Yes.

The circumstances of Basilio’s death cannot be taken as purely intentional on the part of
Basilio to expose himself to the danger. No proof that his death was the result of intentional killing
because there is the possibility that the malefactor had fired the shot merely to scare away the people
around.

The terms and phraseology of the exception clause should be clearly expressed within the
understanding of the insured. Art. 1377 of the New Civil Code provides that in case ambiguity in the
terms of the contract, it will be construed against the party who caused such obscurity.

Therefore ambiguous or obscure terms in the insurance policy are to be construed strictly against the
insurer and liberally in favor of the insured party to ensure the protection of the insured since these
insurance contracts are usually arranged and employed by experts and legal advisers acting exclusively
in the interest of the insurance company.

Biagtan vs Insular Life Assurance Co., Ltd., 44 SCRA 58


Facts:
Juan S. Biagtan was insured with defendant Insular Life Assurance Company under Policy No. 398075
for the sum of P5,000.00 and, under a supplementary contract denominated "Accidental Death Benefit
Clause, for an additional sum of P5,000.00 if "the death of the Insured resulted directly from bodily injury
effected solely through external and violent means sustained in an accident and independently of all
other causes." The clause, however, expressly provided that it would not apply where death resulted
from an injury "intentionally inflicted by another party."
On the night of May 20, 1964, or during the first hours of the following day a band of robbers entered the
house of the insured Juan S. Biagtan.
Biagtan was killed as his house was being robbed. The insurance company paid the basic amount of
P5,000 but refused to pay the additional P5,000 under the accidental death benefit clause, on the
ground that his death was the result of injuries intentionally inflicted by third parties and was not covered.
The trial court ruled that there was no proof that the robbers intended to kill Biagtan, or just to scare him
away by thrusting at him with their knives.
Issue: Whether or not, the wounds received by the insured at the hands of the robbers were inflicted
intentionally?
Ruling: YES.
Unlike the ruling in the case of Calanoc vs. Court of Appeals, where the killing of the victim was held as
accidental and thus covered by the insurance policy, the Supreme Court held that in the instant case, the
insured was killed intentionally. The term “intentional” implies the exercise of the reasoning faculties,
consciousness and volition.
The Supreme Court held pointing out that there were nine wounds in all. The exception in the accidental
benefit clause does not speak of the purpose – whether homicidal or not – of a third party in causing the
injuries, but only of the fact that such injuries have been intentionally inflicted. Nine wounds inflicted with
bladed weapons at close range cannot be considered innocent insofar as intent is concerned. The
manner of execution of the crime permits no other conclusion.
Where a provision of the policy excludes intentional injury, it is the intention of the person inflicting the
injury that is controlling. If the injuries suffered by the insured clearly resulted from the intentional act of a
third party the insurer is relieved from liability.
Under the circumstance, the insurance company was correct in refusing to pay the additional sum of
P2,000.00 under the accidental death benefit clause which expressly provided that it would not apply
where death resulted from an injury "intentionally" inflicted by a third party.

FINMAN GENERAL ASSURANCE CORPORATION vs.THE HONORABLE COURTOF APPEALS


213 SCRA 493, September 2, 1992NOCON, J.:
FACTS:
On October 22, 1986, deceased, Carlie Surposa was insured with petitioner FinmanGeneral Assurance
Corporation with his parents, spouses Julia and Carlos Surposa, and brothers Christopher, Charles,
Chester and Clifton, all surnamed, Surposa, as beneficiaries. While said insurance policy was in full
force and effect, the insured, Carlie Surposa, died on October 18,1988 as a result of a stab wound
inflicted by one of the three (3) unidentified men. Private respondent and the other beneficiaries of said
insurance policy filed a written notice of claim with the petitioner insurance company which denied said
claim contending that murder and assault are not within the scope of the coverage of the insurance
policy. Private respondent filed a complaint with the Insurance Commission which rendered a favorable
response for the respondent. The appellate court ruled likewise. Petitioner filed this petition alleging
grave abuse of discretion on the part of the appellate court in applying the principle of "expresso unius
exclusio alterius" in a personal accident insurance policy, since death resulting from murder and/or
assault are impliedly excluded in said insurance policy considering that the cause of death of the insured
was not accidental but rather a deliberate and intentional act of the assailant. Therefore, said death was
committed with deliberate intent which, by the very nature of a personal accident insurance policy,
cannot be indemnified.
ISSUE:
Whether or not the insurer is liable for the payment of the insurance premiums
RULING:
Yes, the insurer is still liable. Contracts of insurance are to be construed liberally in favor of the insured
and strictly against the insurer. Thus ambiguity in the words of an insurance contract should be
interpreted in favor of its beneficiary. The terms "accident" and "accidental" as used in insurance
contracts have not acquired any technical meaning, and are construed by the courts in their ordinary
and common acceptation. Thus, the terms have been taken to mean that which happen by chance or
fortuitously, without intention and design, and which is unexpected, unusual, and unforeseen. Where the
death or injury is not the natural or probable result of the insured's voluntary act, or if something
unforeseen occurs in the doing of the act which produces the injury, the resulting death is within the
protection of the policies insuring against death or injury from accident. In the case at bar, it cannot be
pretended that Carlie Surposa died in the course of an assault or murder as a result of his voluntary act
considering the very nature of these crimes.

ZENITH INSURANCE CORP. vs. COURT OF APPEALS, ET AL. G.R. No. 85296 May 14, 1990. AN
INSURANCE LAW CASE. BY C Y.
FACTS.
1. Private respondent Fernandez insured his car with the ZENITH INSURANCE COMPANY.
2. The car was disfigured in an accidents.
3. Private respondent try to recover the amount of the insurance policy with the petitioner but the latter
they cannot agree on how much the petitioner will pay to the private respondent.
4. Private respondent file a complaint against the petitioner before the trial court of Cebu who order the
petitioner to pay the private respondent the amount of 20000 as moral damages, 10000 as exemplary
and 5000 as an attorney.
5. Petitioner appealed to the CA who affirm the decision of the trial court.
6. Petitioner filed a petition for review to the Supreme Court claiming that the CA acted in excess of its
jurisdiction when it affirmed the decision of the trial court on the ground that while private respondent ask
for moral damages of 1,0000 only, he was awarded with 2,0000, exemplary damages of 5,000 and he
was awarded 1,0000, an attorney’s fee of 3,000 but he was given 5,000.
ISSUE.
WHETHER OR NOT THE CA ACTED IN EXCESS OF ITS JURISDICTION WHEN IT AFFIRM THE
DECISION OF THE TRIAL COURT IN AWARDING THE DAMAGES.
RULING:
According to the Supreme Court, the act of petitioner of delaying payment for two months cannot be
considered as so wanton or malevolent to
justify an award of P20,000.00 as moral damages, taking into consideration the fact that the actual
damage on the car was only P3,460.
The reason for petitioner's failure to indemnify private respondent within
the two-month period was that the parties could not come to an agreement as regards the amount of the
actual damage on the car. The amount of P10,000.00
prayed for by private respondent as moral damages is equitable.

SUN INSURANCE OFFICE, LTD., petitioner, vs. THE HON. COURT OF APPEALS and NERISSA LIM,
respondents.
Facts of the case:
The sun insurance issued Personal Accident Policy to Felix Lim, Jr. with a face value of P200,000.00.
Two months later, the deceased died with a bullet wound in his head. As beneficiary, his wife Nerissa
Lim sought payment on the policy but her claim was rejected. Both parties petitioner agreed that there
was no suicide. However the insurance co. stated that there was no accident either.
Pilar Nalagon, Lim's secretary, was the only eyewitness to his death.
She testified that on October 6, 1982, at about 10 o'clock in the evening, after his mother's birthday party.
According to Nalagon, Lim was in a happy mood (but not drunk) and was playing with his handgun, from
which he had previously removed the magazine. As she watched television, he stood in front of her and
pointed the gun at her. She pushed it aside and said it might he loaded. He assured her it was not and
then pointed it to his temple. The next moment there was an explosion and Lim slumped to the floor. He
was dead before he fell. 1
The nerissa lim sued the insurance co., in the RTC of Zamboanga City and was sustained. 2 The
petitioner was sentenced to pay her P200,000.00, representing the face value of the policy, with interest
at the legal rate; This decision was affirmed on appeal, and the motion for reconsideration was denied. 3
hence the present petition by sun insurance to fault the Court of Appeals for approving the payment of
the claim and the award of damages.
ISSUE: WON there was an accident that occurred which entitles the widow of the deceased to recover
from the insurance policy? And WON the petitioner acted in bad faith for resisting a lawful and just claim.
SC DECISION:
An accident is an event which happens without any human agency or, if happening through human
agency, an event which, under the circumstances, is unusual to and not expected by the person to
whom it happens. It has also been defined as an injury which happens by reason of some violence or
casualty to the injured without his design, consent, or voluntary co-operation. 5
, the Court is convinced that there was truly an accident that which resulted in lim’s death
Lim was unquestionably negligent and that negligence cost him his own life. But it should not prevent his
widow from recovering from the insurance policy he obtained precisely against accident. There is
nothing in the policy that relieves the insurer of the responsibility to pay the indemnity agreed upon if the
insured is shown to have contributed to his own accident. Indeed, most accidents are caused by
negligence. There are only four exceptions expressly made in the contract to relieve the insurer from
liability, and none of these exceptions is applicable in the case at bar. **
1 On the second issue, petitioner is not guilty of bad faith in resisting a legitimate obligation, believing,
on the ground that the death of the insured was covered by the exception the issue, as for the court, was
highly debatable.
Petition is DENIED

JEWEL VILLACORTA vs. THE INSURANCE COMMISSION


G.R. No. L-54171, 28 October 1980 100 SCRA 467
FACTS:
Villacorta had her Colt Lancer car insured with Empire Insurance Company against own damage, theft
and 3rd party liability. While the car was in the repair shop, one of the employees of the said repair shop
took it out for a joyride after which it figured in a vehicular accident. This resulted to the death of the
driver and some of the passengers as well as to extensive
damage to the car. Villacorta filed a claim for total loss with the said insurance company. However, it
denied the claim on the ground that the
accident did not fall within the provisions of the policy either for the Own Damage or Theft coverage,
invoking the policy provision on “Authorized Driver Clause”. This was upheld by the Insurance
Commission further stating
that the car was not stolen and therefore not covered by the Theft Clause because it is not evident that
the person who took the car for a joyride intends to permanently deprive the insured of his/ her car.
ISSUE:
Whether or not the insurer company should pay the said claim, CONSIDERING THE DRIVER IN
QUESTION WAS NOT AUTHORIZED BY THE INSURED OWNER
HELD:
Yes. Where the insured’s car is wrongfully taken without the insured’s consent from the car service and
repair shop to whom it had been entrusted for check-up and repairs (assuming that such taking was for
a joy ride, in the course of which it was totally smashed in an accident), respondent insurer is liable and
must pay insured for the total loss of the insured vehicle under
the Theft Clause of the policy. Assuming, despite the totally inadequate evidence, that the taking was
“temporary” and for a “joy ride”, the Court sustains as the better view that which holds that when a
person, either
with the object of going to a certain place, or learning how to drive, or enjoying a free ride, takes
possession of a vehicle belonging to another, without the consent of its owner, he is guilty of theft
because by taking possession of the personal property belonging to another and using it, his intent to
gain is evident since he derives there from utility, satisfaction, enjoyment and pleasure. ACCORDINGLY,
the appealed decision is set aside and judgment is hereby rendered sentencing private respondent to
pay petitioner the sum of P35,000.00 with legal interest from the filing of the complaint until full payment
is made and to pay the costs of suit.

Palermo v. Pyramid Insurance


FACTS: On October 12,1968, after having purchased a brand new Nissan Cedric de Luxe Sedan car
bearing Motor No. 087797 from the Ng Sam Bok Motors Co. in Bacolod City, plaintiff insured the same
with the defendant insurance company against any loss or damage for P 20,000.00 and against third
party liability for P 10,000.00. The automobile was, however, mortgaged by the plaintiff with the vendor,
Ng Sam Bok Motors Co., to secure the payment of the balance of the purchase price, which explains
why the registration certificate in the name of the plaintiff remains in the hands of the mortgagee, Ng
Sam Bok Motors Co. On April 17, 1968, while driving the automobile in question, the plaintiff met a
violent accident. The La Carlota City fire engine crashed head on, and as a consequence, the plaintiff
sustained physical injuries, his father, Cesar Palermo, who was with am in the car at the time was
likewise seriously injured and died shortly thereafter, and the car in question was totally wrecked.
Palermo, filed a complaint in the Court of First Instance of Negros Occidental against Pyramid Insurance
Co., Inc., for payment of his claim. Pyramid Insurance Co., Inc., disallowed the claim because at the
time of the accident, the insured was driving his car with an expired driver's license.
ISSUE:
WON Palermo is entitled to the claim
HELD:
YES. AUTHORIZED DRIVER:
Any of the following:
(a) The Insured.
(b) Any person driving on the Insured's order or with his permission. Provided that the person driving is
permitted in accordance with the licensing or other laws or regulations to drive the Motor Vehicle and is
not disqualified from driving such motor vehicle by order of a Court of law or by reason of any enactment
or regulation in that behalf. (Exh. "A.")
There is no merit in the appellant's allegation that the plaintiff was not authorized to drive the insured
motor vehicle because his driver's license had expired. The driver of the insured motor vehicle at the
time of the accident was, the insured himself, hence an "authorized driver" under the policy.
While the Motor Vehicle Law prohibits a person from operating a motor vehicle on the highway without a
license or with an expired license, an infraction of the Motor Vehicle Law on the part of the insured, is
not a bar to recovery under the insurance contract. It however renders him subject to the penal
sanctions of the Motor Vehicle Law.
The requirement that the driver be "permitted in accordance with the licensing or other laws or
regulations to drive the Motor Vehicle and is not disqualified from driving such motor vehicle by order of
a Court of Law or by reason of any enactment or regulation in that behalf," applies only when the driver"
is driving on the insured's order or with his permission." It does not apply when the person driving is the
insured himself.

Figuracion vda. De Maglana v. Consolacion


FACTS:
Lope Maglana was an employee of the Bureau of Customs whose work station was at Lasa, here in
Davao City. One morning, while on his way to his work station, driving a motorcycle owned by the
Bureau of Customs. At Km. 7, Lanang, he met an accident that resulted in his death. The jeep that
bumped the deceased was owned by Destrajo. Destrajo, had an insurance policy issued by AFISCO
Insurance. The trial court ordered that AFISCO should reimburse Destrajo for the amount paid to the
plaintiff as a result of the accident but only to the extent of the insurance coverage. Petitioners contend
that AFISCO’s liability should be direct and primary, and not merely secondary as provided under the
insurance code. Hence, they argued that the P20,000.00 coverage of the insurance policy issued by
AFISCO, should have been awarded in their favor.
ISSUE:
WON AFISCO’s liability is dependent upon the recovery of judgment by the injured party against the
insured.
HELD:
NO. The particular provision of the insurance policy on which petitioners base their claim is as follows:
Sec. 1 — LIABILITY TO THE PUBLIC
1. The Company will, subject to the Limits of Liability, pay all sums necessary to discharge liability of the
insured in respect of
(a) death of or bodily injury to any THIRD PARTY
xXX
3. In the event of the death of any person entitled to indemnity under this Policy, the Company will, in
respect of the liability incurred to such person indemnify his personal representatives in terms of, and
subject to the terms and conditions hereof.
The above-quoted provision leads to no other conclusion but that AFISCO can be held directly liable by
petitioners. As this Court ruled in Shafer vs. Judge, RTC of Olongapo City, Br. 75, "[w]here an insurance
policy insures directly against liability, the insurer's liability accrues immediately upon the occurrence of
the injury or even upon which the liability depends, and does not depend on the recovery of judgment by
the injured party against the insured." 8 The underlying reason behind the third party liability (TPL) of the
Compulsory Motor Vehicle Liability Insurance is "to protect injured persons against the insolvency of the
insured who causes such injury, and to give such injured person a certain beneficial interest in the
proceeds of the policy . . ." 9 Since petitioners had received from AFISCO the sum of P5,000.00 under
the no-fault clause, AFISCO's liability is now limited to P15,000.00.
However, we cannot agree that AFISCO is likewise solidarily liable with Destrajo. where the insurance
contract provides for indemnity against liability to third persons, such third persons can directly sue the
insurer, however, the direct liability of the insurer under indemnity contracts against third party liability
does not mean that the insurer can be held solidarily liable with the insured and/or the other parties
found at fault. The liability of the insurer is based on contract; that of the insured is based on tort While in
solidary obligations, the creditor may enforce the entire obligation against one of the solidary debtors, in
an insurance contract, the insurer undertakes for a consideration to indemnify the insured against loss,
damage or liability arising from an unknown or contingent event.

PCSI vs. CA, 208 SCRA, 487


FACTS:
Spouses Herminio and Evely Lim executed a promissory note in favor of Supercars secured by a chattel
mortgage over a brand new Ford Laser registered under the name of Herminio and insured with PCSI.
Supercars with notice to the spouses assigned to FCP Credit Corp its rights, title and interest on the pro
missory note and chattel mortgage.
Subsequently, the vehicle was carnapped. Evelyn, was the one driving before it was stolen.
The spouses filed a claim for loss with PCSI but was denied on the ground that Evelyn’s driver’s license
was expired at the time of the loss in violation of the authorized driver clause.

ISSUE:
WON PCSI is liable

HELD:
YES. Clearly, the risk against accident is distinct from the risk against theft. The “authorized driver claus
e” in an insurance policy is in contemplation or anticipation of accident in the legal sense in which it shou
ld be understood, and not in in contemplation or anticipation of an event such as theft. Thus, if the insure
d vehicle had figured in an accident at the time she drove it with an expired license, then PCSI could pro
perly resist the claim for indemnification resulting from the accident. But in the present case, the loss of t
he vehicle did not result from an accident where intent was involved; the loss in the present case was ca
used by theft, the commission of which was attended by intent.
It is worthy to note that there is no causal connection between the possession of a valid driver’s license
and the loss of the vehicle. To rule otherwise would render car insurance practically a sham since an ins
urance company can easily escape liability by citing restrictions which are not applicable or germane to t
he claim, thereby reducing indemnity to a shadow.

GEAGONIA vs. CA, COUNTRY BANKERS INSURANCE CORP., G.R. 114427, 2/6/95
FACTS:
Armando Geagonia is the owner of Norman’s Mart and obtained from Country Bankers a fire insurance
policy which covered Stock-in-trade consisting of RTW dry goods.
The policy contained a provision where the insured must give notice to the insurer of any insurance or in
surances already affected or which may be subsequently be effected covering any of the property or pro
perties consisting of stocks in trade, goods in process and/or inventories already insured by such policy
otherwise it shall be deemed forfeited, provided that such condition does not apply when the total insura
nce or insurances in force at the time of the loss is not more than 200k
Subsequently, a fire broke out and destroyed Geagonia’s stocks-in-trade. Country bankers denied the cl
aim because it was found that at the time of the loss, the stocks were likewise covered by two other fire i
nsurances for 100k each by PFIC. It had a mortgage clause which stated that loss, if any, shall be payab
le to Cebu Tesing Textiles.

ISSUE:
WON there was double insurance to justify denial of the claim

HELD:
NO (Country Bankers is liable).
It is a cardinal rule on insurance that a policy or insurance contract is to be interpreted liberally in favor o
f the insured and strictly against the company, the reason being, undoubtedly, to afford the greatest prot
ection which the insured was endeavoring to secure when he applied for insurance. Provisions, conditio
ns, or exceptions in policies which tend to work a forfeiture of insurance policies should be construed mo
st strictly against those for whose benefits they are inserted, and most favorably toward those against w
hom they are intended to operate.
The condition in the policy is commonly known as the additional or “other insurance” clause and has bee
n upheld as valid and as a warranty that no other insurance exists. Its violation would thus avoid the poli
cy. However, in order to constitute a violation, the other insurance must be upon the same subject matte
r, the same insurable interest, and the same risk.
As to a mortgaged property, the mortgagor and the mortgagee have each an independent insurable inter
est therein and both interests may be one policy, or each may take out a separate policy covering his int
erest, either at the same or separate times. The mortgagor’s insurable interest covers the full value of th
e mortgaged property, even though the mortgage debt is equivalent to the full value of the property. The
mortgagee’s insurable interest is to the extent of the debt, since the property is relied upon as security th
ereof, and in insuring he is not insuring the property but his interest or lien thereon.
A double insurance exists where the same person is insured by several insurers separately in respect of
the same subject and cover the same interest. Since the two policies of the PFIC do not cover the same
interest as that covered by the policy in issue, no double insurance exists. The non-disclosure is not fat
al.

11. Fortune Insurance and Surety Co., Inc. v. Court of Appeals


Facts:
On June 29, 1987, Producer’s Bank of the Philippines’ armored vehicle was robbed, in transit, of seven
hundred twenty-five thousand pesos (Php 725,000.00) that it was transferring from its branch in Pasay
to its main branch in Makati. To mitigate their loss, they claim the amount from their insurer, namely
Fortune Insurance and Surety Co.
Fortune Insurance, however, assails that the general exemption clause in the Casualty Insurance
coverage had a general exemption clause, to wit:
GENERAL EXCEPTIONS
The company shall not be liable under this policy in respect of
xxx xxx xxx
(b) any loss caused by any dishonest, fraudulent or criminal act of the insured or any officer, employee,
partner, director, trustee or authorized representative of the Insured whether acting alone or in
conjunction with others. . . .
And, since the driver (Magalong) and security guard (Atiga) of the armored vehicle were charged with
three others as liable for the robbery, Fortune denies Producer’s Bank of its insurance claim.
The trial court and the court appeals ruled in favor of recovery, hence, the case at bar.

Issue:
Whether recovery is precluded under the general exemption clause.

Ruling:
Yes, recovery is precluded under the general exemption clause.
Howsoever viewed, Producers entrusted the three with the specific duty to safely transfer the money to
its head office, with Alampay to be responsible for its custody in transit; Magalong to drive thearmored
vehicle which would carry the money; and Atiga to provide the needed security for the money, the
vehicle, and his two other companions. In short, for these particular tasks, the three acted as agents of
Producers. A "representative" is defined as one who represents or stands in the place of another; one
who represents others or another in a special capacity, as an agent, and is interchangeable with "agent."
In view of the foregoing, Fortune is exempt from liability under the general exceptions clause of the
insurance policy.

12. Edillon v. Manila Bankers Life

Facts:
In April 1969, Carmen Lapuz filled out an application form for insurance under Manila Banker Life
Assurance Corporation. She stated that her date of birth was July 11, 1904. Upon payment of the Php
20.00 premium, she was issued the insurance policy in April 1969. In May 1969, Carmen Lapuz died in a
vehicular accident. Regina Edillon, who was named a beneficiary in the insurance policy sought to
collect the insurance claim but Manila Banker denied the claim. Apparently, it is a rule of the insurance
company that they were not to issue insurance policies to “persons who are under the age of sixteen (16)
years of age or over the age of sixty (60) years …” Note, that Lapuz was already 65 years old when she
was applying for the insurance policy.

Issue:
Whether or not Edillon is entitled to the insurance claim as a beneficiary.

Ruling:
Yes. Carmen Lapuz did not conceal her true age. Despite this, the insurance company still received
premium from Lapuz and issued the corresponding insurance policy to her. When the accident
happened, the insurance policy has been in force for 45 days already and such time was already
sufficient for Manila Banker to notice the fact that Lapuz is already over 60 years old and thereby cancel
the insurance policy. If Manila Banker failed to act, it is either because it was willing to waive such
disqualification; or, through the negligence or incompetence of its employees for which it has only itself
to blame, it simply overlooked such fact. Under the circumstances, Manila Banker is already deemed in
estoppel.

13. PERLA COMPANIA DE SEGUROS, INC vs. CA and CAYAS


FACTS:
Cayas was the registered owner of a Mazda bus which was insured with petitioner PERLA COMPANIA
DE SEGUROS, INC (PCSI). The bus figured in an accident in Cavite, injuring several of its passengers.
One of them, Perea, sued Cayas for damages in the CFI, while three others agreed to a settlement of
P4,000.00 each with Cayas.
After trial, the court rendered a decision in favor of Perea, Cayas ordered to compensate the latter with
damages. Cayas filed a complaint with the CFI, seeking reimbursement from PCSI for the amounts she
paid to ALL victims, alleging that the latter refused to make such reimbursement notwithstanding the fact
that her claim was within its contractual liability under the insurance policy.
The decision of the CA affirmed in toto the decision of the RTC of Cavite, the dispositive portion of which
states:
IN VIEW OF THE FOREGOING, judgment is hereby rendered ordering defendant PCSI to pay plaintiff
Cayas the sum of P50,000.00 under its maximum liability as provided for in the insurance policy; …
In this petition for review on certiorari, petitioner seeks to limit its liability only to the payment made by
private respondent to Perea and only up to the amount of P12,000.00. It altogether denies liability for the
payments made by private respondents to the other 3 injured passengers totaling P12,000.00.

ISSUE:
How much should PCSI pay?

HELD:
The decision of the CA is modified, petitioner only to pay Cayas P12,000,000.00
The insurance policy provides:
5. No admission, offer, promise or payment shall be made by or on behalf of the insured without the
written consent of the Company …
It being specifically required that petitioner’s written consent be first secured before any payment in
settlement of any claim could be made, private respondent is precluded from seeking reimbursement of
the payments made to the other 3 victims in view of her failure to comply with the condition contained in
the insurance policy.
Also, the insurance policy involved explicitly limits petitioner’s liability to P12,000.00 per person and to
P50,000.00 per accident
Clearly, the fundamental principle that contracts are respected as the law between the contracting
parties finds application in the present case. Thus, it was error on the part of the trial and appellate
courts to have disregarded the stipulations of the parties and to have substituted their own interpretation
of the insurance policy.
We observe that although Cayas was able to prove a total loss of only P44,000.00, petitioner was made
liable for the amount of P50,000.00, the maximum liability per accident stipulated in the policy. This is
patent error. An insurance indemnity, being merely an assistance or restitution insofar as can be fairly
ascertained, cannot be availed of by any accident victim or claimant as an instrument of enrichment by
reason of an accident.

14. Aisporna v CA 113 SCRA 459 (1982)


Facts
Mapalad Aisporna, the wife of one Rodolfo Aisporna, an insurance agent, solicited the application of
Eugenio Isidro in behalf of Perla Compana de Seguros without the certificate of authority to act from the
insurance commissioner. Isidro passed away while his wife was issued Php 5000 fromthe insurance
policy. After the death, the fiscal instigated criminal action against Mapalad for violating sec 189 of the
Insurance code for feloniously acting as agent when she solicited theapplication form.
In the trial court, she claimed that she helped Rodolfo as clerk and that she solicited a renewal, not a
new policy from Isidro through the phone. She did this because her husband was absent when he called.
She only left a note on top of her husband’s desk to inform him of what transpired. (She did not accept
compensation from Isidro for her services)
Aisporna was sentenced to pay Php 500 with subsidiary costs in case of insolvency in 1971 in the
Cabanatuan city court.
In the appellate court, she was found guilty of having violating par 1 of sec 189 of the insurance code.
The OSG kept on repeating that she didn’t violate sec 189 of the insurance code.
In seeking reversal of the judgment, Aisporna assigned errors of the appellate court:
1. the receipt of compensation was not a necessary element of the crime in par 1 of sec 189 of the
insurance code
2. CA erred in giving due weight to exhibits F, F1, F17 inclusive sufficient to establish petitioner’s guilt
beyond reasonable doubt.
3. The CA erred in not acquitting the petitioner

Issues:
Won a person can be convicted of having violated the 1st par of the sec 189 of the IC without reference
to the 2nd paragraph of the said section. Or
Is it necessary to determine WON the agent mentioned in the 1st paragraph of the aforesaid section is
governed by the definition of an insurance agent found on its second paragraph

Decision:
Aisporna acquitted

Ruling:
Sect 189 of the I.C., par 1 states that “No insurance company doing business with the Philippine Islands
nor l any agent thereof shall pay any commission or other compensation to any person for services in
obtaining new insurance unless such person shall have first procured from the Insurance Commissioner
a certificate of authority to act as an agent of such company as herein after provided.
No person shall act as agent, sub-agent, or broker in the solicitation of procurement of applications for
insurance without obtaining a certificate from the Insurance Commissioner.
Par2 Any person who for COMPENSATION solicits or obtains insurance for any for any insurance
compna or offers or assumes to act in the negotiating of such insurance shall be an insurance agent in
the intent of this section and shall thereby become liable to all liabilities to which an insurance agent is
subject.
Par 3 500 pseo fine for person or company violating the provisions of the section.
The court held that the 1st par prohibited a person to act as agent without certificate of authorityfrom the
commissioner
In the 2nd par, the definition of an insurance agent is stipulated
The third paragraph provided the penalty for violating the 1st 2 rules
The appellate court said that the petitioner was penalized under the1st paragraph and not the 1nd. The
fact that she didn’t receive compensation wasn’t an excuse for her acquittal because she was actually
punished separately under sec 1 because she did not have a certificate of authority as under par 1.
The SC held that the definition of an insurance agent was made by CA to be limited to paragraph 2 and
not applicable to the 1st paragraph.
The appellate court said that a person was an insurance agent under par 2 if she solicits insurance for
compensation, but in the 1st paragraph, there was no necessity that a person solicits an insurance
compensation in order to be called an agent.
The SC said that this was a reversible error.
The CA said that Aisporna didn’t receive compensation.
The SC said that the definition of an insurance agent was found in the 2nd par of Sec 189 (check the
law) The definition in the 2nd paragraph qualified the definition of an agent used in the 1st and third
paragraphs.

DOCTRINE: The court held that legislative intent must be ascertained from the consideration of the
statute as a whole. The words shouldn’t be studied in isolated explanations but the whole and every part
of the statute must be considered in fixing the meaning of any of its parts in order to pronounce the
harmonious whole.
Noscitur a sociis provides that where a particular word or phrase in a statement is ambiguous in itself,
the true meaning may be made clear in the company it is fixed in. In applying this, the court held that the
definition of an insurance agent in the 2nd paragraph was applicable in the 1stparagraph.
To receive compensation be the agent is an essential element for violation of the 1st paragraph.
The appellate court said that she didn’t receive compensation by the receipt of compensation wasn’t an
essential element for violation of the 1st paragraph.
The SC said that this view wasn’t correct owing to the American insurance laws which qualified
compensation as a qualifying factor in penalizing unauthorized persons who solicited insurance (Texas
code and snyder’s law)

15.COUNTRY BANKERS INSURANCE CORPORATION, vs. LIANGA BAY AND COMMUNITY MULTI-
PURPOSE COOPERATIVEG.R. No. 136914 January 25, 2002DE LEON JR J:

Facts:

The petitioner is a domestic corporation principally engaged in the insurance business wherein it underta
kes, for aconsideration, to indemnify another against loss, damage or liability from an unknown or contingent event inc
luding fire whilethe respondent is a duly registered cooperative judicially declared insolvent and represente
d by the elected assignee,Cornelio Jamero.Sometime in1989, the petitioner and the respondent entered in
to a contract of fire insurance, Fire Insurance Policy No. F-1397. Under Fire Insurance, the petitioner ins
ured the respondent’s stocks-in-trade against fire loss, damage or liabilityduring the period starting from June
20, 1989 to June 20, 1990 for the sum of Two Hundred Thousand Pesos.On July 1, 1989, the respondent’s building lo
cated at Surigao del Sur was gutted by fire and reduced to ashes, resulting inthe total loss of the respondent’s stocks-in
-trade, pieces of furnitures and fixtures, equipments and records. Due to the loss,the respondent filed an insurance clai
m with the petitioner under its Fire Insurance.The petitioner, however, denied the insurance claim on the grou
nd that, based on the submitted documents, the buildingwas set on fire by two NPA rebels who wanted to obtai
n canned goods, rice and medicines as provisions for their comradesin the forest, and that such loss was an exc
epted risk under the policy conditions of Fire Insurance Policy which provides:This insurance does not cover
any loss or damage occasioned by or through or in consequence, directly or indirectly, of anyof the following occurrenc
es, namely:(d) Mutiny, riot, military or popular uprising, insurrection, rebellion, revolution, military or usurped power.Res
pondent then instituted in the trial court the complaint for recovery of "loss, damage or liability" against p
etitioner. Thepetitioner answered the complaint and reiterated the ground it earlier cited to deny the insurance claim.T
he trial court rendered its Decision in favor of the respondent declaring that the defendant-Country Bank
ers was liable toplaintiff-Insolvent Cooperative and to fully pay the insurance claim for the loss the insure
d-plaintiff sustained as a result of the fire under its Fire Insurance in its full face value of P

200,000.00 with interest of 12% per annum from date of filing of thecomplaint until the same is fully paid.Petitioner appe
aled to the Court of Appeals which affirmed the decision of the trial court in its entirety. Hence, this petition.

Issue:
Whether Country Bankers in liable

Ruling:

Yes Country bankers is liable.


The petitioner does not dispute that the respondent’s stocks-in-trade were insured against fire loss, damage or liability u
nder Fire Insurance Policy and that the respondent lost its stocks-in-trade in a fire that occurred within th
e duration of said fireinsurance. The petitioner, however, posits the view that the cause of the loss was a
n excepted risk under the terms of thefire insurance policy. Where a risk is excepted by the terms of a
policy which insures against other perils or hazards, loss from such a risk constitutes a defense w
hich the insurer may urge, since it has not assumed that risk, and from this it follows that an insurer seeking to defeat
a claim because of an exception or limitation in the policy has the burden of proving that the loss comes
within the purview of the exception or limitation set up. If a proof is made of a loss apparently within a contract of insura
nce, the burden is upon the insurer to prove that the loss arose from a cause of loss which is excepted or for which it is
not liable, or from a cause which limits its liability. Stated else wise, since the petitioner in this case is defending on the g
round of non-coverage and relying upon an exemption or exception clause in the fire insurance policy, it h
as the burden of proving the facts upon which such excepted risk is based, by a preponderance of evidence. But p
etitioner failed to do so. The petitioner relies on the Sworn Statements of Jose Lomocso and Ernesto Urbizt
ondo and on the Spot Report of Pfc. Arturo V. Juarbal specifically that: “investigation revealed by Jose Lomo
cso that those armed men wanted to get can goodsand rice for their consumption in the forest PD investigation further
disclosed that the perpetrator are members of the NPA” .Such testimony is considered hearsay and may not be
received as proof of the truth of what he has learned.

16. AMERICAN HOME ASSURANCE COMPANY vs. TANTUCO ENTERPRISES, INC.

FACTS:
Respondent Tantuco Enterprises, Inc. is engaged in the coconut oil milling and refining industry.It owns t
wo oil mills which were separately covered by fire insurance policies issued by petitionerAmerican Home
Assurance Co., Philippine Branch.

The first oil mill was insured for P3,000,000.00 under Policy No. 306-7432324-3 for the period March 1
, 1991 to 1992. The new oil mill was insured forP6,000,000.00 under Policy No. 306-7432321-9 for the s
ame term. Official receipts indicating payment for the full amount of the premium were issued by the peti
tioner's agent .A fire that broke out in the early morning of September 30,1991 gutted and consumed the
new oil mill. Respondent immediately notified the petitioner of the incident but petitioner rejected respon
dent's claim for the insurance proceeds on the ground that no policy was issued by it covering the burne
d oil mill. It stated that the description of the insured establishment referred to another building thus: "Ou
r policy nos. 306-7432321-9 (Ps 6M) and 306-7432324-4 (Ps 3M) extend insurance coverage to your oil
mill under Building No. 5, whilst the affected oil mill was under Building No. 14. "

ISSUE:
Whether or not respondent can claim from the petitioner insurance company.

HELD:
In construing the words used descriptive of a building insured, the greatest liberality is shown by the cou
rts in giving effect to the insurance. In view of the custom of insurance agents to examine buildings befor
e writing policies upon them, and since a mistake as to the identity and character of the building is extre
mely unlikely, the courts are inclined to consider that the policy of insurance covers any building which th
e parties manifestly intended to insure, however inaccurate the description may be. Notwithstanding, the
refore, the misdescription in the policy, it is beyond dispute, to our mind, that what the parties manifestly
intended to insure was the new oil mill. If the parties really intended to protect the first oil mill, then there
is no need to specify it as new .In determining what the parties intended, the courts will read and constru
e the policy as a whole and if possible, give effect to all the parts of the contract, keeping in mind always
, however, the prime rule that in the event of doubt, this doubt is to be resolved against the insurer. In de
termining the intent of the parties to the contract, the courts will consider the purpose and object of the c
ontract.

17. White Gold Marine vs Pioneer Insurance


Facts
Petitioner procured a protection and indemnity coverage for its vessels from The Steamship Mutual
Underwriting Association Ltd. through Pioneer Insurance and Surety Corp. by virtue of a Certificate of
Entry and Acceptance. When White Gold failed topay its account, Steamship Mutual refused to renew its
coverage so it filed a collection case against the latter. Petitioner contends in defense that it didn’t have
the requisite certificate of authority from the Insurance Commissioner under Sec. 187 of the Insurance
Code.

Issue
WON Pioneer still needs a license as an insurance agent/broker for Steamship Mutual;
WON Steamship Mutual’s Protection and Indemnity club is engaged in insurance business in the
Philippines.

Ruling
YES on both issues.
1) A Protection and Indemnity Club is a form of insurance against third party liability where the third party
is anyone other than the P & I Club and the members. Steamship Mutual, as P&I club, is a mutual
insurance association engaged in marine insurance business.

2) Although Pioneer is already a licensed insurance company, it still needs a separate license to act as
an insurance agent for Steamship Mutual as provided by Section 299 of the Insurance Code

18. Republic (CIR) vs Sun Life Assurance of Canada

Facts
On December 29, 1997, the [Court of Tax Appeals] (CTA) rendered its decision in Insular Life Assurance
Co. Ltd. v. [CIR], which held that mutual life insurance companies are purely cooperative companies and
are exempt from the payment of premium tax and DST. This pronouncement was later affirmed by this
court in [CIR] v. Insular Life Assurance Company, Ltd. Sun Life surmised that[,] being a mutual life
insurance company, it was likewise exempt from the payment of premium tax and DST. Hence, on
August 20, 1999, Sun Life filed with the CIR an administrative claim for tax credit of its alleged
erroneously paid premium tax and DST for the aforestated tax periods.
For failure of the CIR to act upon the administrative claim for tax credit and with the 2-year period to file
a claim for tax credit or refund dwindling away and about to expire, Sun Life filed with the CTA a petition
for review. The CTA found in favor of Sun Life.
Seeking reconsideration of the decision of the CTA, the CIR argued that Sun Life ought to have
registered, foremost, with the Cooperative Development Authority before it could enjoy the exemptions
from premium tax and DST extended to purely cooperative companies or associations under [S]ections
121 and 199 of the Tax Code. For its failure to register, it could not avail of the exemptions prayed for.
The CTA denied the CIR’s motion for reconsideration.

Issue:
WON respondent is exempted from payment of tax on life insurance premiums and documentary stamp
tax

Ruling:
YES. The Tax Code defines a cooperative as an association “conducted by the members thereof with
the money collected from among themselves and solely for their own protection and not for profit.”
Without a doubt, respondent is a cooperative engaged in a mutual life insurance business.
First, it is managed by its members. Both the CA and the CTA found that the management and affairs of
respondent were conducted by its member-policyholders. SUNLIFE has been mutualized or converted
from a stock life insurance company to a nonstock mutual life insurance corporation pursuant to Section
266 of the Insurance Code of 1978. On the basis of its bylaws, its ownership has been vested in its
member-policyholders who are each entitled to one vote; and who, in turn, elect from among themselves
the members of its board of trustees.
Second, it is operated with money collected from its members. Since respondent is composed entirely of
members who are also its policyholders, all premiums collected obviously come only from them. The
member-policyholders constitute “both insurer and insured” who “contribute, by a system of premiums or
assessments, to the creation of a fund from which all losses and liabilities are paid.”
Third, it is licensed for the mutual protection of its members, not for the profit of anyone. A mutual life
insurance company is conducted for the benefit of its member-policyholders, who pay into its capital by
way of premiums.
Under the Tax Code although respondent is a cooperative, registration with the Cooperative
Development Authority (CDA) is not necessary in order for it to be exempt from the payment of both
percentage taxes on insurance premiums, under Section 121; and documentary stamp taxes on policies
of insurance or annuities it grants, under Section 199.

19. PHILAMCARE HEALTH SYSTEMS, INC. vs. COURT OF APPEALS and JULITA TRINOS
G.R. No. 125678 | March 18, 2002

Facts:
Ernani Trinos, deceased husband of respondent Julita Trinos, applied for a health care coverage with
petitioner Philamcare Health Systems, Inc.
The application was approved for a period of one year from March 1, 1988 to March 1, 1989 and
extended until June 1, 1990. The amount of coverage was increased to a maximum sum of P75,000.00
per disability.
During the period of his coverage, Ernani suffered a heart attack and was confined for one month. While
her husband was in the hospital, respondent tried to claim the benefits under the health care agreement.
However, petitioner denied her claim saying that the Health Care Agreement was void. Thus, respondent
paid the hospitalization expenses herself, amounting to about P76,000.00.
After her husband was discharged from the MMC, he was later admitted at the Chinese General
Hospital. In the morning of April 13, 1990, Ernani had fever and was feeling very weak; he died on the
same day.
Respondent instituted with the Regional Trial Court (RTC) an action for damages against petitioner. The
RTC ruled against petitioner, which was affirmed by the Court of Appeals.
Petitioner filed a petition for review arguing that the agreement grants "living benefits," such as medical
check-ups and hospitalization which a member may immediately enjoy so long as he is alive upon
effectivity of the agreement until its expiration one-year thereafter. Petitioner also points out that only
medical and hospitalization benefits are given under the agreement without any indemnification, unlike
in an insurance contract where the insured is indemnified for his loss. Moreover, since Health Care
Agreements are only for a period of one year, as compared to insurance contracts which last longer,7
petitioner argues that the incontestability clause does not apply, as the same requires an effectivity
period of at least two years. Petitioner further argues that it is not an insurance company, which is
governed by the Insurance Commission, but a Health Maintenance Organization under the authority of
the Department of Health.
Issue:
Whether or not a health care agreement is not an insurance contract.

Ruling:
A health care agreement is an insurance contract.
Under Section 2 of the Insurance Code defines a contract of insurance as an agreement whereby one
undertakes for a consideration to indemnify another against loss, damage or liability arising from an
unknown or contingent event. An insurance contract exists where the following elements concur: (1) The
insured has an insurable interest; (2) The insured is subject to a risk of loss by the happening of the
designated peril; (3) The insurer assumes the risk; (4) Such assumption of risk is part of a general
scheme to distribute actual losses among a large group of persons bearing a similar risk; and (5) In
consideration of the insurer’s promise, the insured pays a premium.
Further, Section 10 of the Insurance Code provides that every person has an insurable interest in the life
and health of himself.
In the case at bar, the insurable interest of respondent’s husband in obtaining the health care agreement
was his own health. The health care agreement was in the nature of non-life insurance, which is
primarily a contract of indemnity.9 Once the member incurs hospital, medical or any other expense
arising from sickness, injury or other stipulated contingent, the health care provider must pay for the
same to the extent agreed upon under the contract.

20. COMMISSIONER OF INTERNAL REVENUE vs. LINCOLN PHILIPPINE LIFE INSURANCE


COMPANY, INC. (now JARDINE-CMA LIFE INSURANCE COMPANY, INC.) and THE COURT OF
APPEALS
G.R. No. 119176 | March 19, 2002

Facts:
In the years prior to 1984, private respondent issued a special kind of life insurance policy known as the
"Junior Estate Builder Policy," the distinguishing feature of which is a clause providing for an automatic
increase in the amount of life insurance coverage upon attainment of a certain age by the insured
without the need of issuing a new policy. The clause was to take effect in the year 1984. Documentary
stamp taxes due on the policy were paid by petitioner only on the initial sum assured.
Subsequently, petitioner issued deficiency documentary stamps tax assessment for the year 1984 in the
amount of P464,898.75 corresponding to the amount of automatic increase of the sum assured on the
policy issued by respondent.
Private respondent questioned the deficiency assessments and sought their cancellation in a petition
filed in the Court of Tax Appeals.
The Court of Tax Appeals found no valid basis for the deficiency tax assessment on the insurance policy.
The Court of Appeals affirmed the decision of the Court of Tax Appeals decision insofar as it nullified the
deficiency assessment on the insurance policy.
The Commissioner of Internal Revenue filed the present petition questioning that portion of the Court of
Appeals’ decision which invalidated the deficiency assessment on the insurance policy.
Petitioner claims that the "automatic increase clause" in the subject insurance policy is separate and
distinct from the main agreement and involves another transaction; and that, while no new policy was
issued, the original policy was essentially re-issued when the additional obligation was assumed upon
the effectivity of this "automatic increase clause" in 1984; hence, a deficiency assessment based on the
additional insurance not covered in the main policy is in order.

Issues:
1. Whether or not the automatic increase clause is a single agreement embodied in the policy or a
separate agreement.
2. Whether or not the Court of Appeals erred in not computing the amount of tax on the total value of the
insurance assured in the policy including the additional increase assured by the automatic increase
clause.

Ruling:
The petition is impressed with merit.
It is clear from Section 173 that the payment of documentary stamp taxes is done at the time the act is
done or transaction had and the tax base for the computation of documentary stamp taxes on life
insurance policies under Section 183 is the amount fixed in policy, unless the interest of a person
insured is susceptible of exact pecuniary measurement. The amount fixed in the policy is the figure
written on its face and whatever increases will take effect in the future by reason of the "automatic
increase clause" embodied in the policy without the need of another contract.
Here, although the automatic increase in the amount of life insurance coverage was to take effect later
on, the date of its effectivity, as well as the amount of the increase, was already definite at the time of the
issuance of the policy. Thus, the amount insured by the policy at the time of its issuance necessarily
included the additional sum covered by the automatic increase clause because it was already
determinable at the time the transaction was entered into and formed part of the policy.
The deficiency of documentary stamp tax imposed on private respondent is definitely not on the amount
of the original insurance coverage, but on the increase of the amount insured upon the effectivity of the
"Junior Estate Builder Policy."
To claim that the increase in the amount insured (by virtue of the automatic increase clause incorporated
into the policy at the time of issuance) should not be included in the computation of the documentary
stamp taxes due on the policy would be a clear evasion of the law requiring that the tax be computed on
the basis of the amount insured by the policy.

PERFECTION OF INSURANCE CONTRACT

21. Enriquez v Sun Life of Canada 41 Phil 269

FACTS:
September 24, 1917: Joaquin Herrer made application to the Sun Life Assurance Company of Canada
through its office in Manila for a life annuity
2 days later: he paid P6,000 to the manager of the company's Manila office and was given a receipt
According to the provisional receipt, 3 things had to be accomplished by the insurance company before
there was a contract:
(1) There had to be a medical examination of the applicant; -check
(2) there had to be approval of the application by the head office of the company; and - check
(3) this approval had in some way to be communicated by the company to the applicant
November 26, 1917: The head office at Montreal, Canada gave notice of acceptance by cable to Manila
but this was not mailed
December 4, 1917: policy was issued at Montreal
December 18, 1917: attorney Aurelio A. Torres wrote to the Manila office of the company stating that
Herrer desired to withdraw his application
December 19, 1917: local office replied to Mr. Torres, stating that the policy had been issued, and called
attention to the notification of November 26, 1917
December 21, 1917 morning: received by Mr. Torres
December 20, 1917: Mr. Herrer died
Rafael Enriquez, as administrator of the estate of the late Joaquin Ma. Herrer filed to recover from Sun
Life Assurance Company of Canada through its office in Manila for a life annuity
RTC: favored Sun Life Insurance

ISSUE:
WON Mr. Herrera received notice of acceptance of his application thereby perfecting his life annuity

RULING:
NO. Not perfected because it has not been proved satisfactorily that the acceptance of the application
ever came to the knowledge of the applicant.
Art. 1319. Consent is manifested by the meeting of the offer and the acceptance upon the thing and the
cause which are to constitute the contract. The offer must be certain and the acceptance absolute. A
qualified acceptance constitutes a counter-offer.
Acceptance made by letter or telegram does not bind the offerer except from the time it came to his
knowledge. The contract, in such a case, is presumed to have been entered into in the place where the
offer was made.
Judgment is reversed, and the Enriquez shall have and recover from the Sun Life the sum of P6,000
with legal interest from November 20, 1918, until paid, without special finding as to costs in either
instance. So ordered.
22. Great Pacific v CA 89 SCRA 453

FACTS:
Respondent Ngo Hing filed an application with petitioner Great Pacific Life Assurance Company (Pacific
Life) for a twenty-year endowment policy in the life of Helen Go, his one year old daughter. Petitioner
Lapulapu D. Mondragon, the branch manager, prepared application form using the essential data
supplied by respondent. The latter paid the annual premium and Mondragon retained a portion of it as
his commission. The binding deposit receipt was issued to respondent. Mondragon wrote his strong
recommendation for the approval of the insurance application. However, Pacific Life disapproved the
application since the plan was not available for minors below 7 years old but it can consider the same
under another plan. The non-acceptance of the insurance plan was allegedly not communicated by
Mondragon to respondent. Mondragon again asserted his strong recommendation. Helen Go died of
influenza. Thereupon, respondent sought the payment of the proceeds of the insurance, but having
failed in his effort, he filed an action for the recovery of the same. Hence the case at bar.

ISSUE:
WON the binding deposit receipt constituted a temporary contract and thus negate the claim that the
insurance contract was perfected.

RULING:
YES. The provisions printed on the binding deposit receipt show that the binding deposit receipt is
intended to be merely a provisional or temporary insurance contract and only upon compliance of the
following conditions: (1) that the company shall be satisfied that the applicant was insurable on standard
rates; (2) that if the company does not accept the application and offers to issue a policy for a different
plan, the insurance contract shall not be binding until the applicant accepts the policy offered; otherwise,
the deposit shall be refunded; and (3) that if the applicant is not insurable according to the standard
rates, and the company disapproves the application, the insurance applied for shall not be in force at
any time, and the premium paid shall be returned to the applicant.
Clearly implied from the aforesaid conditions is that the binding deposit receipt in question is merely an
acknowledgment, on behalf of the company, that the latter's branch office had received from the
applicant the insurance premium and had accepted the application subject for processing by the
insurance company; and that the latter will either approve or reject the same on the basis of whether or
not the applicant is "insurable on standard rates." Since Pacific Life disapproved the insurance
application of Ngo Hing, the binding deposit receipt in question had never become in force at any time.
Upon this premise, the binding deposit receipt is, manifestly, merely conditional and does not insure
outright. Where an agreement is made between the applicant and the agent, no liability shall attach until
the principal approves the risk and a receipt is given by the agent. The acceptance is merely conditional,
and is subordinated to the act of the company in approving or rejecting the application.
Thus, in life insurance, a "binding slip" or "binding receipt" does not insure by itself. It bears repeating
that through the intra-company communication of 30 April 1957, Pacific Life disapproved the insurance
application in question on the ground that it is not offering the 20-year endowment insurance policy to
children less than 7 years of age. What it offered instead is another plan known as the Juvenile Triple
Action, which Ngo Hing failed to accept. In the absence of a meeting of the minds between Pacific Life
and Ngo Hing over the 20-year endowment life insurance in the amount of P50,000.00 in favor of the
latter's one-year old daughter, and with the non-compliance of the abovequoted conditions stated in the
disputed binding deposit receipt, there could have been no insurance contract duly perfected between
them. Accordingly, the deposit paid by Ngo Hing shall have to be refunded by Pacific Life.

23.Development Bank of the Philippines v CA 231 SCRA 370 (1994)

Facts:
Juan B. Dans, together with his family applied for a loan of P500,000 with DBP. As principal mortgagor,
Dans, then 76 years of age was advised by DBP to obtain a mortgage redemption insurance (MRI) with
DBP MRI pool. A loan in the reduced amount was approved and released by DBP. From the proceeds o
f the loan, DBP deducted the payment for the MRI premium. The MRI premium of Dans, less the DBP s
ervice fee of 10%, was credited by DBP to the savings account of DBP MRI-Pool. Accordingly, the DBP
MRI Pool was advised of the credit. Dans died of cardiac arrest. DBP MRI Pool notified DBP that Dans
was not eligible for MRI coverage, being over the acceptance age limit of 60 years at the time of applicat
ion. DBP apprised Candida Dans of the disapproval of her late husband’s MRI application. DBP offered t
o refund the premium which the deceased had paid, but Candida Dans refused to accept the same dem
anding payment of the face value of the MRI or an amount equivalent of the loan. She, likewise, refused
to accept an ex gratia settlement which DBP later offered. Hence, the case at bar.

Issue:
Whether or not the DBP MRI Pool should be held liable on the ground that the contract wasalready perfe
cted?

Held:
No, it is not liable. The power to approve MRI application is lodged with the DBP MRI Pool. The pool, ho
wever, did not approve the application. There is also no showing that it accepted the sum which DBP cre
dited to its account with full knowledge that it was payment for the premium. There was as a result no pe
rfected contract of insurance, hence the DBP MRI Pool cannot be held liable on a contract that does not
exist. In dealing with Dans, DBP was wearing 2 legal hats: the first as a lender and the second as an ins
urance agent. As an insurance agent, DBP made Dans go through the motion of applying for said insura
nce, thereby leading him and his family to believe that they had already fulfilled all the requirements for t
he MRI and that the issuance of their policy was forthcoming. DBP had full knowledge that the applicatio
n was never going to be approved. The DBP is not authorized to accept applications for MRI when its cli
ents are more than 60 years of age. . Knowing all the while that Dans was ineligible for MRI coverage be
cause of his advanced age, DBP exceeded the scope of its authority when it accepted Dan's application
for MRI by collecting the insurance premium, and deducting its agent's commission and service fee.
The liability of an agent who exceeds the scope of his authority depends upon whether the third person i
s aware of the limits of the agent's powers. There is no showing that Dans knew of the limitation on DBP'
s authority to solicit applications for MRI.
If the third person dealing with an agent is unaware of the limits of the authority conferred by the principa
l on the agent and he (third person) has been deceived by the non-disclosure thereof by the agent, then
the latter is liable for damages to him (V Tolentino, Commentaries and Jurisprudence on the Civil Code
of the Philippines, p. 422 [1992], citing Sentencia [Cuba] of September 25, 1907). The rule that the agen
t is liable when he acts without authority is founded upon the supposition that there has been some wron
g or omission on his part either in misrepresenting, or in affirming, or concealing the authority under whic
h he assumes to act (Francisco, V., Agency 307 [1952], citing Hall v. Lauderdale, 46 N.Y. 70, 75). Inasm
uch as the non-disclosure of the limits of the agency carries with it the implication that a deception was p
erpetrated on the unsuspecting client, the provisions of Articles 19, 20 and 21 of the Civil Code of the Ph
ilippines come into play.

24.Perez v CA G.R. No. 112329. January 28, 2000

Facts:
Primitivo B. Perez had been insured with the BF Lifeman Insurance Corporation for P20,000.00.
Sometime in October 1987, an agent of the insurance corporation, visited Perez in Quezon and
convinced him to apply for additional insurance coverage of P50,000.00. Virginia A. Perez, Primitivo’s
wife, paid P2,075.00 to the agent. The receipt issued indicated the amount received was a "deposit."
Unfortunately, the agent lost the application form accomplished by Perez and he asked the latter to fill up
another application form. The agent sent the application for additional insurance of Perez to the Quezon
office. Such was supposed to forwarded to the Manila office.
Perez drowned. His application papers for the additional insurance of P50,000.00 were still with the
Quezon. It was only after some time that the papers were brought to Manila. Without knowing that Perez
died, BF Lifeman Insurance Corporation approved the application and issued the corresponding policy
for the P50,000.00.
Petitioner Virginia Perez went to Manila to claim the benefits under the insurance policies of the
deceased. She was paid P40,000.00 under the first insurance policy for P20,000.00 but the insurance
company refused to pay the claim under the additional policy coverage of P50,000.00, the proceeds of
which amount to P150,000.00.
The insurance company maintained that the insurance for P50,000.00 had not been perfected at the
time of the death of Primitivo Perez. Consequently, the insurance company refunded the amount paid.
BF Lifeman Insurance Corporation filed a complaint against Virginia Perez seeking the rescission and
declaration of nullity of the insurance contract in question.
Petitioner Virginia A. Perez, on the other hand, averred that the deceased had fulfilled all his prestations
under the contract and all the elements of a valid contract are present.
On October 25, 1991, the trial court rendered a decision in favor of petitioner ordering respondent to pay
150,000 pesos. The Court of Appeals, however, reversed the decision of the trial court saying that the
insurance contract for P50,000.00 could not have been perfected since at the time that the policy was
issued, Primitivo was already dead.
Petitioner’s motion for reconsideration having been denied by respondent court, the instant petition for
certiorari was filed on the ground that there was a consummated contract of insurance between the
deceased and BF Lifeman Insurance Corporation.

Issue:
WON the widow can receive the proceeds of the 2nd insurance policy

Held:
No. Petition dismissed.
Ratio:
Perez’s application was subject to the acceptance of private respondent BF Lifeman Insurance
Corporation. The perfection of the contract of insurance between the deceased and respondent
corporation was further conditioned with the following requisites stated in the application form:
"there shall be no contract of insurance unless and until a policy is issued on this application and that the
said policy shall not take effect until the premium has been paid and the policy delivered to and accepted
by me/us in person while I/We, am/are in good health."
BF Lifeman didn’t give its assent when it merely received the application form and all the requisite
supporting papers of the applicant. This happens only when it gives a policy.
It is not disputed, however, that when Primitivo died on November 25, 1987, his application papersfor
additional insurance coverage were still with the branch office of respondent corporation in Quezon.
Consequently, there was absolutely no way the acceptance of the application could have been
communicated to the applicant for the latter to accept inasmuch as the applicant at the time was already
dead.
Petitioner insists that the condition imposed by BF that a policy must have been delivered to and
accepted by the proposed insured in good health is potestative, being dependent upon the will of the
corporation and is therefore void. The court didn’t agree. A potestative condition depends upon the
exclusive will of one of the parties and is considered void. The Civil Code states: When the fulfillment of
the condition depends upon the sole will of the debtor, the conditional obligation shall be void.
The following conditions were imposed by the respondent company for the perfection of the contract of
insurance: a policy must have been issued, the premiums paid, and the policy must have been delivered
to and accepted by the applicant while he is in good health.
The third condition isn’t potestative, because the health of the applicant at the time of the deliveryof the
policy is beyond the control or will of the insurance company. Rather, the condition is a suspensive one
whereby the acquisition of rights depends upon the happening of an event which constitutes the
condition. In this case, the suspensive condition was the policy must have been delivered and accepted
by the applicant while he is in good health. There was non-fulfillment of the condition, because the
applicant was already dead at the time the policy was issued.
As stated above, a contract of insurance, like other contracts, must be assented to by both parties either
in person or by their agents. So long as an application for insurance has not been either accepted or
rejected, it is merely an offer or proposal to make a contract. The contract, to be binding from the date of
application, must have been a completed contract.
The insurance company wasn’t negligent because delay in acting on the application does not constitute
acceptance even after payment. The corporation may not be penalized for the delay in the processing of
the application papers due to the fact that process in a week wasn’t the usual timeframe in fixing the
application. Delay could not be deemed unreasonable so as to constitute gross negligence.

25.PHILAMCARE HEALTH SYSTEMS, INC., petitioner, vs. COURT OF APPEALS and JULITA TRINOS,
respondents. 379 SCRA 356 (2002)

Facts
Ernani Trinos, deceased husband of respondent Julita Trinos, applied for a health care coverage with
petitioner Philamcare Health Systems, Inc. Accordingly, he was issued Health Care Agreement No.
P010194. Under the agreement, respondent's husband was entitled to avail of hospitalization benefits,
whether ordinary or emergency, listed therein. The amount of coverage was increased to a maximum
sum of P75,000.00 per disability. During the period of his coverage, Ernani suffered a heart attack and
was confined at the Manila Medical Center (MMC) for one month beginning March 9, 1990. While her
husband was in the hospital, respondent tried to claim the benefits under the health care agreement.
However, petitioner denied her claim saying that the Health Care Agreement was void. According to
petitioner, there was a concealment regarding Ernani's medical history. Doctors at the MMC allegedly
discovered at the time of Ernani's confinement that he was hypertensive, diabetic and asthmatic,
contrary to his answer in the application form. Thus, respondent paid the hospitalization expenses
herself, amounting to about P76,000.00.
After her husband was discharged from the MMC, he was attended by a physical therapist at home.
Respondent was constrained to bring him back to the Chinese General Hospital where he died on the
same day.
After trial, the lower court ruled against petitioners. On appeal, the Court of Appeals affirmed the
decision of the trial court but deleted all awards for damages and absolved petitioner Reverente.

Issue:
WON a health care agreement is an insurance contract

Ruling:
An insurance contract exists where the following elements concur: 1. The insured has an insurable
interest; 2. The insured is subject to a risk of loss by the happening of the designated peril; 3. The
insurer assumes the risk; 4. Such assumption of risk is part of a general scheme to distribute actual
losses among a large group of persons bearing a similar risk; and 5. In consideration of the insurer's
promise, the insured pays a premium.
In the case at bar, the insurable interest of respondent's husband in obtaining the health care agreement
was his own health. The health care agreement was in the nature of non-life insurance, which is
primarily a contract of indemnity. Once the member incurs hospital, medical or any other expense
arising from sickness, injury or other stipulated contingent, the health care provider must pay for the
same to the extent agreed upon under the contract.

26.GULF RESORTS, INC., petitioner, vs. PHILIPPINE CHARTER INSURANCE CORPORATION,


respondent. 458 SCRA 550 (2005)

Facts:
Gulf Resorts is the owner of the Plaza Resort situated at Agoo, La Union and had its properties in said
resort insured originally with the American Home Assurance Company (AHAC). In the first 4 policies
issued, the risks of loss from earthquake shock was extended only to petitioner’s two swimming pools.
Gulf Resorts agreed to insure with Phil Charter the properties covered by the AHAC policy provided that
the policy wording and rates in said policy be copied in the policy to be issued by Phil Charter. Phil
Charter issued Policy No. 31944 to Gulf Resorts covering the period of March 14, 1990 to March 14,
1991 for P10,700,600.00 for a total premium of P45,159.92. the break-down of premiums shows that
Gulf Resorts paid only P393.00 as premium against earthquake shock (ES). On July 16, 1990 an
earthquake struck Central Luzon and Northern Luzon and plaintiff’s properties covered by Policy No.
31944 issued by defendant, including the two swimming pools in its Agoo Playa Resort were damaged.
Petitioner advised respondent that it would be making a claim under its Insurance Policy 31944 for
damages on its properties. Respondent denied petitioner’s claim on the ground that its insurance policy
only afforded earthquake shock coverage to the two swimming pools of the resort. The trial court ruled in
favor of respondent. In its ruling, the schedule clearly shows that petitioner paid only a premium of
P393.00 against the peril of earthquake shock, the same premium it had paid against earthquake shock
only on the two swimming pools in all the policies issued by AHAC.

Issue:
Whether or not the policy covers only the two swimming pools owned by Gulf Resorts and does not
extend to all properties damaged.

RULLING:
YES, it only covers the 2 swimming pools. In sum, there is no ambiguity in the terms of the contract and
its riders. From the inception of the policy, petitioner had required the respondent to copy verbatim the
provisions and terms of its latest insurance policy from AHAC-AIU. All the provisions and riders taken
and interpreted together, indubitably show the intention of the parties to extend earthquake shock
coverage to the two swimming pools only. An insurance premium is the consideration paid an insurer for
undertaking to indemnify the insured against a specified peril. In fire, casualty and marine insurance, the
premium becomes a debt as soon as the risk attaches. In the subject policy, no premium payments were
made with regard to earthquake shock coverage except on the two swimming pools. There is no
mention of any premium payable for the other resort properties with regard to earthquake shock.

SUBROGATION

27.MALAYAN INSURANCE CO., INC. vs. THE HON. COURT OF APPEALS 165 SCRA 536

FACTS:
Sio Choy insured his jeep with Malayan Insurance against 3rd party liability. One day the jeep, driven by
an employee of San Leon Rice Mill, figured in an accident with Pantranco Bus.
The passenger of the jeep, Vallejo, who was injured due to the accident, claimed damages from Sio
Choy, Malayan and Pantranco. Pantranco was held not liable.
Malayan insurance paid Vallejo and asked for reimbursement from San Leon as the latter driver caused
the alleged accident. The latter, however denied liability.
RTC ruled that Sio Choy, Malayan and San Leon are solidary liable, thus, the former is entitled to
reimbursement.
CA said although jointly and severally liable, Malayan is not entitled to reimbursement.

ISSUES:
1. WON Sio Choy, Malayan and San Leon Rice Mill are solidary liable.
2. WON Malayan can seek reimbursement.

RULING:
1. Only respondents Sio Choy and San Leon Rice Mill, Inc, (to the exclusion of the petitioner) that
are solidarily liable to respondent Vallejos for the damages awarded to Vallejos.
Sio Choy and San Leon Rice Mill, Inc. are the principal tortfeasors who are primarily liable to respondent
Vallejos. The law states that the responsibility of two or more persons who are liable for a quasi-delict is
solidarily. On the other hand, the basis of petitioner's liability is its insurance contract with respondent
Sio Choy.

While it is true that where the insurance contract provides for indemnity against liability to third persons,
such third persons can directly sue the insurer, 6 however, the direct liability of the insurer under
indemnity contracts against third party liability does not mean that the insurer can be held solidarily liable
with the insured and/or the other parties found at fault. The liability of the insurer is based on contract;
that of the insured is based on tort.
In the case at bar, petitioner as insurer of Sio Choy, is liable to respondent Vallejos, but it cannot, as
incorrectly held by the trial court, be made "solidarily" liable with the two principal tortfeasors namely
respondents Sio Choy and San Leon Rice Mill, Inc. For if petitioner-insurer were solidarily liable with
said two (2) respondents by reason of the indemnity contract against third party liability-under which an
insurer can be directly sued by a third party — this will result in a violation of the principles underlying
solidary obligation and insurance contracts.

2. Malayan is entitled to re-imbursement from San Leon by virtue of SUBROGATION. Article 1217
says,
Art. 1217. Payment made by one of the solidary debtors extinguishes the obligation. If two or more
solidary debtors offer to pay, the creditor may choose which offer to accept.
He who made the payment may claim from his co-debtors only the share which corresponds to each,
with the interest for the payment already made. If the payment is made before the debt is due, no
interest for the intervening period may be demanded.
In accordance with Article 1217, MALAYAN, upon payment to Vallejos and thereby becoming the
subrogee of solidary debtor Sio Choy, is entitled to reimbursement from respondent San Leon Rice Mill,
Inc.
28. MANILA MAHOGANY MFG CORP V CA & ZENITH INSURANCE
FACTS:
Manila Mahogany insured its Mercedes Benz with respondent insurance company. One day, the vehicle
was bumped and damaged by a truck owned by San Miguel Corp (SMC).

Zenith paid P5K to petitioner in amicable settlement. Petitioner’s general manager executed a Release
Claim, subrogating respondent company to all its right to action against SMC.
Later respondent wrote Insurance Adjusters Inc. to demand reimbursement from SMC. Insurance
Adjusters refused saying that SMC had already paid petitioner P4,500 for the damages to petitioner’s
vehicle, as evidenced by a cash voucher and Release of Claim executed by the GM of petitioner
discharging SMC from “all actions, claims, demands the rights of action that now exist or hereafter
develop arising out of or as a consequence of the accident.
Respondent demanded the P4.5K amount from petitioner. Petitioner refused. Suit filed for recovery.
City Court ordered petitioner to pay respondent. CFI affirmed. CA affirmed with modification that
petitioner was to pay respondent the total amount of 5K it had received from respondent.
Petitioner’s argument: Since the total damages were valued at P9,486.43 and only 5K was received by
petitioner from respondent, petitioner argues that it was entitled to go after SMC to claim the additional
which was eventually paid to it.
Respondent’s argument: No qualification to its right of subrogation.

ISSUE:
1.WON petitioner should pay respondent despite the subrogation in the Release of Claim was
conditioned on recovery of the total amount of damages petitioner has sustained.

RULING:
1. NO. SC said no other evidence to support its allegation that a gentleman’s agreement existed
between the parties, not embodied in the Release of Claim, such Release of Claim must be taken as the
best evidence of the intent and purpose of the parties.
CA correct in holding petitioner should reimburse respondent 5K.

When Manila Mahogany executed another release claim discharging SMC from all rights of action after
the insurer had paid the proceeds of the policy – the compromise agreement of 5K- the insurer is
entitled to recover from the insured the amount of insurance money paid.
Petitioner by its own acts released SMC, thereby defeating respondent’s right of subrogation, the right of
action against the insurer was also nullified.
Since the insurer can be subrogated to only such rights as the insured may have, should the insured,
after receiving payment from the insurer, release the wrongdoer who caused the loss, the insurer losses
his rights against the latter. But in such a case, the insurer will be entitled to recover from the insured
whatever it has paid to the latter, unless the release was made w/ the consent of the insurer.

29.PAN MALAYAN INSURANCE CORP. VS. COURT OF APPEALS


FACTS:
Pan Malayan filed a complaint for damages with the RTC of Makati against private respondents Erlinda
Fabie and her driver. Pan Malayan insured a Mitsubishi Colt Lancer car registered in the name of
Canlubang. Due to the carelessness, recklessness and imprudence of the unknown driver of a pick-up,
the insured car was hit and suffered damages in the amount of P42,052.00. Pan Malayan defrayed the
cost of repair of the insured car, and therefore was subrogated to the rights of Canlubang against the
driver of the pick-up and his employer, Erlinda Fabie. Despite repeated demands, defendants failed and
refused to pay the claim of Pan Malay. Defendants/Private Respondents alleged that Pan Malay had no
cause of action against them because payment under the “own damage” clause of the insurance policy
precluded subrogation under Article 2207 of the Civil Code, since indemnification thereunder was made
on the assumption that there was no wrongdoer or no third party at fault. RTC dismissed the case for no
cause of action and denied its motion for reconsideration. The CA affirmed the trial courts decision.
Hence, this petition.

ISSUES:
Whether or not the insurer Pan Malayan may institute ac action to recover the amount it had paid its
assured in settlement of an insurance claim against private respondents.

RULING:
Pan Malayan is correct.
If the insured property is destroyed or damaged through the fault or negligence of a party other than the
assured, then the insurer, upon payment to the assured, will be subrogated to the rights of the assured
to recover from the wrongdoer to the extent that the insurer has been obligated to pay. Payment by the
insurer to the assured operates as an equitable assignment to the former of all remedies, which the
latter ma have against the third party whose negligence or wrongful act caused the loss. The right of
subrogation is not dependent upon, nor does it grow out of any privity of contract or upon written
assignment of claim. It accrues simply upon payment of the insurance claim by the insurer.

30.CEBU SHIPPING AND ENGINEERING WORKS, INC. VS. WILLIAM LINES INC. AND PRUDENTIAL
GUARANTEE AND ASSURANCE COMPANY, INC.

FACTS:
William Lines, Inc. brought its vessel M/V Manila City to the Cebu Shipyard in Lapulapu City for annual
dry-docking and repair. Subject vessel was insured with Prudential Guarantee for P45,000,000.00 for
hull and machinery. The Hull Policy included an “Additional Perils” clause covering loss of or damage to
the vessel through the negligence of, among others, ship repairmen. CSEW was also insured by
Prudential Guarantee for third party liability under s Shiprepairs Legal Liability Insurance Policy for
P10,000,000.00 only. After subject vessel was transferred to the docking quay, it caught fire and sank,
resulting to its eventual total loss. William Lines, Inc. filed a complaint for damages against CSEW,
alleging that the fire which broke out in M/V Manila City was caused by CSEW’s negligence and lack of
care. An amended complaint, impleading Prudential Guarantee as co-plaintiff, was filed after the latter
had paid William Lines, Inc. the value of the hull and machinery insurance of M/V Manila City. RTC
ruled that the cause of the fire was through the negligence of CSEW. CA affirmed the appealed decision.
Hence this petition.

ISSUE:
Whether or not Prudential has the right of subrogation against its own insured and whether or not the
parties intended for them to be a co-assured in the insurance policy.

RULING:
The petition is unmeritorious.
Upon proof of payment by Prudential Guarantee to William Lines, the former was subrogated to the right
of the latter to indemnification from CSEW. Thus, when Prudential, after due verification of the merit and
validity of the insurance claim of William Lines, paid the latter the total amount covered by its insurance
policy, it was subrogated to the right of the latter to recover the insured loss from CSEW, the liable party.
A stipulation in the work order that requires William Lines to maintain insurance on the vessel during the
period of dry-docking or repair, works to the benefit of CSEW. However, the fact that CSEW benefits
from the said stipulation does not automatically make it as a co-assured of William Lines. The hull and
machinery insurance procured by William Lines, Inc. from Prudential named only "William Lines, Inc." as
the assured. Thus, when the insurance policy involved named only William Lines, Inc. as the assured
thereunder, the claim of CSEW that it is a co-assured is unfounded.

31. Delsan Transport Lines, Inc. vs. Court of Appeals and American Home Assurance Corporation
FACTS:
Caltex Philippines entered into a contract of affreightment with the petitioner, Delsan Transport Lines,
Inc., whereby the said common carrier agreed to transport Caltex's industrial fuel oil from the Batangas-
Bataan Refinery to different parts of the country.
The shipment was insured with the private respondent, American Home Assurance Corporation. On
August 14, 1986, petitioner's vessel, the MT Maysun, set sail from Batangas for Zamboanga City.
Unfortunately, the vessel sank in the early morning of August 16, 1986 near Panay Gulf in the Visayas
taking with it the entire cargo of fuel oil.
AHAC paid Caltex the sum of P5,096,635.57 representing the insured value of the lost cargo. Exercising
its right of subrogation under Article 2207 of the New Civil Code, AHAC demanded from Delsan the
same amount it paid to Caltex.
Due to its failure to collect from the petitioner despite prior demand, private respondent filed a complaint
with the Regional Trial Court of Makati City, Branch 137, for collection of a sum of money.
The trial court rendered a decision dismissing the complaint against herein petitioner. The trial court
found that the vessel, MT Maysun, was seaworthy to undertake the voyage and that the incident was
caused by unexpected inclement weather condition or force majeure, thus exempting petitioner from
liability for the loss of its cargo. The decision of the trial court, however, was reversed, on appeal, by the
Court of Appeals.

ISSUES:
(pertaining to subrogation)
1.Whether or not the payment made by the private respondent to Caltex for the insured value of the lost
cargo amounted to an admission that the vessel was seaworthy, thus precluding any action for recovery
against the petitioner?
2.Whether or not the non-presentation of the marine insurance policy bars AHAC’s right of subrogation?

RULING:
First Issue:
Before the Court, petitioner theorized that when private respondent paid Caltex the value of its lost cargo,
the act of the private respondent is equivalent to a tacit recognition that the ill-fated vessel was
seaworthy; otherwise, private respondent was not legally liable to Caltex due to the latter's breach of
implied warranty under the marine insurance policy that the vessel was seaworthy.
The Supreme Court rejected petitioner's theory. According to the Court, the payment made by the
private respondent for the insured value of the lost cargo operates as a waiver of private respondent's
right to enforce the term of the implied warranty against Caltex under the marine insurance policy.
However, the same cannot be validly interpreted as an automatic admission of the vessel's
seaworthiness by the private respondent as to foreclose recourse against the petitioner for any liability
under its contractual obligation as a common carrier. The fact of payment grants the private respondent
subrogatory right which enables it to exercise legal remedies that would otherwise be available to Caltex
as owner of the lost cargo against the petitioner common carrier.
The Court also stressed that the right of subrogation is designed to promote and to accomplish justice
and is the mode which equity adopts to compel the ultimate payment of a debt by one who in justice and
good conscience ought to pay.
It is not dependent upon, nor does it grow out of, any privity of contract or upon written assignment of
claim. It accrues simply upon payment by the insurance company of the insurance claim. Consequently,
the payment made by AHAC (insurer) to Caltex (assured) operates as an equitable assignment to the
former of all the remedies which the latter may have against the petitioner.
Second Issue:
Anent the second issue, it is our view and so hold that the presentation in evidence of the marine
insurance policy is not indispensable in this case before the insurer may recover from the common
carrier the insured value of the lost cargo in the exercise of its subrogatory right.
The subrogation receipt, by itself, is sufficient to establish not only the relationship of herein private
respondent as insurer and Caltex, as the assured shipper of the lost cargo of industrial fuel oil, but also
the amount paid to settle the insurance claim. The right of subrogation accrues simply upon payment by
the insurance company of the insurance claim.

32. Federal Express Corporation vs. American Home Assurance Company and PHILAM Insurance
Company

FACTS:
On January 26, 1994, SMITHKLINE Beecham of Nebraska, USA delivered to Burlington Air Express
(AGENT OF FEDREAL EXPRESS) a shipment of 109 cartons of veterinary biologicals for delivery to
consignee SMITHKLINE and French Overseas Company in Makati City, Metro Manila.
The shipment was covered by Burlington Airway Bill No. 11263825 with the words, ‘REFRIGERATE
WHEN NOT IN TRANSIT’ and ‘PERISHABLE’ stamp marked on its face. That same day, Burlington
insured the cargoes in the amount of $39,339.00 with AHAC.
The following day, Burlington turned over the custody of said cargoes to Federal Express which
transported the same to Manila. The first shipment, consisting of 92 cartons arrived in Manila on January
29, 1994 and was immediately stored at Cargohaus Inc.’s warehouse.
While the second, consisting of 17 cartons, came in two (2) days later, or on January 31, 1994,which
was likewise immediately stored at Cargohaus’ warehouse.
12 days later, the Customs Broker who was assigned by Smithkline of Makati to facilitate the withdrawal
of the Cargoes, did not proceed with such withdrawal for He found out that the Cartons containing the
vaccines were not properly stored as ordered. For this reason, the vaccines were examined, only to find
out that they were damaged and unusable. Consequently Smithkline of Makati abandoned the shipment.
Smithkline of Makati filed a claim with PHILAM, the representative of AHAC in the Philippines. By virtue
of its right of subrogation, AHAC proceeded against FEDERAL EXPRESS.
Federal Express declined the claim of AHAC contending that the latter had no cause of action against
the former. Moreover, Federal Express contended that no notice of claim was filed, hence, not complying
with the condition precedent, AHAC was precluded from asserting its claim against it.

ISSUES:
1.Whether or not AHAC has legal personality to sue, thus, no cause of action against Federal Express?
2.Whether or not AHAC complied with the necessary condition precedent in order to file claims against
Federal Express?

RULING:
First Issue:
Federal Express argued that payment was erroneous for the proper payment should have been made to
Burlington as agent of Federal Express, and as payee of the bill.
Held, Smithkline of Makatin has the personality to claim for the damages because the Certificate of
Insurance is payable to the bearer thereof. Upon payment by AHAC to Smithkline, the latter executed a
subrogation receipt. Hence, AHAC and PHILAM have personality to file claims.

Upon payment to the consignee of an indemnity for the loss of or damage to the insured goods, the
insurer’s entitlement to subrogation pro tanto — being of the highest equity — equips it with a cause of
action in case of a contractual breach or negligence. Further, the insurer’s subrogatory right to sue for
recovery under the bill of lading in case of loss of or damage to the cargo is jurisprudentially upheld.”
Second Issue:
Under the Warsaw Convention, Notice of Claim is a condition precedent to the accrual of a Right of
Action against a carrier for loss or damage to the goods. Being a condition precedent, it must precede a
suit for enforcement. In the instant case, AHAC never complied such requirement. Thus, it cannot file
claims against Federal Express.

INSURABLE INTEREST

33. Spouses Cha vs. CA


Lessons Applicable: Effect of Lack of Insurable Interest (Insurance)
Laws Applicable: Sec. 17, Sec. 18, Sec. 25 of the Insurance Code

FACTS:
Spouses Nilo Cha and Stella Uy-Cha and CKS Development Corporation entered a 1 year lease
contract with a stipulation not to insure against fire the chattels, merchandise, textiles, goods and effects
placed at any stall or store or space in the leased premises without first obtaining the written consent
and approval of the lessor. But it insured against loss by fire their merchandise inside the leased
premises for P500,000 with the United Insurance Co., Inc. without the written consent of CKS
On the day the lease contract was to expire, fire broke out inside the leased premises and CKS learning
that the spouses procured an insurance wrote to United to have the proceeds be paid directly to them.
But United refused so CKS filed against Spouses Cha and United.
RTC: United to pay CKS the amount of P335,063.11 and Spouses Cha to pay P50,000 as exemplary
damages, P20,000 as attorney’s fees and costs of suit
CA: deleted exemplary damages and attorney’s fees

ISSUE:
W/N the CKS has insurable interest because the spouses Cha violated the stipulation
HELD:
NO. CA set aside. Awarding the proceeds to spouses Cha.
Sec. 18. No contract or policy of insurance on property shall be enforceable except for the benefit of
some person having an insurable interest in the property insured
A non-life insurance policy such as the fire insurance policy taken by petitioner-spouses over their
merchandise is primarily a contract of indemnity. Insurable interest in the property insured must exist a t
the time the insurance takes effect and at the time the loss occurs. The basis of such requirement of
insurable interest in property insured is based on sound public policy: to prevent a person from taking
out an insurance policy on property upon which he has no insurable interest and collecting the proceeds
of said policy in case of loss of the property. In such a case, the contract of insurance is a mere wager
which is void under Section 25 of the Insurance Code.
SECTION 25. Every stipulation in a policy of Insurance for the payment of loss, whether the person
insured has or has not any interest in the property insured, or that the policy shall be received as proof
of such interest, and every policy executed by way of gaming or wagering, is void
Section 17. The measure of an insurable interest in property is the extent to which the insured might be
damnified by loss of injury thereof
The automatic assignment of the policy to CKS under the provision of the lease contract previously
quoted is void for being contrary to law and/or public policy. The proceeds of the fire insurance policy
thus rightfully belong to the spouses. The liability of the Cha spouses to CKS for violating their lease
contract in that Cha spouses obtained a fire insurance policy over their own merchandise, without the
consent of CKS, is a separate and distinct issue which we do not resolve in this case.

34. Great Pacific Life vs. CA


Facts:
Great Pacific Life Assurance Corporation (Grepalife) executed a contract of group life insurance with
Development Bank of the Philippines (DBP) wherein Grepalife agreed to insure the lives of eligible
housing loan mortgagors of DBP.
One such loan mortgagor is Dr. Wilfredo Leuterio. In an application form, Dr. Leuterio answered
questions concerning his test, attesting among others that he does not have any heart conditions and
that he is in good health to the best of his knowledge.
However, after about a year, Dr. Leuterio died due to “massive cerebral hemorrhage.” When DBP
submitted a death claim to Grepalife, the latter denied the claim, alleging that Dr. Leuterio did not
disclose he had been suffering from hypertension, which caused his death. Allegedly, such non-
disclosure constituted concealment that justified the denial of the claim.
Hence, the widow of the late Dr. Leuterio filed a complaint against Grepalife for “Specific Performance
with Damages.” Both the trial court and the Court of Appeals found in favor of the widow and ordered
Grepalife to pay DBP.

ISSUE:
Whether the CA erred in holding Grepalife liable to DBP as beneficiary in a group life insurance contract
from a complaint filed by the widow of the decedent/mortgagor

HELD:
The rationale of a group of insurance policy of mortgagors, otherwise known as the “mortgage
redemption insurance,” is a device for the protection of both the mortgagee and the mortgagor. On the
part of the mortgagee, it has to enter into such form of contract so that in the event of the unexpected
demise of the mortgagor during the subsistence of the mortgage contract, the proceeds from such
insurance will be applied to the payment of the mortgage debt, thereby relieving the heirs of the
mortgagor from paying the obligation. In a similar vein, ample protection is given to the mortgagor under
such a concept so that in the event of death, the mortgage obligation will be extinguished by the
application of the insurance proceeds to the mortgage indebtedness. In this type of policy insurance, the
mortgagee is simply an appointee of the insurance fund. Such loss-payable clause does not make the
mortgagee a party to the contract.
The insured, being the person with whom the contract was made, is primarily the proper person to bring
suit thereon. Subject to some exceptions, insured may thus sue, although the policy is taken wholly or in
part for the benefit of another person, such as a mortgagee.
And since a policy of insurance upon life or health may pass by transfer, will or succession to any person,
whether he has an insurable interest or not, and such person may recover it whatever the insured might
have recovered, the widow of the decedent Dr. Leuterio may file the suit against the insurer, Grepalife.
35. Harvardian Colleges c Country Bankers Insurance
Facts:
Harvardian Colleges is a family corporation whose stockholders are Ildefonso Yap, Virginia King Yap and
their children. Harvardian Colleges insured the school for fire with CBI for Php500,000. However, the
insured property was burned which resulted to its total loss. Harvardian Colleges made claims but was
denied by CBI on the ground that there was no insurable interest iver the building on the piece of land
which was in the name of Ildefonso Yap, and not of Harvardian Colleges.

Issue:
Whether or not Harvardian has insurable interest and can collect from the insurance
Ruling:
Yes. Regardless of the nature of the title of the insures, or even if he did not have title to the property
insured, the contract of fire insurance should still be upheld if his interest in or his relation to the property
is such that he will be benefited in its continuing existence or suffer a direct pecuniary loss from its
destruction or injury. The test in determining insurable interest in property is whether one will derive
pecuniary benefit or advantage from its preservation or will suffer pecuniary loss or damage from it
destruction by the happening of the event insured against.

36. Ang Ka Yu v. Phoenix Assurance


Facts:
Ang Ka Yu had a piece of property in his possession. He insured it with Phoenix.
The property was lost, so Ang Ka Yu sought to claim the proceeds.
Phoenix denied liability on the ground that Ang was not the owner but a mere possessor and as such,
had no insurable interest over the property.

Issue:
WON a mere possessor has insurable interest over the property.

Held:
Yes. A person having a mere right or possession of property may insure it to its full value and in his own
name, even when he is not responsible for its safekeeping. The reason is that even if a person is NOT
interested in the safety and preservation of material in his possession because they belong to 3rd
parties, said person still has insurable interest, because he stands either to benefit from their continued
existence or to be prejudiced by their destruction.

CONCEALMENT AND REPRESENTATION

37. THE INSULAR LIFE ASSURANCE CO., LTD., petitioner, vs. SERAFIN D. FELICIANO and ANGEL,
FLORENDA, EUGENIO, HERMINIO and LETICIA, all surnamed FELICIANO, represented by their
guardian ad litem SERAFIN D. FELICIANO, respondents (G.R. No. L-47593, September 13, 1941)
FACTS:
- One Evaristo Feliciano filed an application for insurance with the herein petitioner upon the
solicitation of one of its agents. Two insurance policies to the aggregate amount of P25,000 were issued
to him.
- Feliciano died on September 29, 1935. The defendant company (petitioner) refused to pay on the
ground that the policies were fraudulently obtained, the insured having given false answers and
statements in the application as well as in the medical report.
- The present action was brought to recover on said policies.
- Lower court in favor of plaintiff (respondent)finding that:
• Feliciano was made to sign the application and the examiner's report in blank, and that
afterwards the blank spaces therein were filled in by the agent (Romulo M. David ) and the medical
examiner (Dr. Gregorio Valdez), who made it appear therein that Feliciano was a fit subject for insurance.
• neither the insured nor any member of his family concealed the real state of health of the insured;
that as a matter of fact the insured, as well as the members of his family, told the agent and the medical
examiner that the applicant had been sick and coughing for sometime and that he had also gone three
times to the Santol Sanatarium.
- CA affirmed. Hence, this petition.
ISSUE:
WON the policy remains to be valid in spite of the fact that the agent, without fraud, collusion or bad faith
on the part of the insured, falsified the answers given by the insured.

HELD:
YES
- Insurance companies send detailed instructions to their agents to solicit and procure applications.
These agents are to be found all over the length and breadth of the land. The agents, in short, do what
the company set them to do.
- In the present case, the agent knew all the time the true state of health of the insured. The
insurer's medical examiner approve the application knowing full well that the applicant was sick.
- The situation is one in which one of two innocent parties must bear a loss for his reliance upon a
third person.
- In this case, it was the insurer who gave the agent authority to deal with the applicant. It was the
one who selected the agent, thus implying that the insured could put his trust on him. It seems
reasonable that as between the two of them, the one who employed and gave character to the third
person as its agent should be the one to bear the loss.
- If the policy should be avoided, it must be because it was void from the very beginning.
- The insurer cannot assert the falsity of such answers as a defense to liability on the policy.
- The fact that the insured did not read the application which he signed, is not indicative of bad
faith. It has been held that it is not negligence for the insured to sign an application without first reading it
if the insurer by its conduct in appointing the agent influenced the insured to place trust and confidence
in the agent.
- In the instant case, it has been proved that the insured could not read English, the language in
which the application was written, and that after the contract was signed, it was kept by his mother. As a
consequence, the insured had no opportunity to read or correct any misstatement therein.
- Petition dismissed.

38.THE INSULAR LIFE ASSURANCE CO., LTD., petitioner, vs.SERAFIN D. FELICIANO ET AL.,
respondents. (G.R. No. L-47593 December 29, 1943)
MOTION FOR RECONSIDERATION

FACTS:
- A motion to reconsider and set aside said decision has been filed by the petitioner, and both
parties have submitted exhaustive and luminous written arguments in support of their respective
contentions.
- Agent’s reason for falsifying the application: for the purpose of securing the Company's approval
of the application so that the policy to be issued thereon might be credited to said agent in connection
with the inter-provincial contest which the Company was then holding among its soliciting agents to
boost the sales of its policies.
- Moreover, Agent David bribed Medical Examiner Valdez with money which the former borrowed
from the applicant's mother by way of advanced payment on the premium, according to the finding of the
Court of Appeals.
- petitioner insists: that upon the facts of the case the policies in question are null and void ab initio
and that all that the respondents are entitled to is the refund of the premiums paid thereon.

ISSUE
WON Policy still valid.

HELD:
- When the applicant for insurance, signed the application in blank and authorized the soliciting
agent and/or the medical examiner of the Company to write the answers for him, he made them his own
agents for that purpose, and he was responsible for their acts in that connection. If they falsified the
answers for him, he could not evade the responsibility for the falsification. He was not supposed to sign
the application in blank. He knew that the answers to the questions therein contained would be "the
basis of the policy," and for that very reason he was required with his signature to vouch for the truth
thereof.
- By accepting the policy he became charged with knowledge of its contents, whether he actually
read it or not.
- We cannot bring ourselves to believe that the insured did not take the trouble to read the
answers contained in the photostatic copy of the application attached to and made a part of the policy
before he accepted it and paid the premium thereon. He must have notice that the answers to the
questions therein asked concerning his clinical history were false, and yet he accepted the first policy
and applied for another.
- The insured, therefore, had no right to rely — and we cannot believe he relied in good faith —
upon the oral representation of said agent and medical examiner that he (the applicant) was a fit subject
for insurance notwithstanding that he had been and was still suffering with advanced pulmonary
tuberculosis.
- Altho the agent and the medical examiner knew that statement to be false, no valid contract of
insurance was entered into because there was no real meeting of the minds of the parties.
- From all the facts and circumstances of this case, we are constrained to conclude that the
insured was a coparticipant, and coresponsible with Agent David and Medical Examiner Valdez, in the
fraudulent procurement of the policies in question and that by reason thereof said policies are void ab
initio.
- MR sustained. CA reversed in favor of Petitioner Company.

39.SUNLIFE ASSURANCE COMPANY OF CANADA, petitioner, vs. The Hon. COURT OF APPEALS
and Spouses ROLANDO and BERNARDA BACANI, respondents.
FACTS :
On April 15, 1986, Robert John B. Bacani procured a life insurance contract for himself from petitioner
and was issued a policy valued at P100,000.00, with double indemnity in case of accidental death. The
designated beneficiary was his mother, respondent Bernarda Bacani.

On June 26, 1987, the insured died in a plane crash. Respondent Bernarda Bacani filed a claim with
petitioner, seeking the benefits of the insurance policy taken by her son. Petitioner conducted an
investigation and its findings prompted it to reject the claim on the ground that the insured did not
disclosed material facts relevant to the issuance of the policy, thus rendering the contract of insurance
voidable. A check representing the total premiums paid in the amount of P10,172.00 was attached to
said letter.

Petitioner claimed that the insured gave false statements in his application when he limited his answer to
a consultation with a certain Dr. Reinaldo D. Raymundo of the Chinese General Hospital on February
1986, for cough and flu complications only but did not disclose that two weeks prior to his application for
insurance, the insured was examined and confined at the Lung Center of the Philippines, where he was
diagnosed for renal failure.

ISSUE :
WON there was concealment made by the insured.

RULING :
SC disagrees with the RTC's findings that while indeed there was concealment and misrepresentation,
the same was made in "good faith" and the facts concealed or misrepresented were irrelevant since the
policy was "non-medical".
Section 26 of the Insurance Code is explicit in requiring a party to a contract of insurance to
communicate to the other, in good faith, all facts within his knowledge which are material to the contract
and as to which he makes no warranty, and which the other has no means of ascertaining. The
information which the insured failed to disclose were material and relevant to the approval and the
issuance of the insurance policy. The matters concealed would have definitely affected petitioner's action
on his application, either by approving it with the corresponding adjustment for a higher premium or
rejecting the same. Moreover, a disclosure may have warranted a medical examination of the insured by
petitioner in order for it to reasonably assess the risk involved in accepting the application.
Thus, "good faith" is no defense in concealment.
The argument, that petitioner's waiver of the medical examination of the insured debunks the materiality
of the facts concealed, is untenable. The waiver of a medical examination [in a non-medical insurance
contract] renders even more material the information required of the applicant concerning previous
condition of health and diseases suffered, for such information necessarily constitutes an important
factor which the insurer takes into consideration in deciding whether to issue the policy or not.
Anent the finding that the facts concealed had no bearing to the cause of death of the insured, it is well
settled that the insured need not die of the disease he had failed to disclose to the insurer. It is sufficient
that his non-disclosure misled the insurer in forming his estimates of the risks of the proposed insurance
policy or in making inquiries.
We, therefore, rule that petitioner properly exercised its right to rescind the contract of insurance by
reason of the concealment employed by the insured.
WHEREFORE, the petition is GRANTED and the Decision of the Court of Appeals is REVERSED and
SET ASIDE.

40. THELMA VDA. DE CANILANG, petitioner, vs. HON. COURT OF APPEALS and GREAT PACIFIC
LIFE INSURANCE CORPORATION, respondents.
FACTS:
On 18 June 1982, Jaime Canilang consulted Dr. Wilfredo B. Claudio and was diagnosed as suffering
from "sinus tachycardia." The doctor prescribed the following for him: Trazepam, a tranquilizer; and Aptin,
a beta-blocker drug. Mr. Canilang consulted the same doctor again on 3 August 1982 and this time was
found to have "acute bronchitis."

On the next day, 4 August 1982, Jaime Canilang applied for a "non-medical" insurance policy with
respondent Great Pacific Life Assurance Company naming his wife, petitioner Thelma Canilang, as his
beneficiary. Jaime Canilang was issued a policy, with the face value of P19,700, effective as of 9 August
1982.

On 5 August 1983, Jaime Canilang died of "congestive heart failure," "anemia," and "chronic anemia."
Petitioner, widow and beneficiary of the insured, filed a claim with Great Pacific which the insurer denied
on 5 December 1983 upon the ground that the insured had concealed material information from it.

Petitioner then filed a complaint against Great Pacific with the Insurance Commission for recovery of the
insurance proceeds. A deposition given by Dr. Wilfredo Claudio was presented by petitioner. There Dr.
Claudio stated that he was the family physician of the deceased Jaime Canilang and that he had
previously treated him for "sinus tachycardia" and "acute bronchitis." Great Pacific for its part presented
Dr. Esperanza Quismorio, a physician and a medical underwriter working for Great Pacific. She testified
that the deceased's insurance application had been approved on the basis of his medical declaration.
She explained that as a rule, medical examinations are required only in cases where the applicant has
indicated in his application for insurance coverage that he has previously undergone medical
consultation and hospitalization.

Insurance Commissioner Armando Ansaldo ordered Great Pacific to pay P19,700.00 plus legal interest
and P2,000.00 as attorney's fees. On appeal by Great Pacific, the Court of Appeals reversed and set
aside the decision of the Insurance Commissioner. The Court of Appeals also found that the failure of
Jaime Canilang to disclose previous medical consultation and treatment constituted material information
which should have been communicated to Great Pacific to enable the latter to make proper inquiries.

ISSUE :
WON there was concealment made by the insured.

HELD:

The Supreme Court agrees with the Court of Appeals that the information which Jaime Canilang failed to
discloses was material to the ability of Great Pacific to estimate the probable risk he presented as a
subject of life insurance. Had Canilang disclosed his visits to his doctor, the diagnosis made and the
medicines prescribed by such doctor, in the insurance application, it may be reasonably assumed that
Great Pacific would have made further inquiries and would have probably refused to issue a non-
medical insurance policy or, at the very least, required a higher premium for the same coverage.

The materiality of the information withheld by Great Pacific did not depend upon the state of mind of
Jaime Canilang. A man's state of mind or subjective belief is not capable of proof in our judicial process,
except through proof of external acts or failure to act from which inferences as to his subjective belief
may be reasonably drawn. Neither does materiality depend upon the actual or physical events which
ensue. Materiality relates rather to the "probable and reasonable influence of the facts" upon the party to
whom the communication should have been made, in assessing the risk involved in making or omitting
to make further inquiries and in accepting the application for insurance.

Section 27 of the Insurance Code of 1978 is properly read as referring to "any concealment" without
regard to whether such concealment is intentional or unintentional. The net result therefore of the phrase
"whether intentional or unintentional" is precisely to leave unqualified the term "concealment". In any
case, in the case at bar, the nature of the facts not conveyed to the insurer was such that the failure to
communicate must have been intentional rather than merely inadvertent.

WHEREFORE, the Petition for Review is DENIED for lack of merit and the Decision of the Court of
Appeals dated 16 October 1989 in C.A-G.R. SP No. 08696 is hereby AFFIRMED. No pronouncement as
to costs.

41. Philamcare Health Systems, Inc. vs Court of Appeals and Julita Trinos 379 SCRA 356

Facts: Julita Trinos was the live-in wife of Ernani Trinos, who had a Health Care Agreement with petition
er company. Under coverage, Mr. Trinos suffered a heart attack, was twice confined in a hospital, then s
ubsequently died. Julita Trinos incurred expenses amounting to P76,000.
Philamcare denied the insurance claim on the grounds that a health care agreement is not an insurance
contract. That there was material concealment the insured as it would appear that in the application for h
ealth coverage, petitioners required respondent's husband to sign an express authorization for any pers
on, organization or entity that has any record or knowledge of his health to furnish any and all informatio
n relative to any hospitalization, consultation, treatment or any other medical advice or examination. Also,
it was contended that Julita Trinos was not the legal wife.

Issue(s): (1) WON the agreement was an insurance contract. (2) WON there was material concealment
of facts. (3) WON Julita Trinos is entitled to receive.

Ruling:
Yes. An insurance contract exists where the following elements concur: 1. The insured has an insurable
interest; 2. The insured is subject to a risk of loss by the happening of the designated peril; 3. The insure
r assumes the risk; 4. Such assumption of risk is part of a general scheme to distribute actual losses am
ong a large group of persons bearing a similar risk; and 5. In consideration of the insurer's promise, the i
nsured pays a premium.
No. The answer assailed by petitioner was in response to the question relating to the medical history of t
he applicant. This largely depends on opinion rather than fact, especially coming from respondent's husb
and who was not a medical doctor. Where matters of opinion or judgment are called for, answers made i
n good faith and without intent to deceive will not avoid a policy even though they are untrue. Thus, "(A)lt
hough false, a representation of the expectation, intention, belief, opinion, or judgment of the insured will
not avoid the policy if there is no actual fraud in inducing the acceptance of the risk, or its acceptance at
a lower rate of premium, and this is likewise the rule although the statement is material to the risk, if the
statement is obviously of the foregoing character, since in such case the insurer is not justified in relying
upon such statement, but is obligated to make further inquiry. There is a clear distinction between such
a case and one in which the insured is fraudulently and intentionally states to be true, as a matter of exp
ectation or belief, that which he then knows, to be actually untrue, or the impossibility of which is shown
by the facts within his knowledge, since in such case the intent to deceive the insurer is obvious and am
ounts to actual fraud."
Yes. In a contract of indemnity, payment should be made to the party who incurred the expenses.

PERSONS ENTITLED TO RECEIVE UNDER THE POLICY

42. Bonifacio Brothers, Inc. vs Mora 20 SCRA 261


Facts:
Enrique Mora mortgaged his Odlsmobile sedan car to HS Reyes Inc. with the condition that Mora would
insure the car with HS Reyes as beneficiary. The car was then insured with State Insurance Company a
nd the policy delivered to Mora. During the effectivity of the insurance contract, the car figured in an acci
dent. The company then assigned the accident to an insurance appraiserfor investigation and appraisal
of the damage. Mora without the knowledge and consent of HS Reyes, authorized Bonifacio Bros to fix t
he car, using materials supplied by the Ayala Auto Parts Company.
For the cost of Labor and materials, Mora was billed P2,102.73. The bill was sent to the insurer’s apprai
ser. The insurance company drew a check in the amount of the insurance proceeds and entrusted the c
heck to its appraiser for delivery to the proper party. The car was delivered to Mora without the consent
of HS Reyes, and without payment to Bonifacio Bros and Ayala.

Upon the theory that the insurance proceeds should be directly paid to them, Bonifacio and Ayala filed a
complaint against Mora and the insurer with the municipal court for the collection of P2,102.73. The insu
rance company filed its answer with a counterclaim for interpleader, requiring Bonifacio and HS Reyes t
o interplead in order to determine who has a better right to the proceeds.

Issue:
The main issue raised is whether there is privity of contract between the Bonifacio Bros. Inc and the Ayal
a Auto Parts Co. on the one hand and the insurance company on the other.

Ruling:
No Privity. It is fundamental that contracts take effect only between the parties thereto, except on some
specific instances provided by law where the contract contains some stipulation in favor of a third person
(Art. 1311, Civil Code). Such stipulation is known as stipulation pour autrui or a provision in favor of a thi
rd person not a party to the contract. Under this doctrine, a third person is allowed to avail himself of a b
enefit granted to him by the terms of the contract, provided that the contracting parties have clearly and
deliberately conferred a favor upon such person (Art. 1311, Civil Code; Uy Tam, et al. vs. Leonard, 30 P
hil.. 471 ). Consequently, a third person not a party to the contract has no action against the parties ther
eto, and cannot generally demand the enforcement of the same (Manila Railroad Co. vs. Compañia Tra
nsatlantica, 38 Phil. 676).
The question of whether a third person has an enforceable interest in a contract, must be settled by dete
rmining whether the contracting parties intended to tender him such an interest by deliberately inserting t
erms in their agreement with the avowed purpose of conferring a favor upon such third person. In this co
nnection, this Court has laid down the rule that the fairest test to determine whether the interest of a third
person in a contract is a stipulation pour autrui or merely an incidental interest, is to rely upon the intenti
on of the parties as disclosed by their contract ( Uy Tam, et al. vs. Leonard, supra).
A policy of insurance is a distinct and independent contract between the insured and insurer, and third p
ersons have no right either in a court of equity, or in a court of law, to the proceeds of it, unless there be
some contract of trust, expressed or implied, by the insured and third person (Lampano vs.Jose, 30 Phil.
537).

43. First Integrated Bonding & Insurance Company, Inc., petitioner, vs Hon. Harold M. Hernando, Victori
no Advincula, Romana Advicula, Silverio Blanco & The Sheriff of Manila and his Deputy Sheriffs, respon
dents.

Facts:
Silverio Blanco was the owner of a passenger jeepney which he insured against liabilities for death and i
njuries to third persons with First Integrated Bonding and Insurance Company, Inc. with the face value of
P30,000. On November 25, 1976, the said jeepney driven by Blanco himself bumped a five-year old chil
d, Deogracias Advincula, causing the latter’s death.
The child’s parents, the Advincula spouses brought a complaint for damages in the Regional Trial Court
of Abra against Silverio Blanco as well as impleading First Insurance in the complaint as insurer. On the
basis of the evidence presented by the Advincula spouses, judgment was rendered by the trial court in f
avour of the spouses. The court adjudicated First Integrated Bonding and Insurance Company liable in t
he amount of P23,663.50 which must be satisfied independently by it in favour of the spouses and the b
alance of P6,336.50 shall also be paid by said insurance company to Silverio Blanco, the grand total und
er the policy being P30,000. Herein petitioner filed a petition for relief from judgment from the order of ex
ecution and judgment with preliminary injunction, but was denied by the court. Petitioner the filed a motio
n for reconsideration of the order denying the petition for relief but the same was denied. Hence, this peti
tion for certiorari.
Issue/s:
Whether the trial court erred in holding petitioner liable in excess of the limits of liability as provided for in
the policy contract.

Whether the trial court erred in deciding for the respondent spouse(s) where there exists no cause of act
ion against herein petitioner.

Ruling:
It is the contention of the petitioner that the Advincula spouses have no cause of action against it. Furthe
r as contended, as parents of the victim, they may proceed against the driver, Blanco on the basis of the
provisions of the New Civil Code. However, they have no cause of action against First Insurance, becau
se they are not parties to the insurance contract.
It is settled that where the insurance contract provides for indemnity against liability to a third party, such
third party can directly sue the insurer. The liability of the insurer to such third person is based on contra
ct while the liability of the insured to the third party is based on tort. Such is to protect injured persons ag
ainst the insolvency of the insured who causes such injury, and to give such injured person a certain ben
eficial interest in the proceeds of the policy. It has been held that such created a contractual relation whi
ch inures to the benefit of any and every person who may be negligently injured by the named insured a
s if such injured person were specifically named in the policy.
In the event that the injured fails or refuses to include the insurer as party defendant in his claim for inde
mnity against the insured, the latter is not prevented by law to avail of the procedural rules intended to a
void multiplicity of suits. Not even a ‘no action’ clause under the policy which requires that a final judgme
nt be first obtained against the insured and that only thereafter can the person insured recover on the po
licy can prevail over the Ruled of Court provisions aimed at avoiding multiplicity of suits.
Petitioner cannot evade liability as insurer by hiding under the cloak of the insured. It liability is primary a
nd not dependent on the recovery of judgment from the insured. The insurer’s liability accrues immediat
ely upon the occurrence of the injury or event upon which the liability depends, and does not depend on
the recovery of judgment by the injured party against the insured.
However, it appears that the award of damages in favour of Blanco has no basis as it was not put up as
a claim against the insurer. However, since the decision of the trial court had become final and executor
y, it can no longer be corrected or amended.
Petition dismissed.

44. Sherman Shafer, petitioner, vs Hon. Judge, Regional Trial Court of Olongapo City, Branch 75, and M
akati Insurance Company, Inc., respondents.
Facts:
Petitioner Sherman obtained a private car policy over his Ford Laser car from Makati Insurance Compan
y, Inc., for third party liability. During the effectivity of the policy, information for reckless imprudence res
ulting in damage to property and serious physical injuries was filed against petitioner. The complaint alle
ged that Sherman recklessly drove his car which bumped a Volkswagen owned and driven by Felino Ila
no y Legaspi, thereby causing damage to the car (Volkswagen) and physical injuries were suffered by o
ne Jovencio Poblete, Sr. as a result of such accident who was on board of the said Volkswagen. The ow
ner of the damaged Volkswagen car filed a separate civil action against petitioner for damages while Jov
encio did not reserve his right to file a separate civil action for damages.
Petitioner ten filed a third party complaint against herein private respondent, Makati Insurance Company
, Inc. The court however issued an order dismissing the third party complaint on the ground that it was pr
emature, based on the premise that unless the herein petitioner (accused) is found guilty and sentenced
to pay the offended party (Poblete, Sr.) indemnity for damages, the third party complaint is without caus
e of action. The better procedure is for accused to wait for the outcome of the criminal aspect of the cas
e to determine whether or not accused, also the third party plaintiff, has a cause of action against the thir
d party defendant for the enforcement of its third party liability under the insurance contract.

Issue/s:
Whether the court a quo erred in dismissing the third party complaint of herein petitioner Sherman again
st Makati Insurance Company Inc.

Ruling:
Compulsory Motor Vehicle Liability Insurance (third party liability) is primarily intended to provide compe
nsation for the death or bodily injuries suffered by innocent third parties or passengers as a result of a n
egligent operation and use of motor vehicles. The victims and/or their defendants are assured of immedi
ate financial assistance, regardless of the financial capacity of motor vehicle owners. Where an insuranc
e policy insures directly against liability, the insurer’s liability accrues immediately upon the occurrence o
f the injury or event upon which the liability depends, and does not depend on the recovery of judgment
by the injured party against the insured. The injured for whom the contract of insurance is intended can s
ue directly the insurer. The liability of the insurance company under the Compulsory Motor Vehicle Liabili
ty insurance is for loss or damage.
The court a quo erred in dismissing petitioner’s third party complaint on the ground that petitioner had no
cause of action yet against the insurance company (third party defendant). There is no need on the part
of the insured to wait for the decision of the trial court finding him guilty of reckless imprudence. The occ
urrence of the injury to the third party immediately gave rise to the liability of the insurer under its policy.
Petition granted.

INCONTESTABLE CLAUSE

45.Emilio Tan, et al. vs Court of Appeals and Philam Life

Facts:

This is a petition for review on certiorari of the CA's decision affirming the Insurance Commission in dism
issing petitioners' complaint for the recovery of the proceeds of their late father.

Their father, Tan Lee Siong, applied for Life Insurance with respondent in the amount of 80000 in Sept.
23, 1973. It was issued on Nov. 6, 1973. Subsequently he died of Hepatoma on April 26, 1975. Respond
ent company denied payment and rescinded the policy, returning only the amount of premium paid, on t
he ground of misrepresentation and concealment. The policy in question contained an incontestability cl
ause.

Petitioner's filed a case with the Insurance Commission, but was dismissed. The dismissal was affirmed
by the CA.

Petitioners argue that the insurance law on incontestability prevents the insurer from exercising the right
to rescind after the death of the insured.

They also question the finding of concealment, saying that no evidence was presented to show that it wa
s explained in a layman's language, and that failure of the insurer to conduct medical examination, the in
surer waived whatever imperfection by ratification. They also argue that the application form for insuran
ce pertaining to the medical history were so small as to necessitate the application of the fine print rule.

Issues

Does the death of the insured preclude the insurer from rescinding the policy?
Did the insurer waive the concealment by ratification by not conducting medical examination?
Does the "fine print" or contract of adhesion rule apply in this case?

Ruling.

PETITION DENIED

No.

The so-called "incontestability clause" precludes the insurer from raising the defenses of false represent
ations or concealment of material facts insofar as health and previous diseases are concerned if the insu
rance has been in force for at least two years during the insured's lifetime. The phrase "during the lifetim
e" found in Section 48 simply means that the policy is no longer considered in force after the insured has
died. The key phrase in the second paragraph of Section 48 is "for a period of two years."

The policy was issued on November 6, 1973 and the insured died on April 26, 1975. The policy was thus
in force for a period of only one year and five months. Considering that the insured died before the two-
year period had lapsed, respondent company is not, therefore, barred from proving that the policy is voi
d ab initio by reason of the insured's fraudulent concealment or misrepresentation. Moreover, responden
t company rescinded the contract of insurance and refunded the premiums paid on September 11, 1975,
previous to the commencement of this action on November 27, 1975.

The insurer has two years from the date of issuance of the insurance contract or of its last reinstatement
within which to contest the policy, whether or not, the insured still lives within such period. After two yea
rs, the defenses of concealment or misrepresentation, no matter how patent or well founded, no longer li
e. Congress felt this was a sufficient answer to the various tactics employed by insurance companies to
avoid liability. The petitioners' interpretation would give rise to the incongruous situation where the benefi
ciaries of an insured who dies right after taking out and paying for a life insurance policy, would be allow
ed to collect on the policy even if the insured fraudulently concealed material facts

2. No

The presumption is that a person intends the ordinary consequence of his voluntary act and takes ordina
ry care of his concerns.

The evidence for respondent company shows that on September 19, 1972, the deceased was examined
by Dr. Victoriano Lim and was found to be diabetic and hypertensive; that by January, 1973, the deceas
ed was complaining of progressive weight loss and abdominal pain and was diagnosed to be suffering fr
om hepatoma.
Another physician, Dr. Wenceslao Vitug, testified that the deceased came to see him on December 14,
1973 for consultation and claimed to have been diabetic for five years.

Because of the concealment made by the deceased of his consultations and treatments for hypertension
, diabetes and liver disorders, respondent company was thus misled into accepting the risk and approvin
g his application as medically standard and dispensing with further medical investigation and examinatio
n. For as long as no adverse medical history is revealed in the application form, and applicant for insura
nce is presumed to be healthy and physically fit and no further medical investigation or examination is co
nducted by respondent company

3. Fine print rule. See Sweet Lines v Teves 1978 (Transpo) for further study.

"All provisions, conditions, or exceptions which in any way tend to work a forfeiture of the policy should b
e construed most strongly against those for whose benefit they are inserted, and most favorably toward t
hose against whom they are meant to operate."

There is no showing that the questions in the application form for insurance regarding the insured's medi
cal history are in smaller print than the rest of the printed form or that they are designed in such a way a
s to conceal from the applicant their importance.

LIABILITY UNDER OPEN POLICY

46.DEVELOPMENT INSURANCE CORP v. IAC and PHILIPPINE UNION REALTY DEVELOPMENT C


ORP.

*procedurally heavy case

Facts:

A fire occurred in the building of the private respondent and it sued for recovery of damages from the pet
itioner on the basis of an insurance contract between them. The petitioner allegedly failed to answer on t
ime and was declared in default by the trial court. A judgment of default was subsequently rendered on t
he strength of the evidence submitted ex parte by the private respondent, which was allowed full recover
y of its claimed damages. On learning of this decision, the petitioner moved to lift the order of default, inv
oking excusable neglect, and to vacate the judgment by default. Its motion was denied. It then went to th
e respondent court, which affirmed the decision of the trial court in toto.

The amount of the policy in question is for 2500000. Petitioner questions the actual amount of indemnity
based on Condition 17 of the policy making the insured as its own insurer in case the property at the tim
e of the fire be collectively of greater value than the sum insured, and shall bear a ratable proportion of t
he loss accordingly. The value of the building at the time of the fire was allegedly st 5800000. The policy
in question is an Open Policy.

Issue
Was there excusable neglect justifying the motion to lift the order of default?
What is the amount of indemnity?

Ruling

PETITION DENIED. DECISION AFFIRMED IN FULL.

Yes

It is indisputable that summons was served on it, through its senior vice-president, on June 19, 1980. On
July 14, 1980, ten days after the expiration of the original 15-day period to answer (excluding July 4), its
counsel filed an ex parte motion for an extension of five days within which to file its answer. On July 18,
1980, the last day of the requested extension — which at the time had not yet been granted — the same
counsel filed a second motion for another 5-day extension, fourteen days after the expiry of the original
period to file its answer. The trial court nevertheless gave it five days from July 14, 1980, or until July 19,
1980, within which to file its answer. But it did not. It did so only on July 26, 1980, after the expiry of the
original and extended periods, or twenty-one days after the July 5, deadline. As a consequence, the tria
l court, on motion of the private respondent filed on July 28, 1980, declared the petitioner in default. This
was done almost one month later, on August 25, 1980. Even so, the petitioner made no move at all for t
wo months thereafter. It was only on October 27, 1980, more than one month after the judgment of defa
ult was rendered by the trial court on September 26, 1980, that it filed a motion to lift the order of default
and vacate the judgment by default.

The pattern of inexcusable neglect, if not deliberate delay, is all too clear. The petitioner has slumbered
on its right and awakened too late.
While it is true that in Trajano v. Cruz, which it cites, this Court declared "that judgments by default are g
enerally looked upon with disfavor," the default judgment in that case was set aside precisely because th
ere was excusable neglect.

Besides, the petitioners in Trajano had a valid defense against the complaint filed against them, and this
justified a relaxation of the procedural rules to allow full hearing on the substantive issues raised. In the
instant case, by contrast, the petitioner must just the same fail on the merits even if the default orders w
ere to be lifted. As the respondent Court observed, "Nothing would be gained by having the order of defa
ult set aside considering the appellant has no valid defense in its favor.

2.

With regards to the condition in the policy.

There is no evidence on record that the building was worth P5,800,000.00 at the time of the loss; only th
e petitioner says so and it does not back up its self-serving estimate with any independent corroboration.
On the contrary, the building was insured at P2,500,000.00, and this must be considered, by agreement
of the insurer and the insured, the actual value of the property insured on the day the fire occurred. This
valuation becomes even more believable if it is remembered that at the time the building was burned it
was still under construction and not yet completed.

Open policy
Sec 60 of the Insurance code.

"an open policy is one in which the value of the thing insured is not agreed upon but is left to be ascertai
ned in case of loss." This means that the actual loss, as determined, will represent the total indemnity du
e the insured from the insurer except only that the total indemnity shall not exceed the face value of the
policy.
The actual loss has been ascertained in this case and, to repeat, this Court will respect such factual dete
rmination in the absence of proof that it was arrived at arbitrarily. There is no such showing. Hence, appl
ying the open policy clause as expressly agreed upon by the parties in their contract, we hold that the pri
vate respondent is entitled to the payment of indemnity under the said contract in the total amount of P5
08,867.00.

PRESCRIPTION OF ACTION

47. SUN INSURANCE OFFICE, LTD.vs. COURT OF APPEALS and EMILIO TAN [G.R. No. 89741.
March 13, 1991.]
Facts:
On August 15, 1983, herein private respondent Emilio Tan took from herein petitioner a P300,000.00
property insurance policy to cover his interest in the electrical supply store of his brother housed in a
building in Iloilo City. Four (4) days after the issuance of the policy, the building was burned including the
insured store. On August 20, 1983, Tan filed his claim for fire loss with petitioner, but on February 29,
1984, petitioner wrote Tan denying the latter's claim. On April 3, 1984, Tan wrote petitioner, seeking
reconsideration of the denial of his claim. On September 3, 1985, Tan's counsel wrote the Insurer
inquiring about the status of his April 3, 1984 request for reconsideration. Petitioner answered the letter
on October 11, 1985, advising Tan's counsel that the Insurer's denial of Tan's claim remained unchanged.

Issue:
Whether or not the filing of a motion for reconsideration interrupts the twelve (12) months prescriptive
period to contest the denial of the insurance claim.

Ruling:
No. The filing of a motion for reconsideration does not interrupt the twelve (12) months prescriptive
period to contest the denial of the insurance claim.
The insured was definitely advised of the rejection of his claim through the letter of petitioner dated
February 29, 1984 of the denial of Tan's claim which was clearly manifested in said letter, the pertinent
portion of which reads:
"We refer to your claim for fire loss of 20th August, 1983 at Huervana St., La Paz, Iloilo City.
"We now have the report of our adjusters and after a thorough and careful review of the same and the
accompanying documents at hand, we are rejecting, much to our regret, liability for the claim under our
policies for one or more of the following reasons:

1.. . .
2.. . .
"For your information, we have referred all these matters to our lawyers for their opinion as to the
compensability of your claim, particularly referring to the above violations. It is their opinion and in fact
their strong recommendation to us to deny your claim. By this letter, we do not intend to waive or
relinquish any of our rights or defenses under our policies of insurance."
Condition 27 of the Insurance Policy, which is the subject of the conflicting contentions of the parties,
reads:
"27.Action or suit clause — If a claim be made and rejected and an action or suit be not commenced
either in the Insurance Commission or in any court of competent jurisdiction within twelve (12) months
from receipt of notice of such rejection, or in case of arbitration taking place as provided herein, within
twelve (12) months after due notice of the award made by the arbitrator or arbitrators or umpire, then the
claim shall for all purposes be deemed to have been abandoned and shall not thereafter be recoverable
hereunder

In enunciating the above-cited principle, this Court had definitely settled the rationale for the necessity of
bringing suits against the Insurer within one year from the rejection of the claim. The contention of the
respondents that the one-year prescriptive period does not start to run until the petition for
reconsideration had been resolved by the insurer, runs counter to the declared purpose for requiring that
an action or suit be filed in the Insurance Commission or in a court of competent jurisdiction from the
denial of the claim. To uphold respondents' contention would contradict and defeat the very principle
which this Court had laid down. Moreover, it can easily be used by insured persons as a scheme or
device to waste time until any evidence which may be considered against them is destroyed.

48. JACQUELINE JIMENEZ VDA. DE GABRIEL vs. HON. COURT OF APPEALS and FORTUNE
INSURANCE & SURETY COMPANY, INC. [G.R. No. 103883. November 14, 1996.]
Facts:
Marcelino Gabriel, the insured, was employed by Emerald Construction & Development Corporation
("ECDC") at its construction project in Iraq. He was covered by a personal accident insurance in the
amount of P100,000.00 under a group policy 2 procured from private respondent by ECDC for its
overseas workers. The insured risk was for "(b)odily injury caused by violent accidental external and
visible means which injury (would) solely and independently of any other cause" 3 result in death or
disability.
On 22 May 1982, within the life of the policy, Gabriel died in Iraq. A year later, or on 12 July 1983, ECDC
reported Gabriel's death to private respondent by telephone.4 Among the documents thereafter
submitted to private respondent were a copy of the death certificate 5 issued by the Ministry of Health of
the Republic of Iraq — which stated
"REASON OF DEATH: UNDER EXAMINATION NOW — NOT YET KNOWN "6
and an autopsy report 7 of the NBI to the effect that "(d)ue to advanced state of postmortem
decomposition, cause of death (could) not be determined." 8
Following a series of communications between petitioner and private respondent, the latter, on 22
September 1983, ultimately denied the claim of ECDC on the ground of prescription. 9 Petitioner went to
the Regional Trial Court of Manila. In her complaint against ECDC and private respondent, she averred
that her husband died of electrocution while in the performance of his work

Issue:
Whether or not the petitioner timely filed her notice of claim within the prescriptive period.

Ruling:
NO. The petitioner did not timely file her claim on the insurance proceeds.
Private respondent correctly invoked Section 384 of the Insurance Code; viz:
"Sec. 384.Any person having any claim upon the policy issued pursuant to this chapter shall, without any
unnecessary delay, present to the insurance company concerned a written notice of claim setting forth
the nature, extent and duration of the injuries sustained as certified by a duly licensed physician. Notice
of claim must be filed within six months from date of the accident, otherwise, the claim shall be deemed
waived. Action or suit for recovery of damage due to loss or injury must be brought, in proper cases, with
the Commissioner or the Courts within one year from denial of the claim, otherwise, the claimant's right
of action shall prescribe."
The notice of death was given to private respondent, concededly, more than a year after the death of
petitioner's husband. Private respondent, in invoking prescription, was not referring to the one-year
period from the denial of the claim within which to file an action against an insurer but obviously to the
written notice of claim that had to be submitted within six months from the time of the accident.

Premium Payments

49) Malayan Insurance Co. v. Cruz Arnaldo, 154 SCRA 672


Facts: On June 7, 1981, the Malayan Insurance (MICO) issued to, Coronacion Pinca, Fire Insurance
Policy on her property for the amount of P14,000 effective July 22, 1981, until July 22, 1982.

On October 15, 1981, MICO allegedly cancelled the policy for non-payment, of the premium and sent
the corresponding notice to Pinca. Payment of the premium for Pinca was received by Domingo Adora,
agent of MICO. On January 15, 1982, Adora remitted this payment to MICO, together with other
payments. On January 18, 1982, Pinca's property was completely burned. On February 5, 1982,
Pinca's payment was returned by MICO to Adora on the ground that her policy had been cancelled
earlier. But Adora refused to accept it.

In due time, Pinca made the requisite demands for payment, which MICO rejected. She then went to the
Insurance Commission. It is because she was ultimately sustained by the Arnaldo (Insurance
Commissioner) that MICO has come to us for relief.

Issue: WON, there was a valid cancellation of the policy.

Held: No, there was none.

On procedural aspect:
The appeal filed by MICO was beyond the reglementary period as between the two dates, the court
chooses to believe June 13, 1982, not only because the numbers "6-13-82" appear on both annexes but
also because it is the date authenticated by the administrative division of the Insurance Commission.
Annex "B" is at worst self-serving; at best, it might only indicate that it was received on June 18, 1982,
by the legal department of MICO, after it had been received earlier by some other of its personnel on
June 13, 1982. Whatever the reason for the delay in transmitting it to the legal department need not
detain us here.

Under Section 416 of the Insurance Code, the period for appeal is thirty days from notice of the decision
of the Insurance Commission. The MICO filed its motion for reconsideration on April 25, 1981, or fifteen
days such notice, and the reglementary period began to run again after June 13, 1981, date of its receipt
of notice of the denial of the said motion for reconsideration. As the herein petition was filed on July 2,
1981, or nineteen days later, there is no question that it is tardy by four days.

Counted from June 13, the fifteen-day period prescribed under Rule 45, assuming it is applicable, would
end on June 28, 1982, or also four days from July 2, when the petition was filed.

If it was filed under B.P. 129, then, considering that the motion for reconsideration was filed on the
fifteenth day after MICO received notice of the decision, only one more day would have remained for it
to appeal, to wit, June 14, 1982. That would make the petition eighteen days late by July 2.

Indeed, even if the applicable law were still R.A. 5434, governing appeals from administrative bodies,
the petition would still be tardy. The law provides for a fixed period of ten days from notice of the denial
of a seasonable motion for reconsideration within which to appeal from the decision. Accordingly, that
ten-day period, counted from June 13, 1982, would have ended on June 23, 1982, making the petition
filed on July 2, 1982, nine days late.

Whichever law is applicable, therefore, the petition can and should be dismissed for late filing.

On substantial aspect:
MICO's arguments that there was no payment of premium and that the policy had been cancelled before
the occurence of the loss are not acceptable. Its contention that the claim was allowed without proof of
loss is also untenable.

The petitioner relies heavily on Section 77 of the Insurance Code providing that:
SEC. 77. An insurer is entitled to payment of the premium as soon as the thing is exposed to the peril
insured against. Notwithstanding any agreement to the contrary, no policy or contract of insurance
issued by an insurance company is valid and binding unless and until the premium thereof has been
paid, except in the case of a life or an industrial life policy whenever the grace period provision applies.

The above provision is not applicable because payment of the premium was in fact eventually made in
this case. The payment was made on December 24, 1981, and the fire occured on January 18, 1982.

One wonders: suppose the payment had been made and accepted in, say, August 1981, would the
commencement date of the policy have been changed to the date of the payment, or would the payment
have retroacted to July 22, 1981? If MICO accepted the payment in December 1981 and the insured
property had not been burned, would that policy not have expired just the same on July 22, 1982,
pursuant to its original terms, and not on December 24, 1982?

It would seem from MICO's own theory that the policy would have become effective only upon payment,
if accepted and so would have been valid only from December 24, 1981 but only up to July 22, 1981,
according to the original terms. In others words, the policy would have run for only eight months
although the premium paid was for one whole year.

MICO's acknowledgment of Adora as its agent defeats its contention that he was not authorized to
receive the premium payment on its behalf. It is clearly provided in Section 306 of the Insurance Code
that:
SEC. 306. xxx xxx xxx
Any insurance company which delivers to an insurance agent or insurance broker a policy or contract of
insurance shall be deemed to have authorized such agent or broker to receive on its behalf payment of
any premium which is due on such policy or contract of insurance at the time of its issuance or delivery
or which becomes due thereon.

And it is a well-known principle under the law of agency that:


Payment to an agent having authority to receive or collect payment is equivalent to payment to the
principal himself; such payment is complete when the money delivered is into the agent's hands and is a
discharge of the indebtedness owing to the principal.

Payment was in fact made, rendering the policy operative as of June 22, 1981, and removing it from the
provisions of Article 77, Thereafter, the policy could be cancelled on any of the supervening grounds
enumerated in Article 64 (except "nonpayment of premium") provided the cancellation was made in
accordance therewith and with Article 65.

Section 64 reads as follows:


No policy of insurance other than life shall be cancelled by the insurer except upon prior notice thereof to
the insured, and no notice of cancellation shall be effective unless it is based on the occurrence, after
the effective date of the policy, of one or more of the following:
(a) non-payment of premium;
(b) conviction of a crime arising out of acts increasing the hazard insured against;
(c) discovery of fraud or material misrepresentation;
(d) discovery of willful, or reckless acts or commissions increasing the hazard insured against;
(e) physical changes in the property insured which result in the property becoming uninsurable;or
(f) a determination by the Commissioner that the continuation of the policy would violate or would place
the insurer in violation of this Code.

As for the method of cancellation, Section 65 provides as follows:


All notices of cancellation mentioned in the preceding section shall be in writing, mailed or delivered to
the named insured at the address shown in the policy, and shall state (a) which of the grounds set forth
in section sixty-four is relied upon and (b) that, upon written request of the named insured, the insurer
will furnish the facts on which the cancellation is based.
A valid cancellation must, therefore, require concurrence of the following conditions:
(1) There must be prior notice of cancellation to the insured;
(2) The notice must be based on the occurrence, after the effective date of the policy, of one or more of
the grounds mentioned;
(3) The notice must be (a) in writing, (b) mailed, or delivered to the named insured, (c) at the address
shown in the policy;
(4) It must state (a) which of the grounds mentioned in Section 64 is relied upon and (b) that upon
written request of the insured, the insurer will furnish the facts on which the cancellation is based.

MICO's claims it cancelled the policy in question on October 15, 1981, for non-payment of premium. To
support this assertion, it presented one of its employees, who testified that "the original of the
endorsement and credit memo" — presumably meaning the alleged cancellation — "were sent the
assured by mail through our mailing section" However, there is no proof that the notice, assuming it
complied with the other requisites mentioned above, was actually mailed to and received by Pinca.

On the other hand, there is the flat denial of Pinca, who says she never received the claimed
cancellation and who, of course, did not have to prove such denial.

50) Makati Tuscany Condominium Corp. v. Court of Appeals, 215 SCRA 462
Facts: Sometime in early 1982, American Home Assurance Co. (AHAC), represented by American
International Underwriters (Phils.), Inc., issued in favor of Makati Tuscany Condominium Corporation
(TUSCANY) Insurance Policy No. AH-CPP-9210452 on the latter's building and premises, for a period
beginning 1 March 1982 and ending 1 March 1983, with a total premium of P466,103.05. The premium
was paid on installments on 12 March 1982, 20 May 1982, 21 June 1982 and 16 November 1982, all of
which were accepted by private respondent.

On 10 February 1983, AHAC issued to Tuscany Insurance Policy No. AH-CPP-9210596, which replaced
and renewed the previous policy, for a term covering 1 March 1983 to 1 March 1984. The premium in the
amount of P466,103.05 was again paid on installments on 13 April 1983, 13 July 1983, 3 August 1983, 9
September 1983, and 21 November 1983. All payments were likewise accepted by AHAC.

On 20 January 1984, the policy was again renewed and AHAC issued to Tuscany Insurance Policy No.
AH-CPP-9210651 for the period 1 March 1984 to 1 March 1985. On this renewed policy, Tuscany made
two installment payments, both accepted by AHAC, the first on 6 February 1984 for P52,000.00 and the
second, on 6 June 1984 for P100,000.00. Thereafter, Tuscany refused to pay the balance of the
premium.

Consequently, AHAC filed an action to recover the unpaid balance of P314,103.05 for Insurance Policy
No. AH-CPP-9210651.

In its answer with counterclaim, Tuscany admitted the issuance of Insurance Policy No. AH-CPP-
9210651. It explained that it discontinued the payment of premiums because the policy did not contain a
credit clause in its favor and the receipts for the installment payments covering the policy for 1984-85, as
well as the two (2) previous policies, stated the following reservations:
2. Acceptance of this payment shall not waive any of the company rights to deny liability on any claim
under the policy arising before such payments or after the expiration of the credit clause of the policy;
and
3. Subject to no loss prior to premium payment. If there be any loss such is not covered.

Tuscany further claimed that the policy was never binding and valid, and no risk attached to the policy. It
then pleaded a counterclaim for P152,000.00 for the premiums already paid for 1984-85, and in its
answer with amended counterclaim, sought the refund of P924,206.10 representing the premium
payments for 1982-85.

On 8 October 1987, the trial court dismissed the complaint and the counterclaim upon the following
findings:
While it is true that the receipts issued to the defendant contained the aforementioned reservations, it is
equally true that payment of the premiums of the three aforementioned policies (being sought to be
refunded) were made during the lifetime or term of said policies, hence, it could not be said, inspite of
the reservations, that no risk attached under the policies. Consequently, defendant's counterclaim for
refund is not justified.

As regards the unpaid premiums on Insurance Policy No. AH-CPP-9210651, in view of the reservation in
the receipts ordinarily issued by the AHAC on premium payments the only plausible conclusion is that
AHAC has no right to demand their payment after the lapse of the term of said policy on March 1, 1985.
Therefore, the Tuscany was justified in refusing to pay the same.

The CA modified the decision and held Tuscany to pay the balance of the premiums plus legal interest
and affirmed the denial of the counterclaim. The CA rationcinated that the obligation to pay premiums
when due is ordinarily as indivisible obligation to pay the entire premium. Here, the parties herein agreed
to make the premiums payable in installments, and there is no pretense that the parties never
envisioned to make the insurance contract binding between them. It was renewed for two succeeding
years, the second and third policies being a renewal/replacement for the previous one. And the insured
never informed the insurer that it was terminating the policy because the terms were unacceptable.
Issue: WON, the payment by installment of the premiums due on an insurance policy invalidates the
contract of insurance, in view of Sec. 77 of P.D. 612, otherwise known as the Insurance Code, as
amended.

Held/Ratio: No, while it may be true that under Section 77 of the Insurance Code(An insurer is entitled to
the payment of the premium as soon as the thing is exposed to the peril insured against.
Notwithstanding any agreement to the contrary, no policy or contract of insurance issued by an
insurance company is valid and binding unless and until the premium thereof has been paid, except in
the case of a life or an industrial life policy whenever the grace period provision applies.), the parties
may not agree to make the insurance contract valid and binding without payment of premiums, there is
nothing in said section which suggests that the parties may not agree to allow payment of the premiums
in installment, or to consider the contract as valid and binding upon payment of the first premium.
Otherwise, we would allow the insurer to renege on its liability under the contract, had a loss incurred
(sic) before completion of payment of the entire premium, despite its voluntary acceptance of partial
payments, a result eschewed by a basic considerations of fairness and equity.
To our mind, the insurance contract became valid and binding upon payment of the first premium, and
the plaintiff could not have denied liability on the ground that payment was not made in full, for the
reason that it agreed to accept installment payment. . . .

Tuscany now asserts that its payment by installment of the premiums for the insurance policies for 1982,
1983 and 1984 invalidated said policies because of the provisions of Sec. 77 of the Insurance Code, as
amended, and by the conditions stipulated by the insurer in its receipts, disclaiming liability for loss for
occurring before payment of premiums.

It argues that where the premiums is not actually paid in full, the policy would only be effective if there is
an acknowledgment in the policy of the receipt of premium pursuant to Sec. 78 of the Insurance Code.
The absence of an express acknowledgment in the policies of such receipt of the corresponding
premium payments, and petitioner's failure to pay said premiums on or before the effective dates of said
policies rendered them invalid. Tuscany thus concludes that there cannot be a perfected contract of
insurance upon mere partial payment of the premiums because under Sec. 77 of the Insurance Code,
no contract of insurance is valid and binding unless the premium thereof has been paid, notwithstanding
any agreement to the contrary. As a consequence, petitioner seeks a refund of all premium payments
made on the alleged invalid insurance policies.

We hold that the subject policies are valid even if the premiums were paid on installments. The records
clearly show that petitioner and private respondent intended subject insurance policies to be binding and
effective notwithstanding the staggered payment of the premiums. The initial insurance contract entered
into in 1982 was renewed in 1983, then in 1984. In those three (3) years, the insurer accepted all the
installment payments. Such acceptance of payments speaks loudly of the insurer's intention to honor the
policies it issued to petitioner. Certainly, basic principles of equity and fairness would not allow the
insurer to continue collecting and accepting the premiums, although paid on installments, and later deny
liability on the lame excuse that the premiums were not prepared in full.

We therefore sustain the Court of Appeals. We quote with approval the well-reasoned findings and
conclusion of the appellate court contained in its Resolution denying the motion to reconsider its
Decision —
While the import of Section 77 is that prepayment of premiums is strictly required as a condition to the
validity of the contract, We are not prepared to rule that the request to make installment payments duly
approved by the insurer, would prevent the entire contract of insurance from going into effect despite
payment and acceptance of the initial premium or first installment. Section 78 of the Insurance Code in
effect allows waiver by the insurer of the condition of prepayment by making an acknowledgment in the
insurance policy of receipt of premium as conclusive evidence of payment so far as to make the policy
binding despite the fact that premium is actually unpaid. Section 77 merely precludes the parties from
stipulating that the policy is valid even if premiums are not paid, but does not expressly prohibit an
agreement granting credit extension, and such an agreement is not contrary to morals, good customs,
public order or public policy (De Leon, the Insurance Code, at p. 175). So is an understanding to allow
insured to pay premiums in installments not so proscribed. At the very least, both parties should be
deemed in estoppel to question the arrangement they have voluntarily accepted.
The reliance by petitioner on Arce vs. Capital Surety and Insurance Co. is unavailing because the facts
therein are substantially different from those in the case at bar. In Arce, no payment was made by the
insured at all despite the grace period given. In the case before Us, petitioner paid the initial installment
and thereafter made staggered payments resulting in full payment of the 1982 and 1983 insurance
policies. For the 1984 policy, petitioner paid two (2) installments although it refused to pay the balance.

It appearing from the peculiar circumstances that the parties actually intended to make three (3)
insurance contracts valid, effective and binding, Tuscany may not be allowed to renege on its obligation
to pay the balance of the premium after the expiration of the whole term of the third policy (No. AH-CPP-
9210651) in March 1985. Moreover, as correctly observed by the appellate court, where the risk is entire
and the contract is indivisible, the insured is not entitled to a refund of the premiums paid if the insurer
was exposed to the risk insured for any period, however brief or momentary.

51) South Sea Surety and Insurance Co. v. Court of Appeals, 244 SCRA 744
Facts: It appears that on 16 January 1984, plaintiff [Valenzuela Hardwood and Industrial Supply, Inc.]
entered into an agreement with the defendant Seven Brothers whereby the latter undertook to load on
board its vessel M/V Seven Ambassador the former's lauan round logs numbering 940 at the port of
Maconacon, Isabela for shipment to Manila.

On 20 January 1984, Valenzuela insured the logs, against loss and/or, damage with defendant South
Sea Surety and Insurance Co., Inc. for P2,000,000.00 and the latter issued its Marine Cargo Insurance
Policy No. 84/24229 for P2,000,000.00 on said date.

On 24 January 1984, the Valenzuela Hardwood gave the check in payment of the premium on the
insurance policy to Mr. Victorio Chua.

On 25 January 1984, M/V Seven Ambassador sank that resulted in the loss of Valenzuela Hardwood’s
insured logs.

On 30 January 1984, a check for P5,625.00 (Exh. "E") to cover payment of the premium and
documentary stamps due on the policy was tendered to the insurer but was not accepted. Instead, the
South Sea Surety and Insurance Co., Inc. cancelled the insurance policy it issued as of the date of
inception for non-payment of the premium due in accordance with Section 77 of the Insurance Code.

On 2 February 1984, Valenzuela Hardwood demanded from defendant South Sea Surety and Insurance
Co., Inc. the payment of the proceeds of the policy but the latter denied liability under the policy.
Valenzuela likewise filed a formal claim with defendant Seven Brothers Shipping Corporation for the
value of the lost logs but the latter denied the claim.

The RTC ruled in favor of Valenzuela Hardwood.

The CA affirmed but only as against the insurance corporation (Southsea Surety). It absolved the
shipping entity because of the stipulation in the charter party that the ship owner (Seven Brothers) would
be exempted from liability in case of loss. Hence, this petition.

Issue: WON, Southsea should also be absolved based on non-payment of premiums as Victorio Chua
acted not its agent. (In other words, was Victorio Chua its agent? Because he delivered the check for the
payment of the premium to Southsea only AFTER the loss occurred.)

Held/Ratio: Southsea should not be absolved as Chua was its agent. Section 77 of the Insurance Code
provides:
Sec. 77. An insurer is entitled to payment of the premium as soon as the thing insured is exposed to the
peril insured against. Notwithstanding any agreement to the contrary, no policy or contract of insurance
issued by an insurance company is valid and binding unless and until the premium thereof has been
paid, except in the case of a life or an industrial life policy whenever the grace period provision applies.
Undoubtedly, the payment of the premium is a condition precedent to, and essential for, the
efficaciousness of the contract. The only two statutorily provided exceptions are (a) in case the
insurance coverage relates to life or industrial life (health) insurance when a grace period applies and (b)
when the insurer makes a written acknowledgment of the receipt of premium, this acknowledgment
being declared by law to be then conclusive evidence of the premium payment (Secs. 77-78, Insurance
Code). The appellate court, contrary to what the petition suggests, did not make any pronouncement to
the contrary. Indeed, it has said:
Concerning the issue as to whether there is a valid contract of insurance between plaintiff-appellee and
defendant-appellant South Sea Surety and Insurance Co., Inc., Section 77 of the Insurance Code
explicitly provides that notwithstanding any agreement to the contrary, no policy issued by an insurance
company is valid and binding unless and until premium thereof has been paid. It is therefore important to
determine whether at the time of the loss, the premium was already paid.

No attempt becloud the issues can disguise the fact that the sole question raised in the instant petition is
really evidentiary in nature, i.e., whether or not Victorio Chua, in receiving the check for the insurance
premium prior to the occurrence of the risk insured against has so acted as an agent of petitioner. The
appellate court, like the trial court, has found in the affirmative. Said the appellate court:
In the instant case, the Marine Cargo Insurance Policy No. 84/24229 was issued by defendant insurance
company on 20 January 1984. At the time the vessel sank on 25 January 1984 resulting in the loss of
the insured logs, the insured had already delivered to Victorio Chua the check in payment of premium.
But, as Victorio Chua testified, it was only in the morning of 30 January 1984 or 5 days after the vessel
sank when his messenger tendered the check to defendant South Sea Surety and Insurance Co., Inc.
(TSN, pp. 3-27, 16-17, 22 October 1985).

Appellant surety company insists that Mr. Chua is an administrative assistant for the past ten years and
an agent for less than ten years of the Columbia Insurance Brokers, Ltd. He is paid a salary as a
administrative assistant and a commission as agent based on the premiums he turns over to the broker.
Southsea therefore argues that Mr. Chua, having received the insurance premiums as an agent of the
Columbia Insurance Broker, acted as an agent of the insured under Section 301 of the Insurance Code
which provides as follows:
Sec. 301. Any person who for any compensation, commission or other thing of value, acts, or aids in
soliciting, negotiating or procuring the making of any insurance contract or in placing risk or taking out
insurance, on behalf of an insured other than himself, shall be an insurance broker within the intent of
this Code, and shall thereby become liable to all the duties requirements, liabilities and penalties to
which an insurance broker is subject.

Valenzuela Hardwood, upon the other hand, claim that the second paragraph of Section 306 of the
Insurance Code provide as follows:
Sec. 306. . . . Any insurance company which delivers to an insurance agent or insurance broker a policy
or contract of insurance shall be deemed to have authorized such agent or broker to receive on its
behalf payment of any premium which is due on such policy of contract of insurance at the time of its
issuance or delivery or which becomes due thereon.

On cross-examination in behalf of South Sea Surety and Insurance Co., Inc. Mr. Chua testified that the
marine cargo insurance policy for the plaintiff's logs was delivered to him on 21 January 1984 at his
office to be delivered to the plaintiff.

When the appellant South Sea Surety and Insurance Co., Inc. delivered to Mr. Chua the marine cargo
insurance policy for the plaintiffs logs, he is deemed to have been authorized by the South Sea Surety
and Insurance Co., Inc. to receive the premium which is due on its behalf.

When therefore the insured logs were lost, the insured had already paid the premium to an agent of the
South Sea Surety and Insurance Co., Inc., which is consequently liable to pay the insurance proceeds
under the policy it issued to the insured.

52) SPS. TIBAY v. COURT OF APPEALS and FORTUNE LIFE AND GENERAL INSURANCE CO., INC.
- Abril

Facts: Jan 22 1987, FORTUNE issued a Fire Insurance Policy in favor of Sps. Tibay on their 2-storey
residential bldg. located in Makati city, together with all their personal effects therein. The insurance was
for P600,000 covering the period fromJan 23 1987 to Jan 23 1988. On Jan 23 1987, of the total
premium of P2983.50, Violeta Tibay only paid P600 thus leaving a considerable balance unpaid.
Mar 8 1987, the insured bldg. was completely destroyed by fire. 2 days later (mar 10 1987) Violeta oaid
the balance of the premium. On the same day, she filed with Fortune a claim on the fire insurance policy.
The claim was referred to GASI which immediately wrote Violeta requesting her to furnish it with the
necessary documents for the investigation and processing of her claim. Complied, she signed (mar 28
1987) a non-waiver agreement with GASI to the effect that any action taken by the companies or their
representatives in investigating the claim made by the claimant for his loss which occurred at 5855
Zobel Roxas, Makati on March 8, 1987, or in the investigating or ascertainment of the amount of actual
cash value and loss, shall not waive or invalidate any condition of the policies of such companies held
by said claimant, nor the rights of either or any of the parties to this agreement, and such action shall not
be, or be claimed to be, an admission of liability on the part of said companies or any of them.
Fortune denied the claim for violation of Policy condition no. 2 and sec. 77 of the Insurance Code. Efforts
to settle the case before the Commission proved futile. Mar 3 1988, Violeta and the other petitioners
sued Fortune in the amount of P600,000 plus 12% interest per annum, P100,000 moral damages, and
atty’s fees equivalent to 20% of the total claim.
TC adjudged Fortune liable for the total value of the insured bldg. and personal properties (P600,000).
CA reversed the decision declaring Fortune not liable to the sps. But to return to the sps the premium
plus 12% interest from Mar 10 1987 until full payment.

Issue: Whether or not Fortune remains liable under the fire insurance policy despite the spouses’ failure
to pay their premium in full.

Ruling: No.
Insurance is a contract whereby one undertakes for a consideration to indemnify another against loss,
damage or liability arising from an unknown or contingent event. The consideration is the premium,
which must be paid at the time and in the way and manner specified in the policy, and if not so paid, the
policy will lapse and be forfeited by its own terms.
The pertinent provisions in the Policy on premium read:
THIS POLICY OF INSURANCE WITNESSETH, THAT only after payment to the Company in
accordance with Policy Condition No. 2 of the total premiums by the insured as stipulated above for the
period aforementioned for insuring against Loss or Damage by Fire or Lightning as herein appears, the
Property herein described
xxx

2. This policy including any renewal thereof and/or any endorsement thereon is not in force until the
premium has been fully paid to and duly receipted by the Company in the manner provided herein.
Any supplementary agreement seeking to amend this condition prepared by agent, broker or Company
official, shall be deemed invalid and of no effect.
xxx
Except only in those specific cases where corresponding rules and regulations which are or may
hereafter be in force provide for the payment of the stipulated premiums in periodic installments at fixed
percentage, it is hereby declared, agreed and warranted that this policy shall be deemed effective, valid
and binding upon the Company only when the premiums therefor have actually been paid in full and duly
acknowledged in a receipt signed by any authorized official or representative/agent of the Company in
such manner as provided herein, (Italics supplied)

Clearly the Policy provides for payment of premium in full. Accordingly, where the premium has only
been partially paid and the balance paid only after the peril insured against has occurred, the insurance
contract did not take effect and the insured cannot collect at all on the policy. This is fully supported by
Sec. 77 of the Insurance Code which provides
SEC. 77. An insurer is entitled to payment of the premium as soon as the thing insured is exposed to the
peril insured against. Notwithstanding any agreement to the contrary, no policy or contract of insurance
issued by an insurance company is valid and binding unless and until the premium thereof has been
paid, except in the case of a life or an industrial life policy whenever the grace period provision applies
(Italics supplied).

Apparently the crux of the controversy lies in the phrase unless and until the premium thereof has been
paid. This leads us to the manner of payment envisioned by the law to make the insurance policy
operative and binding. For whatever judicial construction may be accorded the disputed phrase must
ultimately yield to the clear mandate of the law. The principle that where the law does not distinguish the
court should neither distinguish assumes that the legislature made no qualification on the use of a
general word or expression. In Escosura v. San Miguel Brewery, inc.,] the Court through Mr. Justice
Jesus G. Barrera, interpreting the phrase with pay used in connection with leaves of absence with pay
granted to employees, ruled -
x x x the legislative practice seems to be that when the intention is to distinguish between full and partial
payment, the modifying term is used x x x
Citing C. A. No. 647 governing maternity leaves of married women in government, R. A. No. 679
regulating employment of women and children, R.A. No. 843 granting vacation and sick leaves to judges
of municipal courts and justices of the peace, and finally, Art. 1695 of the New Civil Code providing that
every househelp shall be allowed four (4) days vacation each month, which laws simply stated with pay,
the Court concluded that it was undisputed that in all these laws the phrase with pay used without any
qualifying adjective meant that the employee was entitled to full compensation during his leave of
absence.

Petitioners maintain otherwise. Insisting that FORTUNE is liable on the policy despite partial payment of
the premium due and the express stipulation thereof to the contrary, petitioners rely heavily on the 1967
case of Philippine Phoenix and Insurance Co., Inc. v. Woodworks, Inc. where the Court through Mr.
Justice Arsenio P. Dizon sustained the ruling of the trial court that partial payment of the premium made
the policy effective during the whole period of the policy. In that case, the insurance company
commenced action against the insured for the unpaid balance on a fire insurance policy. In its defense
the insured claimed that nonpayment of premium produced the cancellation of the insurance contract.
Ruling otherwise the Court held
It is clear x x x that on April 1, 1960, Fire Insurance Policy No. 9652 was issued by appellee and
delivered to appellant, and that on September 22 of the same year, the latter paid to the former the sum
of P3,000.00 on account of the total premium of P6,051.95 due thereon. There is, consequently, no
doubt at all that, as between the insurer and the insured, there was not only a perfected contract of
insurance but a partially performed one as far as the payment of the agreed premium was concerned.
Thereafter the obligation of the insurer to pay the insured the amount, for which the policy was issued in
case the conditions therefor had been complied with, arose and became binding upon it, while the
obligation of the insured to pay the remainder of the total amount of the premium due became
demandable.

The 1967 Phoenix case is not persuasive; neither is it decisive of the instant dispute. For one, the
factual scenario is different. In Phoenix it was the insurance company that sued for the balance of the
premium, i.e., it recognized and admitted the existence of an insurance contract with the insured. In the
case before us, there is, quite unlike in Phoenix, a specific stipulation that (t)his policy xxx is not in force
until the premium has been fully paid and duly receipted by the Company x x x. Resultantly, it is correct
to say that in Phoenix a contract was perfected upon partial payment of the premium since the parties
had not otherwise stipulated that prepayment of the premium in full was a condition precedent to the
existence of a contract.
In Phoenix, by accepting the initial payment of P3,000.00 and then later demanding the remainder of the
premium without any other precondition to its enforceability as in the instant case, the insurer in effect
had shown its intention to continue with the existing contract of insurance, as in fact it was enforcing its
right to collect premium, or exact specific performance from the insured. This is not so here. By express
agreement of the parties, no vinculum juris or bond of law was to be established until full payment was
effected prior to the occurrence of the risk insured against.
In Makati Tuscany Condominium Corp. v. Court of Appeals the parties mutually agreed that the
premiums could be paid in installments, which in fact they did for three (3) years, hence, this Court
refused to invalidate the insurance policy. In giving effect to the policy, the Court quoted with approval
the Court of Appeals
The obligation to pay premiums when due is ordinarily an indivisible obligation to pay the entire premium.
Here, the parties x x x agreed to make the premiums payable in installments, and there is no pretense
that the parties never envisioned to make the insurance contract binding between them. It was renewed
for two succeeding years, the second and third policies being a renewal/replacement for the previous
one. And the insured never informed the insurer that it was terminating the policy because the terms
were unacceptable.

While it maybe true that under Section 77 of the Insurance Code, the parties may not agree to make the
insurance contract valid and binding without payment of premiums, there is nothing in said section which
suggests that the parties may not agree to allow payment of the premiums in installment, or to consider
the contract as valid and binding upon payment of the first premium. Otherwise we would allow the
insurer to renege on its liability under the contract, had a loss incurred (sic) before completion of
payment of the entire premium, despite its voluntary acceptance of partial payments, a result eschewed
by basic considerations of fairness and equity x x x.

These two (2) cases, Phoenix and Tuscany, adequately demonstrate the waiver, either express or
implied, of prepayment in full by the insurer: impliedly, by suing for the balance of the premium as
inPhoenix, and expressly, by agreeing to make premiums payable in installments as in Tuscany. But
contrary to the stance taken by petitioners, there is no waiver express or implied in the case at bench.
Precisely, the insurer and the insured expressly stipulated that (t)his policy including any renewal thereof
and/or any indorsement thereon is not in force until the premium has been fully paid to and duly
receipted by the Company x x x and that this policy shall be deemed effective, valid and binding upon
the Company only when the premiums therefor have actually been paid in full and duly acknowledged.
Conformably with the aforesaid stipulations explicitly worded and taken in conjunction with Sec. 77 of
the Insurance Code the payment of partial premium by the assured in this particular instance should not
be considered the payment required by the law and the stipulation of the parties. Rather, it must be
taken in the concept of a deposit to be held in trust by the insurer until such time that the full amount has
been tendered and duly receipted for. In other words, as expressly agreed upon in the contract, full
payment must be made before the risk occurs for the policy to be considered effective and in force.
Thus, no vinculum juris whereby the insurer bound itself to indemnify the assured according to law ever
resulted from the fractional payment of premium. The insurance contract itself expressly provided that
the policy would be effective only when the premium was paid in full. It would have been altogether
different were it not so stipulated. Ergo, petitioners had absolute freedom of choice whether or not to be
insured by FORTUNE under the terms of its policy and they freely opted to adhere thereto.

Indeed, and far more importantly, the cardinal polestar in the construction of an insurance contract is the
intention of the parties as expressed in the policy. Courts have no other function but to enforce the same.
The rule that contracts of insurance will be construed in favor of the insured and most strongly against
the insurer should not be permitted to have the effect of making a plain agreement ambiguous and then
construe it in favor of the insured. Verily, it is elemental law that the payment of premium is requisite to
keep the policy of insurance in force. If the premium is not paid in the manner prescribed in the policy as
intended by the parties the policy is ineffective. Partial payment even when accepted as a partial
payment will not keep the policy alive even for such fractional part of the year as the part payment bears
to the whole payment.

Interpreting the contract of insurance stringently against the insurer but liberally in favor of the insured
despite clearly defined obligations of the parties to the policy can be carried out to extremes that there is
the danger that we may, so to speak, kill the goose that lays the golden egg. We are well aware of
insurance companies falling into the despicable habit of collecting premiums promptly yet resorting to all
kinds of excuses to deny or delay payment of just insurance claims. But, in this case, the law is
manifestly on the side of the insurer. For as long as the current Insurance Code remains unchanged and
partial payment of premiums is not mentioned at all as among the exceptions provided in Secs. 77 and
78, no policy of insurance can ever pretend to be efficacious or effective until premium has been fully
paid.

And so it must be. For it cannot be disputed that premium is the elixir vitae of the insurance business
because by law the insurer must maintain a legal reserve fund to meet its contingent obligations to the
public, hence, the imperative need for its prompt payment and full satisfaction. It must be emphasized
here that all actuarial calculations and various tabulations of probabilities of losses under the risks
insured against are based on the sound hypothesis of prompt payment of premiums. Upon this bedrock
insurance firms are enabled to offer the assurance of security to the public at favorable rates. But once
payment of premium is left to the whim and caprice of the insured, as when the courts tolerate the
payment of a mere P600.00 as partial undertaking out of the stipulated total premium of P2,983.50 and
the balance to be paid even after the risk insured against has occurred, as petitioners have done in this
case, on the principle that the strength of the vinculum juris is not measured by any specific amount of
premium payment, we will surely wreak havoc on the business and set to naught what has taken
actuarians centuries to devise to arrive at a fair and equitable distribution of risks and benefits between
the insurer and the insured.
The terms of the insurance policy constitute the measure of the insurers liability. In the absence of
statutory prohibition to the contrary, insurance companies have the same rights as individuals to limit
their liability and to impose whatever conditions they deem best upon their obligations not inconsistent
with public policy. The validity of these limitations is by law passed upon by the Insurance Commissioner
who is empowered to approve all forms of policies, certificates or contracts of insurance which insurers
intend to issue or deliver. That the policy contract in the case at bench was approved and allowed
issuance simply reaffirms the validity of such policy, particularly the provision in question.

53) PAULIN v. INSULAR


47 OG 3012

Facts:
This action was instituted by the minors Vicentita Antigua Paulin and Silvina Paulin, who are
sisters assisted by their guardian ad-litem, for the purpose of collecting the amount of three life
insurance policies issued by the defendant, the Insular Life Assurance Co. ltd. In favor of their father,
Estaban Paulin, who is alleged to have been killed by the guerillas, on or about December 10, 1943.

The aforementioned policies are Policy no. 84029 in the sum of P 2,000, issued on August 1,
1940, with premiums payable on the first day of August of each year, for 20 years and the second Policy
No. 84683, in the sum of P 2, 000, issued on October 1, 1940, with premiums payable on the first day of
October of each year, for 20 years and Policy No. 85061, in the sum of P7, 000 issued on October 1,
1940, with premiums payable on the first day of every month, for 20 years. The three policies carried an
accidental death benefit clause, providing for double indemnity in case of death, under the conditions
therein set forth, and named the plaintiffs as beneficiaries in such case.

Upon demand, made on behalf of the plaintiffs, the defendant refused to pay the sums stated in
said policies, on the ground that the same had lapsed, prior to the date of death of Esteban Paulin, for
non-payment of premiums. Hence complaint herein, which, after due trial, was dismissed by the RTC.

Appellee admits having received, in connection with policy no 84029, the annual premiums due on
August 1, 1940 and August 1, 1941; in connection with policy no. 84683, the annual premiums due on
October 1, 1940, and October 1, 1941; and in connection with policy no. 85061, fifteen monthly
premiums which fell due from October 1, 9140 to December 1, 1941, inclusive.

Issue:
Whether or not the policies have lapsed for the non-payment of premium

Ruling:

Despite appellant’s assertion to the contrary, no evidence whatsoever of such payment, in relation to
the policies nos. 84029 and 84683, has been introduced. Despite appellant’s testimonial and
documentary evidence, they do not bear any contention.

Exhibit B only showed a provisional receipt for the initial premium paid on October 11, 1940 and the
acceptance letter by Esteban Paulin were only submitted in connection with Policy No. 84683.

Exhibit c, with reference to Policy no. 85061, were letters written by Esteban Paulin to the defendant on
the first day of May, July and November 1942 and March, April, June and July 1943, respectively, each
stating that it enclosed a post office money order for P36.12, which, presumably represented the
monthly premiums falling due in the months already mentioned. But these were not part of the records of
the defendant company but found among the personal belongings of the deceased Esteban Paulin.
Each of the letters in the exhibit bore the signature of Esteban which the Court finds them to be originals.
Having remained in the possession of its writer, it tends to show that the premiums for the months
specified were not forwarded to the defendant.

It is urged also that, in view of the Executive Order No. 25, series of 1944, as amended by Executive
Order No. 32, series of 1945, providing for a moratorium in the enforcement of payment of all debts or
monetary obligations contracted prior to the liberalization of the Philippines by the American forces, the
policies could not and did not lapse for non-payment of the premiums. This contention is untenable, for
an insured is not under the obligation to pay premiums. The same do not constitute a debt and the
insurance company cannot compel payment thereof, which is one only of the conditions for the
subsistence or effectivity of an insurance policy.
While the payment of premiums or assessments as specified in the insurance contract is necessary to
bind the insurer to discharge its obligations imposed by the contract, it is generally true in the case of life
insurance contracts that there is no absolute undertaking to pay the premiums or assessment and,
consequently, no personal liability of the insurer, and the insured, if he has not expressly promised to pay,
is at liberty to refuse to make the payment.

54) UCPB GENERAL INSURANCE CO. v. MASAGANA TELEMART 308 SCRA 259 (1999)

Facts: Apr 15 1991, UCPB issued 5 insurance policies covering Masagana’s various properties against
fire, for the period from May 22 1991 to May 22 1992.
March 1992, UCPB evaluated the policies and decided not to renew them upon expiration of their terms
on May 22 1992. UCPB advised Masagana’s broker, Zuellig of its intention not to renew the policies.
Apr 6 1992. UCPB gave written notice to Masagana of the non-renewal of the policies at the address
stated in the policies.
Jun 13 1992, fire razed Masagana’s property covered by 3 of the insurance policies UCPB issued.
Jul 13 1992, Masagana presented to UCPB’s cashier 5 manager’s checks in the total amount of
P225,753.95, representing premium for the renewal of the policies from May 22 1992 to May 22 1993.
No notice of loss was filed by Masagana under the policies prior to Jul 14 1992.
Jul 14 1992, Masagana filed with UCPB its formal claim for indemnification of the insured property.
On the same day, UCPB retuned to Masagana the 5 checks that it tendered, and rejected its claim for
the reasons (a) that the policies had expired and were not renewed, and (b) that the fire occurred on Jun
13 1992, before Masagana’s tender of premium payment.
Jul 21 1992, Masagana filed with RTC against UCPB for recover of P18,645,000, representing the face
value of the policies,
Oct 23 1992, motion to dismiss denied, filed an answer alleging that the complaint fails to state a cause
of action; that UCPB was not liabel to Masagana for insurance proceeds under the policies because at
the time of the loss of the property due to fire, the policies had long expired and were not renewed.

RTC: ruled in favor of Masagana, allowing it to consign with the court full payment of the corresponding
premiums, declaring Masagana to have fully complied with its obligation to pay the premium thereby
rendering the replacement-renewal policy, and ordering UCPB to pay the sums of P18,645,000, 25% of
the total amount due as and for attys fees, and P25,000 for litigation expenses and the costs of suit.

CA: affirmed the decision of the RTC, held that Masagana was allowed a 60 to 90 day credit term for the
renewal of its policies, and that the acceptance of the late premium payment suggested an
understanding that payment could be made later.

Issue: Whether or not the fire insurance policies had expired on the May 22 1992 or had been extended
or renewed by an implied credit arrangement through actual payment of premium was tendered on a
later date after the occurrence of the fire insured against.

Ruling: They expired and were not extended.


The answer is easily found in the Insurance Code. No, an insurance policy, other than life, issued
originally or on renewal, is not valid and binding until actual payment of the premium. Any agreement to
the contrary is void. The parties may not agree expressly or impliedly on the extension of credit or time
to pay the premium and consider the policy binding before actual payment.
The case of Malayan Insurance Co., Inc. vs. Cruz-Arnaldo ciited by the Court of Appeals, is not
applicable. In that case, payment of the premium was in fact actually made on December 24, 1981, and
the fire occurred on January 18, 1982. Here, the payment of the premium for renewal of the policies was
tendered on July 13, 1992, a month after the fire occurred on June 13, 1992. The assured did not even
give the insurer a notice of loss within a reasonable time after occurrence of the fire.
55) UCPB GENERAL INSURANCE CO. v. MASAGANA TELEMART 356 SCRA 307 (2001)

Facts: The SC in its Jue 15 1999 decisions reversed and set aside the assailed decision of the CA which
affirmed with modification the judgment of the RTC (a) allowing Masagana to consign the sum of
P225,735.95 as full payment of the premiums for the renewal of the 5 insurance policies on its
properties; (b) declaring the replacement-renewal policies effective and binding from May 22 1992 until
May 22 1993; and (c) ordering UCPB to pay Masagana indemnity for the said properties.

Operative facts:
All five (5) policies reflect on their face the effectivity term: "from 4:00 P.M. of 22 May 1991 to 4:00 P.M.
of 22 May 1992." On June 13, 1992, plaintiff's properties located at 2410-2432 and 2442-2450 Taft
Avenue, Pasay City were razed by fire. On July 13, 1992, plaintiff tendered, and defendant accepted,
five (5) Equitable Bank Manager's Checks in the total amount of P225,753.45 as renewal premium
payments for which Official Receipt Direct Premium No. 62926 (Exhibit "Q", Record, p. 191) was issued
by defendant. On July 14, 1992, Masagana made its formal demand for indemnification for the burned
insured properties. On the same day, defendant returned the five (5) manager's checks stating in its
letter (Exhibit "R"/"8", Record, p. 192) that it was rejecting Masagana's claim on the following grounds:
"a) Said policies expired last May 22, 1992 and were not renewed for another term;
b) Defendant had put plaintiff and its alleged broker on notice of non-renewal earlier; and
c) The properties covered by the said policies were burned in a fire that took place last June 13, 1992, or
before tender of premium payment."

The Court of Appeals disagreed with Petitioners stand that Respondents tender of payment of the
premiums on 13 July 1992 did not result in the renewal of the policies, having been made beyond the
effective date of renewal as provided under Policy Condition No. 26, which states:
26. Renewal Clause. -- Unless the company at least forty five days in advance of the end of the policy
period mails or delivers to the assured at the address shown in the policy notice of its intention not to
renew the policy or to condition its renewal upon reduction of limits or elimination of coverages, the
assured shall be entitled to renew the policy upon payment of the premium due on the effective date of
renewal.

Both the Court of Appeals and the trial court found that sufficient proof exists that Masagana, which had
procured insurance coverage from UCPB for a number of years, had been granted a 60 to 90-day credit
term for the renewal of the policies. Such a practice had existed up to the time the claims were filed.
In the SC’s 1999 Decision it defined the main issue whether the fire insurance policies issued by UCPB
to Masagana covering the period had been extended or renewed by an implied credit arrangement thru
actual payment of premium was tendered on a later date and after the occurrence of the fire insured
against. The SC ruled in the negative, and accordingly set aside and reversed the decision of the CA.
Masagana filed a MR: that the SC made in the decision it own findings of facts, which are not in accord
with those of the TC and the CA. The courts below correctly found that no notice of non-renewal was
made within 45 days before May 22 1992, or before the expiration date of the fire insurance policies.
Thus, the same were renewed by operation of law thus were effective and valid on Jun 30 1992 when
the fire occurred.

Masagana also disagrees that parties may neither agree expressly or impliedly on the extension of
credit or time to pay the premium nor consider a policy binding before actual payment; that despite the
express provision of sec. 77 of the IC, extension of credit terms in premium payment has been the
prevalent practice in the insurance industry; that extension of credit terms is not a prohibitive injunction
but merely designed for the protection of parties to an insurance contract.
UCPB’ opposition: both the TC and CA overlooked the fact that on Apr 6 1992 UCPB sent by ordinary
mail to Masagana a notice of non-renewal and sent by personal delivery a copy thereof to Masagana’s
broker, Zuellig; that the courts ignored the fact that Masagana was fully aware of the notice of non-
renewal; that a reading of Section 66 of the Insurance Code readily shows that in order for an insured to
be entitled to a renewal of a non-life policy, payment of the premium due on the effective date of renewal
should first be made, thus, Masagana’s argument that Section 77 is not a prohibitive provision finds no
authoritative support.

Upon a meticulous review of the records and reevaluation of the issues raised in the motion for
reconsideration and the pleadings filed thereafter by the parties, we resolved to grant the motion for
reconsideration. The following facts, as found by the trial court and the Court of Appeals, are indeed duly
established:
1. For years, Petitioner had been issuing fire policies to the Respondent, and these policies were
annually renewed.
2. Petitioner had been granting Respondent a 60- to 90-day credit term within which to pay the
premiums on the renewed policies.
3. There was no valid notice of non-renewal of the policies in question, as there is no proof at all that the
notice sent by ordinary mail was received by Respondent, and the copy thereof allegedly sent to Zuellig
was ever transmitted to Respondent.
4. The premiums for the policies in question in the aggregate amount of P225,753.95 were paid by
Respondent within the 60- to 90-day credit term and were duly accepted and received by Petitioners
cashier.

Issue: Whether or not sec. 77 of the Insurance Code of 1978 must be strictly applied to UCPB’s
advantage despite its practice of granting a 60-90 day credit term for the payment of premiums.

Ruling: Section 77 of the Insurance Code of 1978 provides:


SEC. 77. An insurer is entitled to payment of the premium as soon as the thing insured is exposed to the
peril insured against. Notwithstanding any agreement to the contrary, no policy or contract of insurance
issued by an insurance company is valid and binding unless and until the premium thereof has been
paid, except in the case of a life or an industrial life policy whenever the grace period provision applies.
This Section is a reproduction of Section 77 of P.D. No. 612 (The Insurance Code) promulgated on 18
December 1974. In turn, this Section has its source in Section 72 of Act No. 2427 otherwise known as
the Insurance Act as amended by R.A. No. 3540, approved on 21 June 1963, which read:
SEC. 72. An insurer is entitled to payment of premium as soon as the thing insured is exposed to the
peril insured against, unless there is clear agreement to grant the insured credit extension of the
premium due. No policy issued by an insurance company is valid and binding unless and until the
premium thereof has been paid. (Underscoring supplied)
It can be seen at once that Section 77 does not restate the portion of Section 72 expressly permitting an
agreement to extend the period to pay the premium. But are there exceptions to Section 77?
The answer is in the affirmative.

The first exception is provided by Section 77 itself, and that is, in case of a life or industrial life policy
whenever the grace period provision applies.
The second is that covered by Section 78 of the Insurance Code, which provides:
SEC. 78. Any acknowledgment in a policy or contract of insurance of the receipt of premium is
conclusive evidence of its payment, so far as to make the policy binding, notwithstanding any stipulation
therein that it shall not be binding until
A third exception was laid down in Makati Tuscany Condominium Corporation vs. Court of Appeals,
wherein we ruled that Section 77 may not apply if the parties have agreed to the payment in installments
of the premium and partial payment has been made at the time of loss. We said therein, thus:
We hold that the subject policies are valid even if the premiums were paid on installments. The records
clearly show that the petitioners and private respondent intended subject insurance policies to be
binding and effective notwithstanding the staggered payment of the premiums. The initial insurance
contract entered into in 1982 was renewed in 1983, then in 1984. In those three years, the insurer
accepted all the installment payments. Such acceptance of payments speaks loudly of the insurers
intention to honor the policies it issued to petitioner. Certainly, basic principles of equity and fairness
would not allow the insurer to continue collecting and accepting the premiums, although paid on
installments, and later deny liability on the lame excuse that the premiums were not prepaid in full.

By the approval of the aforequoted findings and conclusion of the Court of Appeals, Tuscany has
provided a fourth exception to Section 77, namely, that the insurer may grant credit extension for the
payment of the premium. This simply means that if the insurer has granted the insured a credit term for
the payment of the premium and loss occurs before the expiration of the term, recovery on the policy
should be allowed even though the premium is paid after the loss but within the credit term.

Moreover, there is nothing in Section 77 which prohibits the parties in an insurance contract to provide a
credit term within which to pay the premiums. That agreement is not against the law, morals, good
customs, public order or public policy. The agreement binds the parties. Article 1306 of the Civil Code
provides:
ART. 1306. The contracting parties may establish such stipulations clauses, terms and conditions as
they may deem convenient, provided they are not contrary to law, morals, good customs, public order, or
public policy.
Finally in the instant case, it would be unjust and inequitable if recovery on the policy would not be
permitted against UCPB, which had consistently granted a 60- to 90-day credit term for the payment of
premiums despite its full awareness of Section 77. Estoppel bars it from taking refuge under said
Section, since Respondent relied in good faith on such practice. Estoppel then is the fifth exception to
Section 77.

56) AMERICAN HOME ASSURANCE v. CHUA

Facts: 1990, Chua obtained from AHA a fire insurance covering the stock-in-trade of his business,
Moonlight Enterprises, located in Bukidnon. The insurance was due to expire on Mar 25 1990.
Apr 5 1990: Chua issued PCI bank check in the amt of P2,893.50 to AHA’s agent, James Uy, as
payment for the renewal of the policy. In turn, AHA delivered the Renewal Certificate to Chua. The check
was drawn against a Manila bank and deposited in AHA’s bank acct in CDO City. Subsequently, a new
insurance policy was issued whereby AHA undertook to indemnify Chua for any damage or loss arising
from fire up to P200,000 (Mar 25 1990 – Mar 25 1991).
Apr 6 1990, Moonlight Enterprises was completely razed by fire; total loss P4M – P5M. Chua filed
insurance claim with AHA and 4 other co-insurers (Pioneer, Prudential, Filipino Merchants, and Domestic
Insurance). AHA refused to the claim, hence, action against it before the trial court.
AHA, in its defense, claimed that there was no existing insurance contract when the fire occurred since
Chua did not pay the premium; that even assuming there was a contract, Chua violated several
conditions of the policy, particularly: (1) his submission of fraudulent income tax return and financial
statements, (2) failure to establish actual loss, which AHA assessed at P70,000, and (3) his failure to
notify to AHA of any insurance already effected to cover the insured goods.

TC: ruled in favor of Chua; held that Chua paid by way of check a day before the fire occurred,
deposited in AHA’s account and the same was even acknowledged in the renewal certificate issued by
AHA’s agent; that the alleged fraudulent documents were limited to the disparity between the official
receipts issued by the BIR and the ITRs for the years 1987 to 1989; that all other documents were found
to be genuine; that the failure of Chua to notify AHA of the other insurance contracts were not intentional
nor fraudulent.

CA: affirmed the decision in toto; that Chua’s claim was substantially proved and AHA’s unjustified
refusal to pay the claim entitled Chua to the award of damages.
American Home filed the petition reiterating its stand that there was no existing insurance contract
between the parties. It invoked Section 77 of the Insurance Code, which provides that no policy or
contract of insurance issued by an insurance company is valid and binding unless and until the premium
thereof has been paid and the case of Arce v. Capital Insurance that until the premium is paid there is no
insurance.

Issues:
Whether or not there was a valid payment of premium, considering that Chua’s check was cashed after
the fire.

Whether or not Chua violated the policy by his submission of fraudulent documents and non-disclosure
of other insurance contracts

Ruling:
Yes.

The renewal certificate issued to respondent contained the acknowledgment that premium had been
paid. It is not disputed that the check drawn by respondent in favor of petitioner and delivered to its
agent was honored when presented and petitioner forthwith issued its official receipt to respondent on
10 April 1990. Section 306 of the Insurance Code provides that any insurance company which delivers a
policy or contract of insurance to an insurance agent or insurance broker shall be deemed to have
authorized such agent or broker to receive on its behalf payment of any premium which is due on such
policy or contract of insurance at the time of its issuance or delivery or which becomes due thereon. In
the instant case, the best evidence of such authority is the fact that petitioner accepted the check and
issued the official receipt for the payment. It is, as well, bound by its agent’s acknowledgment of receipt
of payment.

In the instant case, the best evidence of such authority is the fact that petitioner accepted the check and
issued the official receipt for the payment. It is, as well, bound by its agent’s acknowledgment of receipt
of payment.

Section 78 of the Insurance Code explicitly provides:


An acknowledgment in a policy or contract of insurance of the receipt of premium is conclusive evidence
of its payment, so far as to make the policy binding, notwithstanding any stipulation therein that it shall
not be binding until the premium is actually paid.

No.

Submission of the alleged fraudulent documents pertained to respondent’s income tax returns for 1987
to 1989. Respondent, however, presented a BIR certification that he had paid the proper taxes for the
said years. Since this is a question of fact, the finding is conclusive.

Ordinarily, where the insurance policy specifies as a condition the disclosure of existing co-insurers, non-
disclosure is a violation that entitles the insurer to avoid the policy. The purpose for the inclusion of this
clause is to prevent an increase in the moral hazard. The relevant provision is Section 75, which
provides that:

A policy may declare that a violation of specified provisions thereof shall avoid it, otherwise the breach of
an immaterial provision does not avoid the policy.
Respondent acquired several co-insurers and he failed to disclose this information to
petitioner. Nonetheless, petitioner is estopped from invoking this argument due to the loss adjuster’s
admission of previous knowledge of the co-insurers.
It cannot be said that petitioner was deceived by respondent by the latter’s non-disclosure of the other
insurance contracts when petitioner actually had prior knowledge thereof. The loss adjuster, being an
employee of petitioner, is deemed a representative of the latter whose awareness of the other insurance
contracts binds petitioner.

DOUBLE INSURANCE

57) PIONEER INSURANCE AND SURETY CORPORATION VS OLIVA YAP, G.R. NO. L-36232

Facts: Respondent Oliva Yap was the owner of a store in a 2 storey building where she sold shopping
bags and footwear. Chua Soon Poon, Oliva Yap’s son-in-law, was in charge of the store. Respondent
Yap took out a fire insurance policy from petitioner Pioneer insurance and surety corporation. Among the
conditions in the policy executed by the parties are the following:
The Insured shall give notice to the Company of any insurance or insurances already effected, or which
may subsequently be effected, covering any of the property hereby insured, and unless such notice be
given and the particulars of such insurance or insurances be stated in, or endorsed on this Policy by or
on behalf of the Company before the occurrence of any loss or damage, all benefits under this Policy
shall be forfeited. (emphasis supplied)

It is understood that, except as may be stated on the face of this policy there is no other insurance on
the property hereby covered and no other insurance is allowed except by the consent of the Company
endorsed hereon. Any false declaration or breach or this condition will render this policy null and void.
At the time of the insurance, an insurance policy issued by the Great American Insurance Company
covering the same properties was noted on said policy as co-insurance. Later, the parties executed an
endorsement on the policy, stating:
It is hereby declared and agreed that the co-insurance existing at present under this policy is as follows:
P20,000.00 — Northwest Ins., and not as originally stated. (emphasis supplied)
Except as varied by this endorsement, all other terms and conditions remain unchanged.
Still later, Oliva Yap took out another fire insurance policy covering the same properties, this time from
Federal Insurance Company, which new policy was, however, procured without notice to and the written
consent of petitioner Pioneer Insurance & Surety Corporation and therefore, was not noted as a co-
insurance on the policy.

Later, a fire broke out in the building housing Yap’s store and the said store was burned. Respondent
Yap filed an insurance claim but the same was denied on the ground of “breach and/or violation of any
and/or terms and conditions” of the policy.
Oliva Yap filed the present complaint.

Issue: WON petitioner should be absolved from liability on fire insurance policy on account of any
violation by respondent Yap of the co-insurance clause. — Yes

Held: There was a violation by respondent Oliva Yap of the co-insurance clause contained in Policy No.
4219 that resulted in the avoidance of petitioner's liability. The insurance policy for P20,000.00 issued by
the Great American Insurance Company covering the same properties of respondent Yap and duly noted
on Policy No. 4219 as c-insurance, ceased, by agreement of the parties (Exhibit "1-L"), to be recognized
by them as a co-insurance policy. The Court of Appeals says that the Great American Insurance policy
was substituted by the Federal Insurance policy for the same amount, and because it was a mere case
of substitution, there was no necessity for its endorsement on Policy No. 4219. This finding, as well as
reasoning, suffers from several flaws. There is no evidence to establish and prove such a substitution. If
anything was substituted for the Great American Insurance policy, it could only be the Northwest
Insurance policy for the same amount of P20,000.00. The endorsement (Exhibit "1-K") quoted above
shows the clear intention of the parties to recognize on the date the endorsement was made (August 29,
1962), the existence of only one co-insurance, and that is the Northwest Insurance policy, which
according to the stipulation of the parties during the hearing, was issued on August 20, 1962 (t.s.n.,
January 12, 1965, pp. 3-4) and endorsed only on August 20, 1962. The finding of the Court of Appeals
that the Great American Insurance policy was substituted by the Federal Insurance policy is
unsubstantiated by the evidence of record and indeed contrary to said stipulation and admission of
respondent, and is grounded entirely on speculation, surmises or conjectures, hence, not binding on the
Supreme Court.

The Court of Appeals would consider petitioner to have waived the formal requirement of endorsing the
policy of co-insurance "since there was absolutely no showing that it was not aware of said substitution
and preferred to continue the policy." The fallacy of this argument is that, contrary to Section 1, Rule 131
of the Revised Rules of Court, which requires each party to prove his own allegations, it would shift to
petitioner, respondent's burden of proving her proposition that petitioner was aware of the alleged
substitution, and with such knowledge preferred to continue the policy. Respondent Yap cites Gonzales
La O vs. Yek Tong Lin Fire and Marine Insurance Co., Ltd. to justify the assumption but in that case,
unlike here, there was knowledge by the insurer of violations of the contract, to wit: "If, with the
knowledge of the existence of other insurances which the defendant deemed violations of the contract, it
has preferred to continue the policy, its action amounts to a waiver of the annulment of the contract ..." A
waiver must be express. If it is to be implied from conduct mainly, said conduct must be clearly indicative
of a clear intent to waive such right. Especially in the case at bar where petitioner is assumed to have
waived a valuable right, nothing less than a clear, positive waiver, made with full knowledge of the
circumstances, must be required.

By the plain terms of the policy, other insurance without the consent of petitioner would ipso facto avoid
the contract. It required no affirmative act of election on the part of the company to make operative the
clause avoiding the contract, wherever the specified conditions should occur. Its obligations ceased,
unless, being informed of the fact, it consented to the additional insurance.

The validity of a clause in a fire insurance policy to the effect that the procurement of additional
insurance without the consent of the insurer renders ipso facto the policy void is well-settled: In
Milwaukee Mechanids' Lumber Co., vs. Gibson, 199 Ark. 542, 134 S. W. 2d 521, 522, a substantially
identical clause was sustained and enforced, the court saying: "The rule in this state and practically all of
the states is to the effect that a clause in a policy to the effect that the procurement of additional
insurance without the consent of the insurer renders the policy void is a valid provision. The earlier
cases of Planters Mutual Insurance Co., vs. Green, 72 Ark. 305, 80 S.W. 92, are to the same effect."
And see Vance, Insurance, 2nd Ed., 725. (Reach vs. Arkansas Farmers Mut. Fire Ins. Co., [Ark. Nov. 14,
1949] 224 S. W. 2d 48, 49.)

The annotation then, must be deemed to be a warranty that the property was not insured by any other
policy. Violation thereof entitled the insurer to rescind. (Sec. 69, Insurance Act.) Such misrepresentation
is fatal in the light of our views in Santa Ana vs. Commercial Union Assurance Company, Ltd., 55 Phil.
329. The materiality of non-disclosure of other insurance policies is not open to doubt.
Furthermore, even if the annotations were overlooked the defendant insurer would still be free from
liability because there is no question that the policy issued by General Indemnity has not been stated in
nor endorsed on Policy No. 471 of defendant. And as stipulated in the above-quoted provisions of such
policy "all benefit under this policy shall be forfeited. (Emphasis supplied)

The obvious purpose of the aforesaid requirement in the policy is to prevent over-insurance and thus
avert the perpetration of fraud. The public, as well as the insurer, is interested in preventing the situation
in which a fire would be profitable to the insured. According to Justice Story: "The insured has no right to
complain, for he assents to comply with all the stipulation on his side, in order to entitle himself to the
benefit of the contract, which, upon reason or principle, he has no right to ask the court to dispense with
the performance of his own part of the agreement, and yet to bind the other party to obligations, which,
but for those stipulation would not have been entered into."
In view of the above conclusion, We deem it unnecessary to consider the other defenses interposed by
petitioner.

58) UNION MANUFACTURING CO., INC AND THE REPUBLIC BANK VS PHILIPPINE GUARANTY
CO., INC., G.R. NO. L-27932, OCTOBER 30, 1972

Facts: On January 12, 1962, Union Manufacturing obtained certain loans, overdrafts and other credit
accommodations from Republic bank and to secure payment thereof, said Union Manufacturing
executed a real and chattel mortgages on certain properties, which are more particularly described and
listed at the back of the mortgage contract. That as additional condition of the mortgage contract, Union
Manufacturing undertook to secure insurance coverage over the mortgaged properties.

Union manufacturing failed to secure insurance coverage on the mortgaged properties despite the fact
that Cua Tok, its general manager, was reminded of said requirement, Republic Bank procured from
Philippine Guaranty an insurance coverage on loss against fire over the properties of Union
Manufacturing as described in Philippine Guaranty’s “Cover Note” with the annotation that the loss or
damage, if any, under said cover note, is payable to republic bank as its interest may appear, subject
however to the printed conditions of said defendant’s fire insurance policy form.

A fire insurance policy was issued in favor of assured, Union Manufacturing for which a premium was
paid by the republic bank to Phil. Guaranty. It appears that although said renewal premium was paid by
the Republic Bank, such payment was for the account of Union Manufacturing and that the cash
voucher for the payment of the first premium was paid also by the Republic Bank but for the account of
Union Manufacturing.

A fire occurred in the premises of Union Manufacturing. They then filed a fire claim with the Philippine
Guaranty which was denied on the ground that: “Policy condition no. 3 and/or the ‘Other Insurance
Clause’ of the policy violated because you did not give notice to us the other insurance which you had
taken from New India and Manila Insurance with the results that these insurances, of which we became
aware of only after the fire, were not endorsed on our policy; and Policy condition no. 1 was not
complied with because you have failed failed to give to our representatives the required documents and
other proofs with respect to your claim and matters touching on our liability, if any, and the amount of
such liability.”

That when the defendant Philippine Guaranty issued a fire insurance policy to cover the properties of the
Union Manufacturing Co., the same properties were already covered by fire policy of the Sincere
insurance company and Oceanic Insurance. That when said Philippine Guaranty’s fire insurance policy
was already in full force and effect, the Union Manufacturing without the consent of the Philippine
Guaranty, obtained another insurance policy over the same property prior to the fire from New india
Assurance, Sincere Insurance company and Manila Insurance Co.
Issue: WON Republic Bank can recover from the Philippine Guaranty — No

Held: Why the appellant Republic Bank could not recover, as payee, in case of loss as its "interest may
appear subject to the terms and conditions, clauses and warranties" of the policy was expressed in the
appealed decision thus: "However, inasmuch as the Union Manufacturing Co., Inc. has violated the
condition of the policy to the effect that it did not reveal the existence of other insurance policies over the
same properties, as required by the warranty appearing on the face of the policy issued by the
defendant and that on the other hand said Union Manufacturing Co., Inc. represented that there were no
other insurance policies at the time of the issuance of said defendant's policy, and it appearing
furthermore that while the policy of the defendant was in full force and effect the Union Manufacturing
Co., Inc. secured other fire insurance policies without the written consent of the defendant endorsed on
the policy, the conclusion is inevitable that both the Republic Bank and Union Manufacturing Co., Inc.
cannot recover from the same policy of the defendant because the same is null and void." The tone of
confidence apparent in the above excerpts from the lower court decision is understandable. The
conclusion reached by the lower court finds support in authoritative precedents. It is far from easy,
therefore, for appellant Republic Bank to impute to such a decision a failure to abide by the law. Hence,
as noted at the outset, the appeal cannot prosper. An affirmance is indicated.

It is to Santa Ana v. Commercial Union Assurance Co., a 1930 decision, that one turns to for the first
explicit formulation as to the controlling principle. As was made clear in the opinion of this Court, penned
by Justice Villa-Real: "Without deciding whether notice of other insurance upon the same property must
be given in writing, or whether a verbal notice is sufficient to render an insurance valid which requires
such notice, whether oral or written, we hold that in the absolute absence of such notice when it is one
of the conditions specified in the fire insurance policy, the policy is null and void." The next year, in Ang
Giok Chip v. Springfield Fire & Marine Ins. Co., the conformity of the insured to the terms of the policy,
implied from the failure to express any disagreement with what is provided for, was stressed in these
words of the ponente, Justice Malcolm: "It is admitted that the policy before us was accepted by the
plaintiff. The receipt of this policy by the insured without objection binds both the acceptor and the
insured to the terms thereof. The insured may not thereafter be heard to say that he did not read the
policy or know its terms, since it is his duty to read his policy and it will be assumed that he did so.” As
far back as 1915, in Young v. Midland Textile Insurance Company, it was categorically set forth that as a
condition precedent to the right of recovery, there must be compliance on the part of the insured with the
terms of the policy. As stated in the opinion of the Court through Justice Johnson: "If the insured has
violated or failed to perform the conditions of the contract, and such a violation or want of performance
has not been waived by the insurer, then the insured cannot recover. Courts are not permitted to make
contracts for the parties. The function and duty of the courts consist simply in enforcing and carrying out
the contracts actually made. While it is true, as a general rule, that contracts of insurance are construed
most favorably to the insured, yet contracts of insurance, like other contracts, are to be construed
according to the sense and meaning of the terms which the parties themselves have used. If such terms
are clear and unambiguous they must be taken and understood in their plain, ordinary and popular
sense." More specifically, there was a reiteration of this Santa Ana ruling in a decision by the then
Justice, later Chief Justice, Bengzon, in General Insurance & Surety Corp. v. Ng Hua. 12 Thus: "The
annotation then, must be deemed to be a warranty that the property was not insured by any other policy.
Violation thereof entitles the insurer to rescind. (Sec. 69, Insurance Act) Such misrepresentation is fatal
in the light of our views in Santa Ana v. Commercial Union Assurance Company, Ltd. ... . The materiality
of non-disclosure of other insurance policies is not open to doubt.” As a matter of fact, in a 1966 decision,
Misamis Lumber Corp. v. Capital Ins. & Surety Co., Inc., Justice J.B.L. Reyes, for this Court, made
manifest anew its adherence to such a principle in the face of an assertion that thereby a highly
unfavorable provision for the insured would be accorded recognition. This is the language used: "The
insurance contract may be rather onerous ('one sided', as the lower court put it), but that in itself does
not justify the abrogation of its express terms, terms which the insured accepted or adhered to and
which is the law between the contracting parties.”

There is no escaping the conclusion then that the lower court could not have disposed of this case in a
way other than it did. Had it acted otherwise, it clearly would have disregarded pronouncements of this
Court, the compelling force of which cannot be denied. There is, to repeat, no justification for a reversal.
MARINE INSURANCE

59) ORIENTAL ASSURANCE CORPORATION VS COURT OF APPEALS AND PANAMA SAWMILL,


G.R. NO. 94052, AUGUST 9, 1991

Facts: Panama Sawmill bought 1,208 pieces of apitong log which it transported by sea to Manila through
Transpacific towage and insured it against loss with Oriental Assurance corporation. Oriental assurance
issued a marine insurance policy which stated, among others, that the insurance warranted against total
loss only.

The 2 barges were towed by 1 tug-boat but during the voyage, rough seas and strong winds caused
damage to the barge resulting in the total loss of 497 pieces of logs out of the 598 pieces loaded thereon.
Panama demanded payment for the loss but Oriental assurance refused on the ground that its
contracted liability was for “total loss only.” The rejection was upon the recommendation of the Tan
Gatue Adjustment Company.

Panama filed a Complaint for Damages against Oriental Assurance et al. with RTC.

Issue: WON Oriental Assurance can be held liable under its marine insurance policy based on the theory
of a divisible contract of insurance and, consequently, a constructive total loss.

Held: No, as no liability attaches. In the absence of either actual or constructive total loss, there can be
no recovery by the insured Panama against the insurer, Oriental Assurance.

The terms of the contract constitute the measure of the insurer’s liability and compliance therewith is a
condition precedent to the insured's right to recovery from the insurer. Whether a contract is entire or
severable is a question of intention to be determined by the language employed by the parties. The
policy in question shows that the subject matter insured was the entire shipment of 2,000 cubic meters
of apitong logs. Only one premium was paid for the entire shipment, making for only one cause or
consideration. The insurance contract must, therefore, be considered indivisible.
More importantly, the insurer's liability was for "total loss only." A total loss may be either actual or
constructive. An actual total loss is caused by:
A total destruction of the thing insured;
The irretrievable loss of the thing by sinking, or by being broken up;
Any damage to the thing which renders it valueless to the owner for the purpose for which he held it; or
Any other event which effectively deprives the owner of the possession, at the port of destination, of the
thing insured.

A constructive total loss is one which gives to a person insured a right to abandon, under Section 139 of
the Insurance Code. This provision reads:
A person insured by a contract of marine insurance may abandon the thing insured, or any particular
portion thereof separately valued by the policy, or otherwise separately insured, and recover for a total
loss thereof, when the cause of the loss is a peril injured against, (a) If more than three-fourths thereof in
value is actually lost, or would have to be expended to recover it from the peril; (b) If it is injured to such
an extent as to reduce its value more than three-fourths xxx
The CA treated the loss as a constructive total loss, and for the purpose of computing the more than
three-fourths value of the logs actually lost, considered the cargo in one barge as separate from the logs
in the other. Thus, it concluded that the loss of 497 pieces of logs from barge TPAC-1000,
mathematically speaking, is more than three-fourths (¾) of the 598 pieces of logs loaded in that barge
and may, therefore, be considered as constructive total loss. Erroneous.
The requirements for the application of Section 139 of the Insurance Code, have not been met. The logs
involved, although placed in two barges, were not separately valued by the policy, nor separately insured.
Resultantly, the logs lost in barge TPAC-1000 in relation to the total number of logs loaded therein on the
same barge cannot be made the basis for determining constructive total loss. The logs having been
insured as one inseparable unit, the correct basis for determining the constructive total loss is the totality
of the shipment of logs. So, out of the 1,208, pieces of logs, only 497 pieces were lost or 41.45% of the
entire shipment. Since it is less than 75% of the value of all 1,208 pieces of logs, the shipment cannot be
said to have sustained a constructive total loss.
60) ISABELA ROQUE AND ONG CHIONG VS HON. INTERMEDIATE APPELLATE COURT AND
PIONEER INSURANCE AND SURETY CORPORATION, G.R. NO. L-66935, NOVEMBER 11, 1985

Facts: Manila Bay Lighterage Corporation, a common carrier, entered into a contract with the petitioners
whereby the former would load and carry on board logs in its barge Marble 10. The petitioners insured
the logs against loss with Pioneer Insurance and Surety Corporation.
However, the shipment of pieces of logs never reached its destination because the barge sank. As found
by both the trial court and appellate courts, the barge where the logs were loaded was not seaworthy
such that it developed a leak. The appellate court further found that one of the hatches was left open
causing water to enter the barge and because the barge was not provided with the necessary cover or
tarpaulin, the ordinary splash of sea waves brought more water inside the barge.

The petitioners wrote to Manila Bay a letter demanding payment for the loss and another letter was sent
to Pioneer claiming the amount under the insurance policy but latter refused on the ground that its ability
depended upon the “Total loss by Total loss of vessel only.” Hence, petitioner commenced the civil case.
Petitioners: The implied warranty of seaworthiness provided for in the Insurance Code refers only to the
responsibility of the shipowner who must see to it that his ship is reasonably fit to make in safety the
contemplated voyage. A mere shipper of cargo, having no control over the ship, has nothing to do with
its seaworthiness. A cargo owner has no control over the structure of the ship, its cables, anchors, fuel
and provisions, the manner of loading his cargo and the cargo of other shippers, and the hiring of a
sufficient number of competent officers and seamen.

SC: Unmeritorious. There is no dispute over the liability of the common carrier Manila Bay. In fact, it did
not bother to appeal the questioned decision. However, the petitioners state that Manila Bay has ceased
operating as a firm and nothing may be recovered from it. They are, therefore, trying to recover their
losses from the insurer.

The liability of the insurance company is governed by law. Section 113 of the Insurance Code provides:
In every marine insurance upon a ship or freight, or freightage, or upon anything which is the subject of
marine insurance, a warranty is implied that the ship is seaworthy.
Section 99 of the same Code also provides in part:
Marine insurance includes: (1) Insurance against loss of or damage to: (a) Vessels, craft, aircraft,
vehicles, goods, freights, cargoes, merchandise x x x
The term "cargo" can be the subject of marine insurance and that once it is so made, the implied
warranty of seaworthiness immediately attaches to whoever is insuring the cargo whether he be the
shipowner or not. In every contract of insurance upon anything which is the subject of marine insurance,
a warranty is implied that the ship shall be seaworthy at the time of the inception of the voyage.
Moreover, the fact that the unseaworthiness of the ship was unknown to the insured is immaterial in
ordinary marine insurance and may not be used by him as a defense in order to recover on the marine
insurance policy.

Since the law provides for an implied warranty of seaworthiness in every contract of ordinary marine
insurance, it becomes the obligation of a cargo owner to look for a reliable common carrier which keeps
its vessels in seaworthy condition. The shipper of cargo may have no control over the vessel but he has
full control in the choice of the common carrier that will transport his goods. Or the cargo owner may
enter into a contract of insurance which specifically provides that the insurer answers not only for the
perils of the sea but also provides for coverage of perils of the ship.
We are constrained to apply Section 113 of the Insurance Code to the facts of this case. In marine cases,
the risks insured against are "perils of the sea." The purpose of such insurance is protection against
contingencies and against possible damages and such a policy does not cover a loss or injury which
must inevitably take place in the ordinary course of things.

Perils of the sea:


extends only to losses caused by sea damage, or by the violence of the elements, and does not
embrace all losses happening at sea
losses from extraordinary occurrences only, such as stress of weather, winds and waves, lightning,
tempests, rocks and the like
include only such losses as are of extraordinary nature, or arise from some overwhelming power, which
cannot be guarded against by the ordinary exertion of human skill and prudence
damage done to a vessel by perils of the sea includes every species of damages done to a vessel at
sea, as distinguished from the ordinary wear and tear of the voyage, and distinct from injuries suffered
by the vessel in consequence of her not being seaworthy at the outset of her voyage (as in this case)
everything which happens thru the inherent vice of the thing, or by the act of the owners, master or
shipper, shall NOT be reputed a peril, if not otherwise borne in the policy
Petitioners: The loss of the cargo was caused by the perils of the sea, not by the perils of the ship. As
found by the trial court, the barge was turned loose from the tugboat east of Cabuli Point "where it was
buffeted by storm and waves." Moreover, barratry (any willful misconduct on the part of the master or
crew, in pursuance of an unlawful or fraudulent purpose, without consent of the owner and to the
prejudice of the owner’s interest; still covered under perils of the sea) against which the cargo was also
insured, existed when the personnel of the tugboat and the barge committed a mistake by turning loose
the barge from the tugboat east of Cabuli Point.

SC: Unmeritorious. The facts clearly negate the petitioners' claim under the insurance policy. The loss of
the cargo was due to the perils of the ship rather than the perils of the sea. The entrance of the sea
water into the ship's hold through the defective pipe was not due to any accident which happened during
the voyage, but to the failure of the ship's owner properly to repair a defect of the existence of which he
was apprised. The loss was therefore more analogous to that which directly results from simple
unseaworthiness than to that which result from the perils of the sea.

Perils of the ship:


loss in the ordinary course of events
results from the natural and inevitable action of the sea, from the ordinary wear and tear of the ship, or
from the negligent failure of the ship's owner to provide the vessel with proper equipment to convey the
cargo under ordinary conditions

GR: The insurer does not undertake to insure against perils of the ship.
EXN: To make insurer liable, there must some casualty – something which could not be foreseen as one
of the necessary incidents of the adventure. The purpose of the policy is to secure an indemnity against
accidents which may happen, not against events which must happen.
Therefore, the insurer is not liable. The shipowner excepts the perils of the sea from his engagement
under the bill of lading, while this is the very perils against which the insurer intends to give protection.
The owners of the lost logs must look to the shipowner for redress and not to the insurer.

61) FILIPINO MERCHANTS INSURANCE CO., INC VS COURT OF APPEALS AND CHOA TIEK SENG,
G.R. NO. 85141, NOVEMBER 28, 1989

Facts: Choa Tiek Seng insured with Filipino Merchants Insurance the shipment of 600 metric tons (but
actually was only 59.94 m. tons) of fishmeal in new gunny bags of 90 kilos each against all risks under
warehouse to warehouse terms. The fishmeal were unloaded from the ship unto the arrastre contractor.
The condition of the bad order was reflected in the turn over survey report. The cargo was also surveyed
by the arrastre contractor before delivery of the cargo to the consignee and the condition in such
delivery was reflected covering a total of 227 bags in bad order condition. Choa Tiek Sieng filed a formal
claim statement against the vessel but the Filipino Merchants Insurance refused to pay the claim. Choa
filed an action with RTC.

RTC rendered decision in favor of Choa. The CA affirmed decision.

Petitioners: An "all risks" marine policy has a technical meaning in insurance in that before a claim can
be compensable it is essential that there must be "some fortuity, " "casualty" or "accidental cause" to
which the alleged loss is attributable and the failure of herein private respondent, upon whom lay the
burden, to adduce evidence showing that the alleged loss to the cargo in question was due to a
fortuitous event precludes his right to recover from the insurance policy.

SC: Untenable. The "all risks clause" of the Institute Cargo Clauses read as follows:
5. This insurance is against all risks of loss or damage to the subject-matter insured but shall in no case
be deemed to extend to cover loss, damage, or expense proximately caused by delay or inherent vice or
nature of the subject-matter insured. Claims recoverable hereunder shall be payable irrespective of
percentage.

An "all risks policy" should be read literally as meaning all risks whatsoever and covering all losses by an
accidental cause of any kind. The terms "accident" and "accidental", as used in insurance contracts,
have not acquired any technical meaning. They are construed by the courts in their ordinary and
common acceptance. Thus, the terms have been taken to mean that which happens by chance or
fortuitously, without intention and design, and which is unexpected, unusual and unforeseen. An accident
is an event that takes place without one's foresight or expectation; an event that proceeds from an
unknown cause, or is an unusual effect of a known cause and, therefore, not expected.
The very nature of the term "all risks" must be given a broad and comprehensive meaning as covering
any loss other than a willful and fraudulent act of the insured. This is pursuant to the very purpose of an
"all risks" insurance to give protection to the insured in those cases where difficulties of logical
explanation or some mystery surround the loss or damage to property. An "all asks" policy has been
evolved to grant greater protection than that afforded by the "perils clause," in order to assure that no
loss can happen through the incidence of a cause neither insured against nor creating liability in the ship;
it is written against all losses, that is, attributable to external causes.

Generally, the burden of proof is upon the insured to show that a loss arose from a covered peril, but
under an "all risks" policy the initial burden is on the insured to prove that the cargo was in good
condition when the policy attached and that the cargo was lost, destroyed or deteriorated when
unloaded from the vessel; thereafter, the burden then shifts to the insurer to prove that the loss was due
to excepted perils.

There being no showing that the loss was caused by any of the excepted perils, the insurer is liable
under the policy. There is no evidence presented to show that the condition of the gunny bags in which
the fishmeal was packed was such that they could not hold their contents in the course of the necessary
transit, much less any evidence that the bags of cargo had burst as the result of the weakness of the
bags themselves. Had there been such a showing that spillage would have been a certainty, there may
have been good reason to plead that there was no risk covered by the policy. Under an 'all risks' policy, it
was sufficient to show that there was damage occasioned by some accidental cause of any kind, and
there is no necessity to point to any particular cause.
Filipino Merchants is to pay Choa P51,568.62 with interest at legal rate from the date of the filing of the
complaint.

62) CHOA TIEK SENG, doing business under the name and style of SENG'S COMMERCIAL
ENTERPRISES vs. HON. COURT OF APPEALS, FILIPINO MERCHANTS' INSURANCE COMPANY,
INC., BEN LINES CONTAINER, LTD. AND E. RAZON, INC, G.R. No. 84507, [March 15, 1990] -
Deogracias

FACTS: On November 4, 1976 petitioner imported some lactose crystals from Holland. The importation
involved 600 6-ply paper bags with polyethylene inner bags, each bag at 25 kilos net. The goods were
loaded at the port at Rotterdam in sea vans on board the vessel "MS Benalder” as the mother vessel,
and thereafter aboard the feeder vessel "Wesser Broker V-25" of respondent Ben Lines Container, Ltd.
The goods were insured by the respondent Filipino Merchants' Insurance Co., Inc. for P98,882.35, the
equivalent of US$8,765.00 plus 50% mark-up against all risks under the terms of the insurance cargo
policy. Upon arrival at the port of Manila, the cargo was discharged into the custody of the arrastre
operator/broker respondent E. Razon, Inc., prior to the delivery to petitioner. Of the 600 bags delivered
to petitioner, 403 were in bad order. The surveys showed that the bad order bags suffered spillage and
loss later valued at P33,117.63.

Petitioner filed a claim against respondent insurance company, which was rejected, alleging that
assuming that spillage took place while the goods were in transit, petitioner and his agent failed to avert
or minimize the loss by failing to recover spillage from the sea van, thus violating the terms of the
insurance policy sued upon; and that assuming that the spillage did not occur while the cargo was in
transit, the said 400 bags were loaded in bad order, and that in any case, the van did not carry any
evidence of spillage. Choa filed complaint in RTC. RTC dismissed case. CA affirmed dismissal.
CA: The cargo in question was insured in an "against all risk policy." Insurance "against all risk" has a
technical meaning in marine insurance. Under an "all risk" marine policy, there must be a general rule be
a fortuitous event in order to impose liability on the insurer; losses occasioned by ordinary
circumstances or wear and tear are not covered, thus, while an "all risk" marine policy purports to cover
losses from casualties at sea, it does not cover losses occasioned by the ordinary circumstances of a
voyage, but only those resulting from extra and fortuitous events.

ISSUE: Whether or not an "all risks" coverage covers only losses occasioned by or resulting from "extra
and fortuitous events" despite the clear and unequivocal definition of the term made and contained in the
policy sued upon.

HELD: No. CA erred. An all risk insurance policy insures against all causes of conceivable loss or
damage, except as otherwise excluded in the policy or due to fraud or intentional misconduct on the part
of the insured. It covers all losses during the voyage whether arising from a marine peril or not, including
pilferage losses during the war.
In the present case, the "all risks" clause of the policy sued upon reads as follows:
This insurance is against all risks of loss or damage to the subject matter insured but shall in no case be
deemed to extend to cover loss, damage, or expense proximately caused by delay or inherent vice or
nature of the subject matter insured. Claims recoverable hereunder shall be payable irrespective of
percentage.

The insurance policy covers all loss or damage to the cargo except those caused by delay or inherent
vice or nature of the cargo insured. It is the duty of the respondent insurance company to establish that
said loss or damage falls within the exceptions provided for by law, otherwise it is liable therefor. In this
case, the damage caused to the cargo has not been attributed to any of the exceptions provided for nor
is there any pretension to this effect. Thus, respondent insurance company must pay.

63) DELSAN TRANSPORT LINES, INC. vs. THE HON. COURT OF APPEALS and AMERICAN HOME
ASSURANCE CORPORATION, G.R. No. 127897, [November 15, 2001]

FACTS: Caltex Philippines entered into a contract of affreightment with the Delsan Transport Lines, Inc.,
whereby the said common carrier agreed to transport Caltex's industrial fuel oil from the Batangas-
Bataan Refinery to different parts of the country. The shipment was insured with American Home
Assurance Corporation. On August 14, 1986, petitioner's vessel, the MT Maysun, set sail from Batangas
for Zamboanga City. Unfortunately, the vessel sank taking with it the entire cargo of fuel oil. Private
respondent paid Caltex the sum of P5,096,635.57 representing the insured value of the lost cargo.
Exercising its right of subrogation under Article 2207 of the New Civil Code, the private respondent
demanded of the petitioner the same amount it paid to Caltex.

Due to its failure to collect from the petitioner despite prior demand, private respondent filed a complaint
with the RTC for collection of a sum of money. RTC dismissed complaint, finding that the vessel, MT
Maysun, was seaworthy to undertake the voyage as determined by the Philippine Coast Guard per
Survey Certificate Report upon inspection during its annual dry-docking and that the incident was
caused by unexpected inclement weather condition or force majeure, thus exempting petitioner from
liability for the loss of its cargo.

On appeal, CA reversed decision.


Delsan: When American Homes paid Caltex, it was equivalent to a tacit recognition that the ill-fated
vessel was seaworthy; otherwise, the former was not legally liable to latter due to the latter's breach of
implied warranty under the marine insurance policy that the vessel was seaworthy.

ISSUES:
1.WON the payment made by American Home to Caltex for the insured value of the lost cargo
amounted to an admission that the vessel was seaworthy, thus precluding any action for recovery
against the petitioner.

2. WON the non-presentation of the marine insurance policy bars the complaint for recovery of sum of
money for lack of cause of action.
HELD:
1. NO. The payment made by American Home for the insured value of the lost cargo operates as waiver
of its right to enforce the term of the implied warranty against Caltex under the marine insurance policy.
However, the same cannot be validly interpreted as an automatic admission of the vessel’s
seaworthiness by American Home as to foreclose recourse against Delsan for any liability under its
contractual obligation as a common carrier. The fact of payment grants American Home subrogatory
right which enables it to exercise legal remedies that would otherwise be available to Caltex as owner of
the lost cargo against Delsan, the common carrier.

From the nature of their business and for reasons of public policy, common carriers are bound to
observe extraordinary diligence in the vigilance over the goods and for the safety of passengers
transported by them, according to all the circumstances of each case. In the event of loss, destruction or
deterioration of the insured goods, common carriers shall be responsible unless the same is brought
about, among others, by flood, storm, earthquake, lightning or other natural disaster or calamity. In all
other cases, if the goods are lost, destroyed or deteriorated, common carriers are presumed to have
been at fault or to have acted negligently, unless they prove they observed extraordinary diligence.
In order to escape liability for the loss of its cargo of industrial fuel oil belonging to Caltex, Delsan
attributes the sinking of MT Maysun to fortuitous event or force majeure. But the testimony of the captain
and chief mate that there were strong winds and waves 20 feet high was effectively rebutted and belied
by the weather report of PAGASA. Thus, as the CA correctly ruled, Delsan’s vessel, MT Maysun, sank
with its entire cargo for the reason that it was not seaworthy. There was no squall or bad weather or
extremely poor sea condition in the vicinity where the said vessel sank.

Additionally, the exoneration of MT Maysun’s officers and crew merely concern their respective
administrative liabilities. It does not in any way operate to absolve Delsan the common carrier from its
civil liability arising from its failure to observe extraordinary diligence in the vigilance over the goods it
was transporting and for the negligent acts or omissions of its employees, the determination of which
properly belongs to the courts. In the case at bar, Delsan is liable for the insured value of the lost cargo
of industrial fuel oil belonging to Caltex for its failure to rebut the presumption of fault or negligence as
common carrier occasioned by the unexplained sinking of its vessel, MT Maysun, while in transit.

2. NO. The presentation in evidence of the marine insurance policy is not indispensable in this case
before the insurer may recover from the common carrier the insured value of the lost cargo in the
exercise of its subrogatory right. The subrogation receipt, by itself, is sufficient to establish not only the
relationship of American Home as insurer and Caltex, as the assured shipper of the lost cargo of
industrial fuel oil, but also the amount paid to settle the insurance claim. The right of subrogation accrues
simply upon payment by the insurance company of the insurance claim.

64) FRANCISCO DEL VAL, ET AL. vs. ANDRES DEL VAL,, G.R. No. 9374, [February 16, 1915]

: The parties are siblings who were the only heirs at law and next of kin of Gregorio del Val, who passed
away intestate. During the lifetime of the deceased he took out insurance on his life for the sum of
P40,000 and made it payable to Andres del Val as sole beneficiary. After his death, the defendant
Andres collected the face of the policy. He paid the sum of P18,365.20 to redeem certain real estate
which the decedent had sold to third persons with a right to repurchase. The redemption of said
premises was made by the attorney of the defendant in the name of the petitioners and Andres as heirs
of the deceased vendor. Andres, on death of the Gregorio, took possession of most of the latter’s
personal property and also the balance on the insurance policy amounting to P21,634.80.
Plaintiffs: The amount of the insurance policy belonged to the estate of the deceased and not to the
defendant personally, hence they are entitled to a partition not only of the real and personal property, but
also of the P40,000 life insurance. The complaint prays a partition of all the property, both real and
personal, left by the deceased, and that the defendant account for P21,634.80. They also wanted to
divide this equally among the plaintiffs and defendant along with the other property of deceased.
Defendant Andres: Redemption of the real estate sold by his father was made in the name of the
plaintiffs and himself instead of in his name alone without his knowledge or consent. He also averred
that it was not his intention to use the proceeds of the insurance policy for the benefit of any person but
himself, he alleging that he was and is the sole owner thereof and that it is his individual property.
ISSUE: Can the proceeds of the policy be divided among the heirs?

HELD: NO. The proceeds of the life-insurance policy belong exclusively to the defendant as his
individual and separate property. The proceeds of an insurance policy belong exclusively to the
beneficiary and not to the estate of the person whose life was insured, and such proceeds are the
separate and individual property of the beneficiary, and not of the heirs of the person whose life was
insured. This doctrine in America is embedded in the Code of Commerce where:
The amount which the underwriter must deliver to the person insured, in fulfillment of the contract, shall
be the property of the latter, even against the claims of the legitimate heirs or creditors of any kind
whatsoever of the person who effected the insurance in favor of the former.
The contract of life insurance is a special contract and the destination of the proceeds is determined by
special laws which deal exclusively with that subject. The Civil Code has no provisions which relate
directly and specifically to life- insurance contracts or to the destination of life insurance proceeds. That
was under the Code of Commerce.
The plaintiffs claim that the property repurchased with the insurance proceeds belongs to the heirs in
common and not to the defendant alone. This wasn’t agreed upon by the court unless the facts
appeared that Andres acted as he did with the intention that the other heirs should enjoy with him the
ownership of the estate.

65) THE BANK OF THE PHILIPPINE ISLANDS, administrator of the estate of the late Adolphe Oscar
Schuetze, vs. JUAN POSADAS, JR., Collector of Internal Revenue, Jr., G.R. No. 34583, [1931]

FACTS: BPI, as administrator, appealed to CFI absolving defendant, Collector of Internal Revenue, from
the complaint filed against it in recovering the inheritance tax amounting to P1,209 paid by Rosario
Gelano Vda de Schuetze, under protest, and sum of P20,150 representing the proceeds of the
insurance policy of the deceased.

Rosario and Adolphe were married in January 1914. The wife was actually residing and living in
Germany when Adolphe died in December 1927. The latter while in Germany, executed a will in March
1926, pursuant with its law wherein Rosario was named his universal heir. The deceased possessed
not only real property situated in the Philippines but also personal property consisting of shares of stocks
in 19 domestic corporations. Included in the personal property is a life insurance policy issued at Manila
on January 1913 for the sum of $10,000 by the Sun Life Assurance Company of Canada, Manila Branch.
In the insurance policy, the estate of the deceased was named the beneficiary without any qualification.
Rosario is the sole and only heir of the deceased. BPI, as administrator of the decedent’s estate and
attorney in fact of Rosario, having been demanded by Posadas to pay the inheritance tax, paid under
protest. Notwithstanding various demands made by plaintiff, Posadas refused to refund such amount.

ISSUE: Whether the proceeds of the life insurance policy is paraphernal or community property. (if
community, then not subject to inheritance tax)

HELD: As all the premiums on the life-insurance policy taken out by the late Adolphe Oscar Schuetze,
were paid out of the conjugal funds, with the exception of the first, the proceeds of the policy,
excluding the proportional part corresponding to the first premium, constitute community property,
notwithstanding the fact that the policy was made payable to the deceased's estate, so that one-
half of said proceeds belongs to the estate, and the other half to the deceased's widow, Rosario. CIR is
ordered to return to BPI the one-half of the tax collected upon the amount of P20,150, being the
proceeds of the insurance policy on the life of the late Adolphe, after deducting the proportional part
corresponding to the first premium.

Both according to our Civil Code and to the ruling of those North American States where the Spanish
Civil Code once governed, the proceeds of a life-insurance policy whereon the premiums were paid with
conjugal money, belong to the conjugal partnership.
A life-insurance policy belongs exclusively to the beneficiary upon the death of the person insured, and
that in the present case, as Adolphe named his own estate as the sole beneficiary of the insurance on
his life, upon his death the latter became the sole owner of the proceeds, which therefore became
subject to the inheritance tax. An heir appointed beneficiary to a life- insurance policy taken out
by the deceased, becomes the absolute owner of the proceeds of such policy upon the death of the
insured.

The proceeds of a life-insurance policy payable to the insured's estate, on which the premiums were
paid by the conjugal partnership, constitute community property, and belong one-half to the husband and
the other half to the wife, exclusively. But if the premiums were paid partly with paraphernal and partly
conjugal funds, the proceeds are likewise in like proportion paraphernal in part and conjugal in part.
The proceeds of a life-insurance policy payable to the insured's estate as the beneficiary, if delivered to
the testamentary administrator of the former as part of the assets of said estate under probate
administration, are subject to the inheritance tax according to the law on the matter, if they belong to the
assured exclusively, and it is immaterial that the insured was domiciled in these Islands or outside.

66) THE INSULAR LIFE ASSURANCE COMPANY, LTD. vs. CARPONIA T. EBRADO and PASCUALA
VDA. DE EBRADO, G.R. No. L-44059, [October 28, 1977]

FACTS: On September 1, 1968, Buenaventura Cristor Ebrado was issued by the Insular Life Assurance
Co., Ltd., a policy on a whole-life plan for P5,882 with a rider for Accidental Death Benefits for the same
amount. He designated Carponia T. Ebrado as the revocable beneficiary in his policy. He referred to her
as his wife.

On October 21, 1969, Buenventura C. Ebrado died as a result of an accident when he was hit by a
falling branch of a tree. As the insurance policy was in force, The Insular Life Assurance Co., Ltd. stands
liable to pay the coverage of the policy in an amount of P11,745.73, representing the face value of the
policy in the amount of P5,882.00 plus the additional benefits for accidental death also in the amount of
P5,882.00 and the refund of P18.00 paid for the premium due November, 1969, minus the unpaid
premiums and interest thereon due for January and February, 1969, in the sum of P36.27.

Carponia filed with the insurer a claim for the proceeds of the policy as the designated beneficiary
therein, although she admits that she and the insured Buenaventura were merely living as husband and
wife without the benefit of marriage. Pascuala Vda. de Ebrado also filed her claim as the widow of the
deceased insured. She asserts that she is the one entitled to the insurance proceeds, not the common-
law wife, Carponia.

ISSUE: Can the common-law wife, Carponia Ebrado, named as beneficiary in the life insurance policy of
a legally married man claim the proceeds thereof in case of death of the latter?

HELD: NO. The general rules of civil law should be applied to resolve matters not specifically provided in
the Insurance Law. Article 2011 of the New Civil Code states:
The contract of insurance is governed by special laws. Matters not expressly provided for in such special
laws shall be regulated by this Code.

And under Article 2012 of the same Code:


Any person who is forbidden from receiving any donation under Article 739 cannot be named beneficiary
of a fife insurance policy by the person who cannot make a donation to him.
Common-law spouses are, definitely, barred from receiving donations from each other. Also, conviction
for adultery or concubinage is not required as only preponderance of evidence is necessary.
In essence, a life insurance policy is no different from a civil donation insofar as the beneficiary is
concerned. Both are founded upon the same consideration: liberality. A beneficiary is like a donee,
because the premiums of the policy which the insured pays out of liberality, the beneficiary will receive
the proceeds or profits of said insurance.
CASH SURRENDER VALUE

67) THE MANUFACTURERS LIFE INSURANCE CO., plaintiff-appellant, vs. BIBIANO L. MEER, in the c
apacity as Collector of Internal Revenue, defendant-appellee (GR No. L-2910, 1951)

Facts: The Manufacturers Life Insurance Company is duly registered and licensed to engage in life insur
ance business in the Philippines for more than five years until 1941. But due to the exigencies of the war
it closed the branch office at Manila during 1942 up to September 1945. In the course of its operations b
efore the war, plaintiff issued a number of life insurance policies in the Philippines containing nonforfeitur
e clauses, viz:
8. Automatic Premium Loan — This Policy shall not lapse for non-payment of any premium after it has b
een three full years in force, if, at the due date of such premium, the Cash Value of this Policy and of any
bonus additions and dividends left on accumulation (after deducting any indebtedness to the Company
and the interest accrued thereon) shall exceed the amount of said premium. In which event the company
will, without further request, treat the premium then due as paid, and the amount of such premium, with
interest from its actual due date at six per cent per annum, compounded yearly, and one per cent, comp
ounded yearly, for expenses, shall be a first lien on this Policy in the Company's favour in priority to the c
laim of any assignee or any other person. The accumulated lien may at any time, while the Policy is in fo
rce, be paid in whole or in part xxx

From January 1, 1942 to December 31, 1946 for failure of the insured under the above policies to pay th
e corresponding premiums for one or more years, the plaintiff's head office at Toronto, applied the provisi
ons of the automatic premium loan clauses; and the net amount of premiums so advanced or loaned tot
alled P1,069,254.98. On this sum the defendant CIR assessed P17,917.12 — which plaintiff paid supra
protest. The assessment was made pursuant to section 255 of the National Internal Revenue Code as a
mended, which partly provides:
SEC. 255. Taxes on insurance premiums — There shall be collected from every person, company, or cor
poration (except purely cooperative companies or associations) doing insurance business of any sort in t
he Philippines a tax of one per centum of the total premiums collected . . . whether such premiums are p
aid in money, notes, credits, or any substitute for money but premiums refunded within six months after
payment on account of rejection of risk or returned for other reason to person insured shall not be includ
ed in the taxable receipts . . . .

Plantiff: When there are premium loans or premium advances, as above stated, by virtue of the non-forfe
iture clauses, it did not collect premiums within the meaning of the above sections of the law, and therefo
re it is not amenable to the tax therein provided.

Issues:
WON premium advances made by plaintiff-appellant under the automatic premium loan clause of its poli
cies are 'premiums collected' by the Company subject to tax.

WON in the application of the automatic premium loan clause of plaintiff-appellant's policies, there is ‘pa
yment in money, notes, credits, or any substitutes for money.’

WON the collection of the alleged deficiency premium taxes constitutes double taxation.

Whether the making of premium advances, granting for the sake of argument that it amounted to collecti
on of premiums, were done in Toronto, Canada, or in the Philippines.

WON the fact that plaintiff-appellant was not doing business in the Philippines from January 1, 1942 to S
eptember 30, 1945 exempts it from payment of premium taxes corresponding to said period.

Held:
Illustration: Suppose that 'A', 30 years of age, secures a 20-year endowment policy for P5,000 from plain
tiff-appellant Company and pays an annual premium of P250. 'A' pays the first ten yearly premiums amo
unting to P2,500 and on this amount plaintiff-appellant pays the corresponding taxes under section 255
of the NIRC. Suppose also that the cash value of said policy after the payment of the 10th annual premi
um amounts to P1,000. When on the 11th year the annual premium fell due and the insured remitted no
money within the month's grace, the insurer treated the premium then over due as paid from the cash va
lue, the amount being a loan to the policyholder who could discharge it at any time with interest at 6 per
cent. The insurance contract, therefore, continued in force for the 11th year.

Yes. "How could there be such a collection" plaintiff argues "when as a result thereof, insurer becomes a
creditor, acquires a lien on the policy and is entitled to collect interest on the amount of the unpaid premi
ums?" Wittingly or unwittingly, the "premium" and the "loan" have been interchanged in the argument. Th
e insurer "became a creditor" of the loan, but not of the premium that had already been paid. And it is en
titled to collect interest on the loan, not on the premium. In other words, "A" paid the premium for the ele
venth year; but in turn he became a debtor of the company for the sum of P250. This debt he could repa
y either by later remitting the money to the insurer or by letting the cash value compensate for it. The de
bt may also be deducted from the amount of the policy should "A" die thereafter during the continuance
of the policy. The debt may also be deducted from the amount of the policy should the insured die therea
fter during the continuance of the policy.

There was an increase in the assets of the insurer. There was the new credit for the advances made. Tr
ue, the plaintiff could not sue the insured to enforce that credit. But it has means of satisfaction out of the
cash surrender value. If the credit is paid out of the cash surrender value, there were no new funds add
ed to the company's assets. Cash surrender value "as applied to a life insurance policy, is the amount of
money the company agrees to pay to the holder of the policy if he surrenders it and releases his claims
upon it. The more premiums the insured has paid the greater will be the surrender value; but the surrend
er value is always a lesser sum than the total amount of premiums paid."

The cash value or cash surrender value is therefore an amount which the insurance company holds in tr
ust for the insured to be delivered to him upon demand. It is therefore a liability of the company to the ins
ured. Now then, when the company's credit for advances is paid out of the cash value or cash surrender
value, that value and the company's liability is thereby diminished pro tanto. The decrease of a person's l
iabilities means a corresponding increase in his net assets.

Yes. The insurer agreed to consider the premium paid on the strength of the automatic loan. The premiu
m was therefore paid by means of a "note" or "credit" or "other substitute for money" and the tax is due b
ecause section 255 above quoted levies taxes according to the total premiums collected by the insurer "
whether such premiums are paid in money, notes, credits or any substitute for money.

No. Appellant goes back to the illustration, "A failed to pay the premium on the 11th year and the insurer
advanced P250 from the cash value. If the amount of P250 is deducted from the cash value of P1,000 of
the policy, then taxing this P250 anew as premium collected, as was done in the present case, will amo
unt to double taxation since taxes had already been collected on the cash value of P1,000 as part of the
P2,500 collected as premiums for the first ten years." The trouble with the argument is that it assumes al
l advances are necessarily repaid from the cash value. That is true in some cases. In others the insured
subsequently remits the money to repay the advance and to keep unimpaired the cash reserve of his pol
icy. Of the total amount advanced (P1,069,255) P158,667 had actually been repaid at the time of assess
ment notice. Besides, the premiums paid and on which taxes had already been collected, were those for
the 10 years. The tax demanded is on the premium for the 11th year. Further, there is no constitutional p
rohibition against double taxation.

Philippines. Appellant: as the advances of premiums were made in Toronto, such premiums are deemed
to have been paid there — not in the Philippines — and therefore those payments are not subject to loca
l taxation. The law does not contemplate premiums collected in the Philippines. Subscribing to this would
make foreign insurers evade the tax by contriving to require that premium payments shall be made at th
eir head offices. It is enough that the insurer is doing insurance business in the Philippines, irrespective
of the place of its organization or establishment. In any event there is no constitutional prohibition agains
t double taxation.

Untenable. Although during those years the appellant was not open for new business because its branch
office was closed, still it was practically and legally, operating in this country by collecting premiums on it
s outstanding policies, incurring the risks and/or enjoying the benefits consequent thereto, without havin
g previously taken any steps indicating withdrawal in good faith from this field of economic activity. Furth
er, in objecting to the payment of the tax, plaintiff-appellant never insisted, before the BIR that it was not
engaged in business in this country during those years.

SURETYSHIP

68) DEVELOPMENT BANK OF THE PHILIPPINES, petitioner, vs. COURT OF APPEALS and the ESTA
TE OF THE LATE JUAN B. DANS, represented by CANDIDA G. DANS, and the DBP MORTGAGE RED
EMPTION INSURANCE POOL, respondents GR No. 109937, 1994

Facts:
In May 1987, Juan B. Dans, together with his family, applied for a loan of P500,000 with DBP Basilan Br
anch. As the principal mortgagor, Dans, then 76 years of age, was advised by DBP to obtain a mortgage
redemption insurance with the DBP Mortgage Redemption Insurance Pool (DBP MRI Pool). A loan, in th
e reduced amount of P300,000, was approved and released by DBP. From the proceeds of the loan, DB
P deducted the amount of P1,476 as payment for the MRI premium. Dans accomplished and submitted t
he "MRI Application for Insurance" and the "Health Statement for DBP MRI Pool." The MRI premium of
Dans, less the DBP service fee of 10 percent, was credited by DBP to the savings account of the DBP M
RI Pool. Accordingly, the DBP MRI Pool was advised of the credit.

On September 3, 1987, Dans died of cardiac arrest. The DBP, upon notice, relayed this information to th
e DBP MRI Pool. On September 23, 1987, the DBP MRI Pool notified DBP that Dans was not eligible for
MRI coverage, being over the acceptance age limit of 60 years at the time of application. DBP apprised
Candida Dans of the disapproval of her late husband's MRI application. The DBP offered to refund the pr
emium of P1,476, but Candida refused to accept it, demanding payment of the face value of the MRI or
an amount equivalent to the loan. She, likewise, refused to accept anex gratia settlement of P30,000, wh
ich the DBP later offered.

Respondent Estate, through Candida Dans as administratrix, filed a complaint with RTC against DBP an
d the insurance pool for "Collection of Sum of Money with Damages." RTC decided in favor of responde
nt Estate and against DBP. The DBP MRI Pool, however, was absolved from liability, after the trial court f
ound no privity of contract between it and the deceased. The trial court declared DBP in estoppel for hav
ing led Dans into applying for MRI and actually collecting the premium and the service fee, despite know
ledge of his age ineligibility. The CA affirmed in toto.

Issue: WON DBP and DBP MRI Pool are liable.

Held: DBP, yes and DBP MRI Pool, no. As to DBP MRI Pool, it did not approve the application of Dans.
There is also no showing that it accepted the sum of P1,476, which DBP credited to its account with full
knowledge that it was payment for Dan's premium. Under the provisions of the Health Statement for DB
P Pool, the MRI coverage shall take effect: (1) when the application shall be approved by the insurance
pool; and (2) when the full premium is paid during the continued good health of the applicant. These two
conditions, being joined conjunctively, must concur. There was no perfected contract of insurance; hence
, the DBP MRI Pool cannot be held liable on a contract that does not exist.

As to DBP, it was wearing two legal hats: the first as a lender, and the second as an insurance agent.
In dealing with Dans, DBP As an insurance agent, DBP made Dans go through the motion of applying fo
r said insurance, thereby leading him and his family to believe that they had already fulfilled all the requir
ements for the MRI and that the issuance of their policy was forthcoming. Apparently, DBP had full knowl
edge that Dan's application was never going to be approved. The maximum age for MRI acceptance is 6
0 years as clearly and specifically provided in the Group Mortgage Redemption Insurance Policy. Under
Article 1987 of the Civil Code of the Philippines, "the agent who acts as such is not personally liable to th
e party with whom he contracts, unless he expressly binds himself or exceeds the limits of his authority
without giving such party sufficient notice of his powers." The DBP is not authorized to accept application
s for MRI when its clients are more than 60 years of age. Knowing all the while that Dans was ineligible f
or MRI coverage because of his advanced age, DBP exceeded the scope of its authority when it accepte
d Dan's application for MRI by collecting the insurance premium, and deducting its agent's commission a
nd service fee. There is no showing that Dans knew of the limitation on DBP's authority to solicit applicat
ions for MRI. If the third person dealing with an agent is unaware of the limits of the authority conferred b
y the principal on the agent and he (third person) has been deceived by the non-disclosure thereof by th
e agent, then the latter is liable for damages to him. Inasmuch as the non-disclosure of the limits of the a
gency carries with it the implication that a deception was perpetrated on the unsuspecting client, the pro
visions of Articles 19, 20 and 21 of the Civil Code of the Philippines come into play.

Article 19 – Every person must, in the exercise of his rights and in the performance of his duties, act with
justice give everyone his due and observe honesty and good faith.

Article 20 – Every person who, contrary to law, willfully or negligently causes damage to another, shall in
demnify the latter for the same.

Article 21 – Any person, who willfully causes loss or injury to another in a manner that is contrary to mor
als, good customs or public policy shall compensate the latter for the damage.

The DBP's liability, however, cannot be for the entire value of the insurance policy. To assume that were i
t not for DBP's concealment of the limits of its authority, Dans would have secured an MRI from another i
nsurance company, and therefore would have been fully insured by the time he died, is highly speculativ
e. Considering his advanced age, there is no absolute certainty that Dans could obtain an insurance cov
erage from another company. It must also be noted that Dans died almost immediately, i.e., on the ninet
eenth day after applying for the MRI, and on the twenty-third day from the date of release of his loan. On
e is entitled to an adequate compensation only for such pecuniary loss suffered by him as he has duly pr
oved. Damages, to be recoverable, must not only be capable of proof, but must be actually proved with
a reasonable degree of certainty. Speculative damages are too remote to be included in an accurate esti
mate of damages.

WHEREFORE, the decision of the Court of Appeals is MODIFIED and petitioner DBP is ORDERED: (1)
to REIMBURSE respondent Estate of Juan B. Dans the amount of P1,476 with legal interest from the da
te of the filing of the complaint until fully paid; and (2) to PAY said Estate P50,000 as moral damages an
d P10,000 as attorney's fees. With costs against petitioner.

Note: Wa bitaw ko kasabot ngano ni-fall under Suretyship ni siya nga topic, kay wa jud na namention ma
ski kausa. Basin ang pgconstitute sa Mortgage Redemption Insurance Contract? Which in this case was
not perfected.
CLAIMS SETTLEMENT

69) TIO KHE CHIO, petitioner, vs. THE HONORABLE COURT OF APPEALS and EASTERN ASSURAN
CE AND SURETY CORPORATION, respondents GR No. 76101-02, 1991

Facts: On December 18, 1978, Tio Khe Chio imported 1,000 bags of fishmeal valued at $36,000.30 from
Agro Impex, Texas, USA. The goods were insured with respondent EASCO and shipped on board the M
/V Peskov, a vessel owned by Far Eastern Shipping Company. When the goods reached Manila, they w
ere found to have been damaged by sea water which rendered the fishmeal useless. Petitioner filed a cl
aim with EASCO and Far Eastern Shipping. Both refused to pay. Petitioner sued them before the then C
FI for damages. EASCO, as the insurer, filed a counterclaim against the petitioner for the recovery of P1
8,387.86 representing the unpaid insurance premiums.

CFI ordered EASCO and Far Eastern Shipping to pay petitioner solidarily the sum of P105,986.68 less t
he amount of P18,387.86 for unpaid premiums with interest at the legal rate from the filing of the complai
nt, the sum of P15,000.00 as attorney's fees and the costs. The judgment became final as to EASCO bu
t the shipping company appealed to the CA and was absolved from liability by the said court.

The trial court, upon motion by petitioner, issued a writ of execution against EASCO. The sheriff enforcin
g the writ reportedly fixed the legal rate of interest at 12%. Respondent EASCO moved to quash the writ
alleging that the legal interest to be computed should be 6%. The trial court denied EASCO's motion. EA
SCO then filed a petition for certiorari and prohibition before the Court of Appeals.

CA held that the interest that the private respondent is entitled to collect from the petitioner is hereby red
uced to 6% per annum.

Issue: What is the legal rate of interest to be imposed in actions for damages arising from unpaid insura
nce claims? Petitioner Tio Khe Chio claims that it should be 12% pursuant to Articles 243 and 244 of the
Insurance Code while EASCO claims that it should be 6% under Article 2209 of the Civil Code.

Held: 6%.
Section 243 of the Insurance Code provides:
xxx Refusal or failure to pay the loss or damage within the time prescribed herein will entitle the assured
to collect interest on the proceeds of the policy for the duration of the delay at the rate of twice the ceilin
g prescribed by the Monetary Board, unless such failure or refusal to pay is based on the ground that th
e claim is fraudulent.

Section 244 of the aforementioned Code also provides:


In case of any litigation for the enforcement of any policy or contract of insurance, it shall be the duty of t
he Commissioner or the Court, as the case may be, to make a finding as to whether the payment of the
claim of the insured has been unreasonably denied or withheld; and in the affirmative case, the insuranc
e company shall be adjudged to pay damages which shall consist of attorney's fees and other expenses
incurred by the insured person by reason of such undeniable denial or withholding of payment plus inter
est of twice the ceiling prescribed by the Monetary Board of the amount of the claim due the insured xxx
The CA made no finding that there was an unjustified refusal or withholding of payment on petitioner's cl
aim. In fact, the CA had this to say:
xxx EASCO's refusal to settle the claim to Tio Khe Chio was based on some ground which, while not suff
icient to free it from liability under its policy, nevertheless is sufficient to negate any assertion that in refu
sing to pay, it acted unjustifiably. xxx The case posed some genuine issues of interpretation of the terms
of the policy as to which persons may honestly differ. This is the reason the trial court did not say EASC
O's refusal was unjustified.

So the aforecited sections of the Insurance Code are not pertinent to the instant case. They apply only w
hen the court finds an unreasonable delay or refusal in the payment of the claims.

Neither does Circular No. 416 of the Central Bank which took effect on July 29, 1974 pursuant to PD 11
6 (Usury Law) which raised the legal rate of interest from 6% to 12% apply to the case at bar as contend
ed by the petitioner. The adjusted rate mentioned in the circular refers only to loans or forbearances of m
oney, goods or credits and court judgments thereon but not to court judgments for damages arising from
injury to persons and loss of property which does not involve a loan.

The legal rate of interest is 6% per annum, and not 12%, where a judgment award is based on an action
for damages for personal injury, not use or forbearance of money, goods or credit. The interest by way o
f damages is governed by Article 2209 of the Civil Code, which reads:
If the obligation consists in the payment of a sum of money and the debtor incurs in delay, the indemnity
for damages, there being no stipulation to the contrary, shall be the payment of interest agreed upon, an
d in the absence of stipulation, the legal interest which is six per cent per annum.

Since the contending parties did not allege the rate of interest stipulated in the insurance contract, the le
gal interest was properly pegged by the Appellate Court at 6%.

70) FINMAN GENERAL ASSURANCE CORPORATION, petitioner, vs. COURT OF APPEALS and USIP
HIL INCORPORATED, respondents GR No. 138737, 2001

Facts: Usiphil Inc. insured certain properties (office, furniture, fixtures, shop machinery and other trade e
quipment) from fire with Finman (then doing business under the name Summa Insurance Corporation).
Sometime in 1982, Usiphil filed an insurance claim amounting to P987,126.11 for the loss of the insured
properties due to fire. Finman appointed Adjuster H.H. Bayne to undertake the valuation and adjustment
of the loss. Usiphil submitted its Sworn Statement of Loss and Formal Claim, signed by Reynaldo Cayet
ano, Usiphil’s Manager, and a Proof of Loss signed by its Accounting Manager Pedro Palallos and count
ersigned by H.H. Bayne's Adjuster F.C. Medina.

Palallos personally followed-up private respondent's claim with Finman’s President Joaquin Ortega. Duri
ng their meeting, Ortega instructed their Finance Manager, Rosauro Maghirang, to reconcile the records.
Thereafter, Maghirang and Palallos signed a Statement/Agreement, dated February 28, 1985, which ind
icated that the amount due respondent was P842,683.40.

Despite repeated demands, petitioner refused to pay the insurance claim. Thus, private respondent was
constrained to file a complaint against petitioner for the unpaid insurance claim. Finman maintained that
Usiphil cannot recover because it failed to comply with Policy Condition No. 13 regarding the submission
of certain documents to prove the loss. RTC ruled in favor of Usiphil. CA modified decision as to #1 belo
w, but affirmed in toto the others: Finman is
1. To pay the plaintiff the sum of P842,683.40 and to pay 24% interest per annum from 03 May 1985 unti
l fully paid;
2. To pay the plaintiff the sum equivalent to 10% of the principal obligation as and for attorney's fees, plu
s P1,500.00 per court appearance of counsel;
3. To pay the plaintiff the amount of P30,000.00 as exemplary damages in addition to the actual and com
pensatory damages awarded xxx

Issues:
Was there compliance by Usiphil of Condition No. 13?
Was the 24% interest rate proper?

Held:
Yes. Both the trial court and the CA concur in holding that private respondent had substantially complied
with Policy Condition No. 13. Requirements under which were allegedly communicated to private respon
dent in the two letters of H.H. Bayne to private respondent. The letter stated, among others:
To be able to expedite adjustment of this case, please submit to us without delay the following document
s and/or particulars:
For FFF, Machineries/Equipment Claims and For Stock Claim
1. Your formal claim (which may be accomplished in the enclosed form) accompanied by a detailed inve
ntory of the documents submitted.
2. Certification from the appropriate government office indicating the date of the occurrence of the fire, th
e property involved, its location and possible point of origin.
3. Proof of premium payment.
4. Three color photographs of the debris properly captioned/identified/dated and initiated by the claimant
at the back.
Though our adjusters will also take photographs in the manner prescribed above, please do not rely on
his photographs in the preservations of your evidence of loss thru pictures.
5. Copies of purchase invoices xxx

Factual findings and conclusions of the trial court and the CA are entitled to great weight and respect, an
d will not be disturbed on appeal in the absence of any clear showing that the trial court overlooked certa
in facts or circumstances which would substantially affect the disposition of the case. There is no cogent
reason to deviate from this salutary rule in the present case.

Policy Condition No. 13 reads:


The insured shall give immediate written notice to the Company of any loss, protect the property from fur
ther damage, forthwith separate the damaged and undamaged personal property, put it in the best possi
ble order, furnish a complete inventory of the destroyed, damaged, and undamaged property, showing in
detail quantities, costs, actual cash value and the amount of loss claimed; AND WITHIN SIXTY DAYS A
FTER THE LOSS, UNLESS SUCH TIME IS EXTENDED IN WRITING BY THE COMPANY, THE INSUR
ED SHALL RENDER TO THE COMPANY A PROOF OF LOSS, signed and sworn to by the insured, stati
ng the knowledge and belief of the insured as to the following: the time and origin of the loss, the interest
of the insured and of all others in the property, the actual cash value of each item thereof and the amou
nt of loss thereto, all encumbrances thereon, all other contracts of insurance, whether valid or not, coveri
ng any of said property, any changes in the title, use, occupation, location, possession or exposures of s
aid property since the issuing of this policy by whom and for what purpose any buildings herein describe
d and the several parts thereof were occupied at the time of loss and whether or not it then stood on lea
sed ground, and shall furnish a copy of all the descriptions and schedules in all policies, and if required v
erified plans and specifications of any building, fixtures, or machinery destroyed or damaged. The insure
d, as often as may be reasonably required, shall exhibit to any person designated by the company all th
at remains of any property herein described, and submit to examination under oath by any person name
d by the Company, and subscribe the same; and, as often as may be reasonably required, shall produce
for examination all books of account, bills, invoices, and other vouchers or certified copies thereof if orig
inals be lost, at such reasonable time and place as may be designated by the Company or its representa
tive and shall permit extracts and copies thereof to be made.

No claim under this policy shall be payable unless the terms of this condition have been complied with.
Usiphil, after the occurrence of the fire, immediately notified petitioner thereof. Thereafter, private respon
dent submitted the following documents: (1) Sworn Statement of Loss and Formal Claim and; (2) Proof o
f Loss. The submission of these documents, to the Court's mind, constitutes substantial compliance with
the above provision. As regards the submission of documents to prove loss, substantial, not strict, compl
iance with the requirements will always be deemed sufficient.

In any case, Finman (formerly Summa Insurance) itself acknowledged its liability when through its Finan
ce Manager, Rosauro Maghirang, it signed the document indicating that the amount due Usiphil is P842,
683.40. Even assuming that plaintiff-appellee indeed failed to submit certain required documents as proo
f of loss per Section 13, such violation was waived by the insurer Summa when it signed the document-b
reakdown of the amount due to plaintiff-appellee on the insurance claim. By such act, defendant-appella
nt acknowledged its liability under the insurance policy. Finman alleges that Maghirang was without auth
ority to sign and therefore without authority to bind Finman. Untenable. At a meeting between Usiphil's c
orporate president Pedro Pallalos and Finman’s Joaquin Ortega, the latter summoned Rosauro Maghira
ng to reconcile the claims of plaintiff-appellee. One who clothes another with apparent authority as his a
gent and holds him to the public as such, cannot later be allowed to deny the authority of such person to
act as his agent when such third person entered into the contract in good faith and in an honest belief th
at he is such agent.

Yes. It is authorized by Sections 243 and 244 of the Insurance Code.


SECTION 243. The amount of any loss or damage for which an insurer may be liable, under any policy o
ther than life insurance policy, shall be paid within thirty days after proof of loss is received by the insurer
and ascertainment of the loss or damage is made either by agreement between the insured and the ins
urer or by arbitration; but if such ascertainment is not had or made within sixty days after such receipt by
the insurer of the proof of loss, then the loss or damage shall be paid within ninety days after such recei
pt. Refusal or failure to pay the loss or damage within the time prescribed herein will entitle the assured t
o collect interest on the proceeds of the policy for the duration of the delay at the rate of twice the ceiling
prescribed by the Monetary Board, unless such failure or refusal to pay is based on the ground that the
claim is fraudulent.

SECTION 244. In case of any litigation for the enforcement of any policy or contract of insurance, it shall
be the duty of the Commissioner or the Court, as the case may be, to make a finding as to whether the
payment of the claim of the insured has been unreasonably denied or withheld; and in the affirmative ca
se, the insurance company shall be adjudged to pay damages which shall consist of attorney's fees and
other expenses incurred by the insured person by reason of such unreasonable denial or withholding of
payment plus interest of twice the ceiling prescribed by the Monetary Board of the amount of the claim d
ue the insured, from the date following the time prescribed in section two hundred forty-two or in section
two hundred forty-three, as the case may be, until the claim is fully satisfied: Provided, That the failure to
pay any such claim within the time prescribed in said sections shall be considered prima facie evidence
of reasonable delay in payment.

Notably, under Section 244, a prima facie evidence of unreasonable delay in payment of the claim is cre
ated by the failure of the insurer to pay the claim within the time fixed in both Sections 243 and 244. Furt
her, Section 29 of the policy itself provides for the payment of such interest:
Settlement of claim clause. The amount of any loss or damage for which the company may be liable, un
der this policy shall be paid within thirty days after proof of loss is received by the company and ascertai
nment of the loss or damage is made either in an agreement between the insured and the company or b
y arbitration; but if such ascertainment is not had or made within sixty days after such receipt by the com
pany of the proof of loss, then the loss or damage shall be paid within ninety days after such receipt. Ref
usal or failure to pay the loss or damage within the time prescribed herein will entitle the assured to colle
ct interest on the proceeds of the policy for the duration of the delay at the rate of twice the ceiling prescr
ibed by the Monetary Board unless such failure or refusal to pay is based on the grounds (sic) that the cl
aim is fraudulent.

The policy itself obliges petitioner to pay the insurance claim within 30 days after proof of loss and ascert
ainment of the loss made in an agreement. The amount due private respondent was P842,683.40 on Apr
il 2, 1985. Finman had until May 2, 1985 to pay Usiphil’s insurance. For its failure to do so, the CA and th
e trial court rightfully directed petitioner to pay, inter alia, 24% interest per annum.

S-ar putea să vă placă și